Você está na página 1de 175

SPE 3rd Edition Solution Manual Chapter 13

New Problems and new solutions are listed as new immediately after the solution number. These new
problems are: 13.A12, 13.A13, 13.D3, 13.D5, 13D6, 13D10, 13D22, 13D30-13D34, 13D36-13D42,
13.E2, 13.E3, 13.G1, 13.G2 . Chapters 13 and 14 from the 2nd edition were rearranged to place all the
extraction material into chapter 13 and the material for other separations in Chapter 14. Thus, the
numbers of many problems have changed.
13.A3.

The amount of solvent should be increased. This will decrease F/S and move the mixing
point M towards S. As a result the saturated extract product E N will be moved down
(less solute). The difference point will be moved towards the triangular diagram. The
combined effect will be that fewer stages are required. By adjusting F/S a condition
requiring exactly two stages can be found.

13.A5.

The vertical axis will be the extract phase and the hypotenuse will be the raffinate phase.
These will be connected by tie lines. Usual procedure can be used.

13.A7.

Situation where E = R and point is at infinity. All operating lines are parallel.
However, this does not correspond to minimum number of stages in extraction.

13.A.11 c.
13.A12. a. C will be spread out and go into both raffinate and extract streams.
b. C will concentrate around the feed edge. If C is very dilute in the feed, can concentrate C.
Then by stopping the feed but continuing to flow solvents, solutes A and B can be
removed. Solute C can now be collected by withdrawing a stream near the feed stage.
13.B1.

Specify:

T, p, z A , z B , F, x Ao , x B , y A N , y BN plus:

y B1 , R, E, N F
x B N , R, E, N F
x A N , R, E, N F
R, E, y B1 , x A N
N, N F , y A1 , E
N, N F , x B N , R
N, N F , x A N , R, etc.
Could also not be given one of standard variables (such as solvent concentration).
13.B2.

a).
One can build stages which are cross-flow (e.g. see Figure 12-12) within a countercurrent cascade. This effectively increases stage efficiency. Not that upward flowing less dense
liquid will be mixed.
b.)
Build chambered stages within a counter-current cascade to prevent mixing of the
dense liquid and give better cross flow on each stage.
c.)
Put in baffles to prevent MIXING of both less and more dense liquids. This will be
more effective if counter-current is arranged so that flow across stages is always in same
direction (see sketch)

319

13.C.7. Start by defining and the coordinates of as:


E o R1 , x A
E o y Ao R1x A1 , x D
E o y Do
nd
rd
Removing from 2 and 3 equations we obtain

Assume that E o

Rj

xA

E o y Ao

R 1 x A1

Eo

R1

(13-43a)

xD

E o y Do

R 1 x D1

Eo

R1

(13-43b)

R 1 . Next write the three independent mass balances around stages 1 to j.

Ej ,

R1x D1

xA

R j 1x A j 1

E j y Aj ,

xD

R j 1x D j 1

E j y Dj

These equations are now in a form similar to the form of the mixing equations developed
previously. To develop the three point form of a straight line use the first equation to remove
from the other two equations, solve for R j 1 E j in each of these equations, and finally set the
results equal to each other. The development proceeds as follows:
Use

Ej

R j 1 to remove from the other mass balances.

Ej

Rj

xA

Solve for R j

R j 1x A j 1
Ej ,

Rj
Ej

E j yA j , E j
yAj

xA

xAj 1

xA

Rj
Rj
Ej

xD

R j 1x Dj 1

yDj

xD

xDj 1

xD

Finally, set these equations equal to each other.


yA j x A
yDj x D
yA j x A
rearrange to:
xAj 1 xA
xDj 1 xD
xDj xD

yAj 1

xA

x Dj 1

xD

E j y Dj

This last equation says that the slope of the line between the points

xD , xA

y D j , y A j and

is equal to the slope of the line between the points x D j 1 , x A j 1 and x D , x A

and

thus the lines are colinear. Furthermore, the lever-arm rule is valid for this system.
13.D1.

a. If we have a single column with only pure solvent then

320

E
44, R

is on op. line. E

2.273
0.0037 . Thus, cannot get to x o

Op line intersect equilibrium at x


b. Now E

and point x N , y N

x N yN 1
E
100 , Slope R E

0.001, 0.0
0.012 .

74 and R E 1.35 .

44 30

44 0

0.001, but y N

30 0.004

0.00162
74
At x 0.001, equilibrium value of y 1.613 x 0.001613 . Alternative works, but
have pinch point and need very large number of stages.

Still want x N

c. This alternative (Say use 25 kg/min of 0.004 butanol)


44 0 25 .004
yN 1
0.00145 which is below equilibrium point.
69
Now
R E 100 69 1.449 m equil 1.613 Thus, this will work. Obtain

0.00145 69

0.012 100

0.001 100

0.0174
69
Op line closer to equilibrium require lot more stages.
20 0.004
If use 20 kg/min of 0.004 butanol: y N 1
0.00125
64
0.00125 64 0.011 100
R E 100 64 1.5625 , y1
0.01844
64
Will also work. Becoming close to pinch at top equil y1* 0.019356
y1

If 15 kg/min, y N

15 .004
1

0.0010, R E

59
1.0010 59

y1

0.011 100

100 59 1.6949

0.1964

59

y1*

m eq

0.19356

Wont work.
Thus, there is a small range where option c will work, but with many stages.
13.D2.

20, E

20, x IN

Kremser equation

Eq (13-11)

x F , y IN

0, m

20

mE

8.333 20

y1
yN

y1 8.3333 x F

y
1

*
1

y1*

8.3333 x F

8.3333, b

0.12 , y1*

R
mE
N 1
R
mE
1-0.12
1- 0.12

0, N

mx 0

2
8.3333 x F , y N+1

7.3460 x F , y1

0.9873 x F

321

Rx F

Mass balance,

Recovery = 1 x N x F

Rx N

Ey1 , x N

0.01269 x F
20
0.9873 , which is higher than 0.963 obtained in cross-flow.

13.D3. New Problem in 3rd edition.


From M.B.

R .013
Where R = 100 and the unknowns are E and yout.

y out

and Equilibrium:

1.613 x out
E

E .001

1.613 .007

R .013 .007
y out

20 x F 19.746 x F

R .007

0.01129

100 0.006

0.001

E y out

0.01129 0.001

58.309 kg h

Alternative Solution: 1 Equilibrium Stage

y 1.613x

1.613
y

y out

0.01129
R

from graph

.001

x out

.007

0.001 0.01129

0.013 0.007
R
1.715

1.715

58.309

.013

Another alternative solution:

322

E, y1

x0

y2

y1

Or

Op. Line:

Eq.

R, x 0

R E

Slope

y1

Points x1 , y 2 ,

R
E

x0

x 0 , y1

But x1 and y1 unknown

x
where y 2

x1

On Op. Line

R, x1

E, y 2

y2

y1,in

y1,IN

x1

T & E in this configuration

R
Slope
known, N = 1
If Eq. line is straight, can Use Kremser with N=1.
E
Both representations are correct. Treating similar to a flash is easier.
13.D4.

Since concentrations are low, use wt. fractions and total flow rates.
Equilibrium: y 0.828 x or m 0.828

550 lb h, E

mE R

700 lb h, x 0

1.0538 and R mE
*
1

b
n

0.828 .0097
mE
1
R

yN

R
1

x0

xN
n

.0003

550

0.0003,

.00046

0.0075

0.00803

y N 1 y1*
y1 y1*
n

y1

0.00046, y N

.94893

mx IN

Kremser Eq. (13-11b), N

0.0097, x N

R
mE

0.0097

700
.0077316
.0538
.0004716

mE
R

550
700

1.5038
33.6

n .94893

13.D5. New problem in 3rd edition.


Part a) Can do with Kremser eq or graphically.
y m x b, m 0.828, b 0
R
400
R 400, E 560,
mE
0.828 560

x0

0.005, x N

0.0003, y N+1

0.0001,

0.862664
mE

.862664

1.159

323

Since

1.0 , can use equation such as 13-11b


mE
y1* m x 0 b 0.828 0.005 0.00414

y N 1 y1*
y1 y1*

mE
R

n 1 1.1592
N
where

y1

yN

x0

E
1.52637

mE
R

R
mE

0.0001 0.00414
0.0034571 0.00414
xN

1.1592

n0.862664
400
0.0001
0.005 0.0003
560

0.003457

10.332
0.14773
Alternate solution: Eq. (12-28) becomes
L

R, V

yN

x *N

E , N

0.0001

b)

x *N
x *N

x0
xN

R
mE

0.000120773 , N

0.828

yN

R E

0.0001

xN

y1
Part b.

x0

E
100

yN

R
1

0.005

140
Kremser Eq.

slope
x0

0.0003

13.D6. New problem in 3rd edition.


Part a. Ey N 1 Rx 0 Ey1 Rx N

y1

min

0.005

10.332

.14773

0.828 x 0
y1*

Slope = 0.828

n mE R

1.52637

y1*

Equil.
y

R
mE

min

yN

x0

E MIN

xN

0.828 0.005

0.00414

0.00414 0.0001
0.005 0.0003

R 0.85957

400
0.85957

0.85957

465.3 kg h

Ext. M.B.

xN

0.0002

100
140

0.0005

0.003414

324

Convert

yN

E, x *N

100

0.0002
1.208

0.00016556

0.5913
mV
mE
1.208 140
Lots of different forms can be used.
n
N

For example

0.4087

N
Part c.

x *N
x *N

L
mV

n mV L
n

Becomes

x0
x0

L
1
mV

x0
xN

R
mE

x *N
x *N

R
mE

n mE R

0.005 0.00016556
0.0005 0.00016556
1
n
0.5913

0.5913

1.8717
0.5254
Eq.

y EQ

3.6

y 1.208x

1.208x

0.00604

R
E MIN

y N 1 0.0002
y 1.208x
xN
R

0.0005

slope

x0

0.005

0.00604 0.0002

1.29777
0.005 0.0005
R
100
E MIN
77.05
1.29777 1.29777
Maximum extract out y EQ x 0
0.00604.
Part d. The roles of extract and diluents are switched in the two problems, which changes the definitions
of y and x.
E MIN

13.D7.
Equilibrium:

30, R

500, y N

0.828x, m

0.0002, x 0

0.828, x *N

yN

0.0111, x N
.828

0.00037

0.00024155

325

Since rather dilute and linear equilibrium use one of the Kremser equations.

n
N

Where

E
500

x *N
x *N

R
ME

(12-28 (modified))

mE
n
R

x0

x *N

xN

x *N

R/mE
1.21

83.756 . Solution is trial-and-error.

Calculated N
Negative-Not possible Need

0.8626
.929
.9435
.945015

700
650
640
639

xo
xN

R
mE

R
mE

17.01 E too high


26.175 E too high
29.84 E too high
30.30 E too low

By linear interpolation need E ~ 639.6 kg/h. Can use other forms of the Kremser equation.
Was 13.D10 in 2nd edition.
x is raffinate R L
Convert Kremser
y
y extract, V E
13.D8.

a)

Use 12-31

xN

x *N

Other forms OK x 0

*
N

x *N

xN

x0
1

b)

yn

KE
R
KE
R

1 mV L
mV
L

K, b

1 KE R

N 1

R
E

x0

30.488 25
100
30.488 25

R x0
xN

0.00001376

100

Can use External balance or Kremser to find y out

y1

KE
R

Ey N

0.00092

Ey1

13.D9.

mx

100
25

y1

xN
0.00092 0.00001379

0.003625

Assume very dilute, R = 1500 kg/h, E = 750 kg/h


Equil. Y K d X becomes y K d x
From Table 13-3. K d,oleic
99% recovery oleic:

m oleic

4.14, K d,linoleic

.99 .0025 1500

md,linoleic

2.17

y1,oleic 750 y 1,oleic

.00495

326

Use Kremser, Eq. (13-11b).

y1*

m oleic E

4.14 750

1500
4.14 .0025

m oleic x 0,oleic
n

For linoleic acid:

yN

0, N

y N 1 y1*
y1 y1*

mE
R

1500

m lin E

750 2.17

Can use Eq. (13-11a):

y1 .00651
Recovery of linoleic:

m L x 0,L

yN

y1*

mE
R

5.44

.9216 ,

2.17 003
R
mE
N
R
1
mE
.07834

.00651

y1*

y1

0.01035

R
n
mE

5.44, y1*

2.07

.00651

.40866
Re c .003 1500

.00124796 y1linoleic

0.00526

.00526 750 Rec = 0.877

th

13.D10. New problem in 4 edition.


Analytical or graphical solution OK.
Stage 1
F1x F1 E i
Equilibrium

x 2,out

1.02 x1

Fx F1

1.02 E R1 x1

Mix with Feed 2

1.02 E

R1

1.02 50

R 1 x1

E2

y 2,in

E 2 y 2,out

y 2,out

1.02 x 2,out

0.0099338 100
R2

R 3,in

R 1 x1

100 0.015

F xF

R 1 x1
1.02 E 2

Ey1,out

y1,out

x1

Stage 2

y1,in

151

100

0.0099338

R 2 x 2,out , R1

0.006579, R 2

R2

R1

F1

F1

100

100

100 70 170

327

x 3,in
x 3,in

Stage 3

x 3,out

x 4,out
y 4,out

0.006579 100

0.005 70

170

0.0059286

E3

y3,in

E 3 y3,out

R 3 x 3,out

1.02 x 3,out

R 3 x 3,in
1.02 E 3

R 4 x 4,out
y 4,out

x F2 F2

R 3,in

R 3 x 3,in
y3,out

Stage 4

x 2,out R 2

E4

170 0.0059286
R3

y 4,in

1.02 50

E 4 y 4,out

170

0.00456

R 4 x 4,out , R 4

R3

170

1.02 x 4,out

R 4 x 4,out
1.02 E 4
1.02 x 4out

170 0.00456

R4

51 170

0.003508

0.003578

328

329

13.D11.

2501, E

Equilibrium: K D

E 1000

1.57 . For dilute this becomes m

xN R

Abietic Acid Recovery:

xN

.0475

.0475

2501

Top op. Eq.:

Goes through pt x 0
Bottom Op. Eq.: y

0.0000190 , y1

R
E

yN

R
1

y1

x3

0.17594
Rx in

.0475

.05 F x F

.05 1.0 .05

1000

2500

1000

0.0000025

y 4 0, y*1

1.2399

0.00742

Ey in

2.5

x N through point x N , y N

1 1.2399

0 0.00742

y1
Overall bal.

0.00742

.95 1.0 0.5

x0

m 1.613, R mE 1.2399 , y N
Eq. (13-11a)

13.D13. a.

y1

0, y1 . Slope

E
Need 8 stages (see Figure).

13.D12.

.95 F x F

K D in wt. frac. units.

Ey out

mxin

0 , R E

0.00742

0.17594

0.00742
10 0.0046

0.006114
5 0.006114
10

0.1 1000 0.003

xy . 90% recovery, 10% left

,out

2.501

0.00154

0.3 kg out

0.0003

330

O xy

For ortho, y max

95% recovery, 5% left

0.15 0.005

0.25 , x O,out

0.00025

0.00075
R
E

For para

0.05 1000 0.005

0.00075 0
max,ortho

y max

0.08 0.003

0.00024

0.00024 0

0.003 0.0003

max,para

b. The p-xylene recovery controls.

E 1.5 11250 16875 ,

0.1579

0.005 0.00025

1000

E min

0.08888

0.08888
11, 250

0.0592592
E
Can use Kremser eq. (13-11b) for -xy to find N

n
N

mE
1
R

y N 1 y1*
y1 y1*

mE
R

R
mE
m 0.080, R E 0.0592592, y N 1 0 , y1* mx o,p
0.080 0.003 0.00024
Mass balance: 90% entering -xy leaves w. solvent.
0.9 1000 0.003
y1
0.00016
wt frac
16,875
R
0.0592592
R
mE
.080
1.35
0.74074, n
0.300106 ,
R
0.0592592
mE
0.080
mE
n

331

0 0.00024
0.00016 0.00024

.35

1.35

n 0.30

4.012
0.300106
0.30016
Note: Can use other forms of Kremser eq if desired.
c. For o-xy check if recovery > 95%
R
1
*
y1 unknown, y N 1 0
y1 y1
mE
Eq. (13-11a)
N 1
y1* mx 0
0.15 0.005 0.00075
y N 1 y1*
R
1
mE
R
0.0592592
0.39506, N 4.012
mE
0.15
y1

yN

y1*
1

R
mE
N
R
mE

External M.B.

y1*

Ey1

R xN

Rx 0

xN

0.39506

5.012

0.00075

0.0029194

R x0

Ey1
R

a)

Ey N 1

% Recovery
13.D14. (was 14.D4. in 2nd ed.)

1 0.39506

0.00075

Ey1
Rx 0

16875 .00029194
1000
100

10.0

MF

15.0

SM

7.3584 E 5

98.53%

Once have M, use trial-and-error to find tie through M. (final result is shown). This
gives E and R. y A .115, yw 0.04, xA .23, xw .73.
b) Plot raffinate, R x A

.1 . Find tie line through this point (not trial-and-error). This gives
E. Draw Line ER. Intersection with line SF gives M.
S
S
MF
. Find S 85.7 kg/h.
F 15.0 SM

332

13.D15.

Since dilute, use Kremser equations. Assume units are weight fractions.
a) Column 1 at 40C. x N 0.0008, N 11,, x 0 0.01, E 1000,, R 100
Equilibrium: m

0.1022, thus y1*

mx 0

0.001022. Kremser (Eq. 13-11a):

1
1.022
0.93664
12
y N 1 0.001022
1
1
1.022
This simplifies to: y1 .093664y N 1 .00092628
y1

0.001022

External MB: y N 1E Rx o
which simplifies to:

yN

y1E Rx N , y N
1

1000
Solve 2 eqs and 2 unknowns: y1,coll
b) Column 2 at 25C: y N

y1,col2

yN

1,col1

1,col2

y1,col1

.6929 10 5 , x 0

1000

1 1000 y1

.08

1000 y1 .92

0.00092693, y N+1,coll

0.6929 10

0.00092693 ,

0, N

9, m

0.0328, E 1000, y1*

mx 0

Use Kremser to solve for R. This is trial and error. For example, Using Eq. (13-11a),
R
R
1
1
*
0.0328 1000
y1 y1
mE
0.007475
N 1
10
y N 1 y1*
R
R
1
1
mE
32.8

50

60

50.5

50.35

RHS 0.007855 0.001981 0.007307 0.007467


Within error R = 50.35
y N 1E R x 0 y1E .92693 0 .006929
xN
R
50.35

0.0183

333

c) Could be practical if ms were larger, and have bigger shift in m. A similar scheme is used
commercially for citric acid. Not practical here since have to pump around too much
solvent. In addition, benzene is carcinogenic and would probably not be used as solvent.

R E 10 8 1.25, R mE

13.D16. a.)

*
1

y
y1

m x A0

1.613 0.01

0.01613

0.0002 0.01613
xA

1.25 1.613 0.77495


0.01613. Use Eq. (13-11a),

1 0.77495
1
x A0

0.77495
E

yN

E
1

R
R
b.) Graphical check works fine (not shown)

yj

13.D17.

x6
Note:

x6

R
Ej

xj

y IN

0.27044 y1

R
E0

y1

x j 1,

7.02498 E

10

Ej

0.01182
4

0.0018 (See graph)

x N,countercurrent

0.000702 even though use more total solvent.

334

13.D18. (was 14.D2. in 2nd ed.)


Lever arm rule:

Plot S, F, R and E. Draw lines SF and RE. Intersection is point M.

MF

20.3

SM

4.5

Or Mass Bal. S + R = M and S y A


Solve simultaneously

4.511 S

F xA

100 4.511

M x A ( S .15

451.1 kg/h

.5 F .21 M )

S = 483.3

335

Difference is due to accuracy in reading numbers. Lever-Arm Rule more accurate!

13.D19.

Equil.

Kd

Acetone

y0
xN

FD

1000 .9

yA x A
0
1

0, x 1

0.10 wt frac

900 kg/h water, FS


FD FS

Equil.

Y0

0.287 0.158 1.816

900
1364.1

0.005
X N+1

X1
0.10

.9
1371 .995

0.005
0.995

0.00503

0.1111

1364.1 kg/h chloroform.

0.6598

336

XA

xA
0

yA = 1.86 xA

YA
0

0.01
0.03

0.0101
0.0309

0.01816
0.05448

0.01850
0.0576

0.05

0.0526

0.0908

0.09987

0.07

0.0753

0.1271

0.1456

0.09

0.9890

0.1634

0.1954

0.1

0.1111

0.1816

0.2219

External M.B.

FD
FS

XN

Y6

FD
FS

X1

YN or YN

0.6598 0.1111

0.06999, y N
Results pretty close to 13.D43. 2

1
2

vs 2

2
3

0.6598 .00503

YN
1 YN

0.0655

w i accuracy of graphs.

Note: The graph below should read acetone, not acetic acid as the solute.

337

13.D20. a) Batch Operation Mix together & settle. Find fraction recovered:
R
R
Operating Eq.:
y
x
x 0 , R 5, S
4, x 0 x F
S
S
Which is,
y
1.25 x
1.25 x F
Equilibrium

8.333 x, m

Eq. (13-21) written for batch

8.333

R S x 0
m

Frac. Rec 1 0.1304 0.8696


b) Continuous solvent addition:
S
1
n x t ,final x t ,feed
Eq. (13-28)
R t m

x t,final x F
Recovery = 99.87%.

exp

0.8 8.33

y iN

1.25 x F

R S

0.8

9.583

1
8.333

0.1304 x F

x t ,final
xF

0.00127

338

13.D21 (Was 14.D1 in 2nd ed.) a. Let A = methylcyclohexane and D = n-heptane.


Mass Balances: F1 F2 S M or M 350

F1 x AF

Then

F2 x AF

S y AS

x AM

F1 x A F

F2 x A F

F1 x D F

M
F2 x D F

x DM

M x AM , F1 x DF

F2 x DF

S y DS

M x DM

S y AS

100 .6

50 .2

S y DS

350
100 4 50 .8

350

0.2
0.229

Plot M. Find tie line through M. (See figure.) This gives location of points E and R.
Find x DR 0.48, x AR 0.42, y AE 0.06, y DE 0.05 .

b.

Mass balances: M

E R and Mx AM

Ey AE

Rx AR

Solving simultaneously: E = 214 and R = 136 kg/h


13.D.22. New problem in 3rd edition.
1
Af
D s2 4 0.411 and Pperf
2

With interface at center, heavy phase flow area is


1
D s D 5 2.630
2

Chord

.1

.4115

Ds 2

0.5115

Center

Interface

(length = C)
arc

.1

C/2

.1

C
2

1.00326 m
339

Draw right triangle for interface below center to calculate new perimeter.
0.1
.1
sin
.1955
11.274
r
.5115
Then angle of arc,
180 2
157.452
3.14159 0.5115 157.452
r
Length of
arc
1.4056
180
180
Perf C arc length 2.4089m
Mensuration formulas are from CRC Standard Mathematical Table.
Re settler

4Q

Perf

4 0.006 998

2.4089 0.95 10

10, 466

Interference somewhat more likely than in Example 13-5.


13.D23 (was 14.D7. in 2nd ed.)
Pyrdine
F x AF
Plot M on line FS .
y p 0.223,

a) F S 500 300 M
S y AO 500 .3 0 M x AM x AM

150 800

0.1875

By T & E find tie line through M (Use Conjugate line)


x p 0.84 ; y w 0.02, x w 0.84 ;

Mass balances: R1

E1

800 , 0.84R

0.02E

0.43M

Solve simultaneously, E1 ~ 400, R1 ~ 400 (Note: More accurate than pyrdine values.)

R 1 S2

b)

R1x A1

S2 y A0
60

x pyr M 2

700

Find tie line by T & E: y pyr2


MB:

R 2xw2

E2 yw 2
R2

Solve simultaneously: E 2

400 300

700

400 0.15

0.053 ; y w 2

M x m2w 0.945 R 2
M

60

M 2 x AM 2

0.086

0.120; x pyr2

E2

M2

0.005, x w 2

0.005 E 2

0.945

700 0.48

700

346 and R 2

354

340

341

13.D24 (was 14.D10. in 2nd ed.)


a) Feed 40% MCH 55% n-heptane, F = 200. Solvent 95% aniline
& 5% n-heptane, Stotal 600 . S F M 800

Lever arm rule:

FM
MS

. Find M (Easy way is divide line FS into 4 parts)

Use tie line through M to find points E & R (T & E)


Extract: y MCH ~ 0.045, Raffinate:x MCH ~ 0.36 wt fracs
Mass balance E + R = 800 = M and lever arm rule
Solve simultaneously:
b)

gives points R1 and E1.

Find:

F S
R1

MR

ME

. Measure distances on figure.

R = 124.61 kg/h, E = 800 R = 675.39

2 stage cross flow. Stage 1: F = 200, = 300,

Mass balance 500

R1

FM

MS

. Find point M. Tie line through M

E1 and lever arm rule

207.04 kg h , E1

R1

E 1M 1

E1

M 1R 1

292.95

Note: Isotherms are very sensitive. Thus, calculation is not extremely accurate.
Stage 2: Mass balance R 1 S2

M2

507.04

R2

E 2 and lever arm

S2

M 2R1

E1

M 2S 2

Find M 2 and from tie line through M 2 find R 2 . Then can find R2 and E2 from mass balance
(given above) and new application of lever arm rule,
Solving simultaneously, R 2

R2

E 2M

E2

R 2M

196.16 kg h. E 2

310.88

342

13.D25 (was 14.D9. in 2nd ed.) a. Draw lines from S to F and from R 1 to E N . Intersection gives point
M (see Figure). Then from lever-arm,

b.

FM

SM

1.25 S

is at intersection of lines E N R N

1.25 2000

2500

and E 0 R1 . Then step off stages as shown. Need 2 stages.

343

13.D26. (was 14.D6. in 2nd ed.)


Guess a value for M and step off stages. Repeat until need 3 stages.
After three trials found M shown in Figure. This required 3 1/10 stages which is close enough.
Extract Composition: Acetic Acid = 10.5%, Water = 3.5%.
Raffinate Composition: Acetic Acid = 5%, Water = 93%
Solvent Flow Rate: F S F
Raffinate Flow Rate:

R1 E 0
EN

Extract Flow Rate:

SM SF 15 57

F S R1

13.D27 (was 14.D12. in 2nd ed.)

y AE

E0

MF

E0M

Lever arm rule:

1.112

y wE0

R1

R1

R 1 5600, R 1

2000 5600 772

6830

0 (Pure solvent)

. Step off stages

211.2 kg/h & lever arm:

Solve simultaneously, R1

770 kg/h.

. Find M. Line RM intersects satd extract at E N , y A N

Lines F E N & R1E 0 intersect at


M.B. E N

E0

2000 S 2000 S = 5600 kg/h

64.25, E N

0.18

3 more than enough. Need ~ 2

EN

MR 1

R1

ENM

2.287 (from graph).

146.95 kg/h

344

345

13.D28 (was 14.D14. in 2nd ed.)

To find : 1) Plot E N and R N

2)

Ej

Rj

EN

E N x AN

xA

RN

1500

R N 1x A N 1

0.06666

3) is on line through points E N and R N 1 .


Plot . Or, use lever-rule.

RN

R N 1E N
EN

1.5

Step off three stages starting at point E N . This gives points


R 1 x A1

Mass Balance: E 0

0.275, x D1

RN

E 0 0.13

and

R1

E N E0

1000 0.4

Solving simultaneously, R 1
13.D29 (was 14.D16. in 2nd ed.)

0.675 and E 0 y A1

R1

EN

R 1 0.275

655 kg/h, E 0

.13, and y D 0

RN

0.0 .

R 1 1500

2500 0.2

2155 kg/h

a) Plot Points F, S, E N and R 1

Find point at intersection of lines FE N and R 1S


2 stages is more than enough. (see graph)
b)

Draw lines FS and E N NOT calc. value E 2 R1 .


Intersection is mixing point M

dist. S to M

S
F 0.786 1000 0.786 1272 kg/h.

dist F to M

Mass balance
Give S

F + S = M and Lever arm

0.786

346

Alternate: Overall MB, F S M and Diluent mass balance,


650 F x F,D S yS,D M x M,D 0.28 M

Solve simultaneously:

2321 and S

1321 kg/h. But this is less accurate.

13.D.30. New problem in 3rd edition.


Equation (13-59) becomes Qc /Ai < ut /(1 + safety factor).
Using the equals sign and solving for the safety factor Sf we have,
Sf = ut Ai / Qc -1 = 0.00172 (1.0)(4.0)/.006 1 = 0.1467
where Ai = Ds Ls. Thus safety factor is 14.67% instead of 20%. This may still be acceptable.
50
13.D.31. New problem in 3rd edition. Soln. A. Kremser Soln.
R mE
0.30998 1.0
161.3
R 50, E 100, m 1.613, b 0, y 2 0.0, x 0 0.01
For example, Use 13-11.

yN

y1
x1
Soln. b.)

y1*

y1

*
1

R
mE
N
R
mE

y1*

mx 0
becomes

0.01613 0.01613 .7633696

y1 m

0.01613

y1 0.01613
0 0.01613

1 0.30998
1

0.30998

0.7633696

0.00381684

0.00381684

0.0023663
1.613
Do mass balances and equilibrium for single stage.

347

Sy IN Fx F Sy Fx
0 0.5 100y
50x
also y x 1.613 . Solve simultaneously and obtain identical result.
Soln. c.
Do graphically as single stage system.
Soln. d.
Do graphically as counter-current system, N=1. Solution is valid, but awkward.
13.D.32. New problem in 3rd edition. Fixed Dispersed Phase.
Q sol Q feed Q feed
Q tol
At feed conditions
tol
Q sol
Q tol Q feed
Q feed
Q feed
Q feed

0.6 .006

Q sol Q feed
Q sol
.006 .6 .006
1
Q feed
Equation 13-48 operation in ambivalent range.
a)

.6

tol

.3
L

1
0.3

From Example 13.5

0.375

0.625

1.6

Q sol Q feed
1 Q sol Q feed
.375

865 0.95 10
998. 0.59 10

0.3

1.10235

0.375

The

1.10235 0.6614
0.625
Either phase can be dispersed.
1.0
b)
0.5 , also ambivalent range
d
2.0
.5
Either phase dispersed
1.10235 1.10235
.5
2.0
c)
.6667. According to 13-48 at border.
d
3.0
.6667
water probably dispersed
1.10235 2.2
.3333
5.0
d)
.8333 Equation (13-48), water (heavy) dispersed.
d
6.0
0.8333
1.10235 5.5 water dispersed.
0.16667
13.D.33. New problem in 3rd edition. t re s Vliq Qd Qc
1.5 min 90s

Qd

Qc

0.0072 m3 s , Vliq

90 s 0.0072 m3 s

Note that there is a 1 inch air gap at top


Vliq
H t 0.0254 d 2tan k 4 0.648 , H t

Vliq

2d tan k

0.0254

Using Goal Seek d tan k

d 2tan k 4

0.648m3

2d tan k

0.648

0.7489 and H tan k

1.4978

348

13.D.34. New problem in 3rd edition. N = 500 rpm = 8.335 rps


d i 0.20 d tan k
0.2 0.8279 0.16558 m

Use water values for


Re L ,estimate

d i2 N

998 kg m3 and

0.16558

8.335 998

Curve b in Figure 13-32 again predicts a constant N p0


Then from Equation (13-52), P

4.0

8.335

N P0

.16558

40

d 5i g c where g c

1.0

0.95 10 3 kg m s

240, 064

0.95 10

0.034587

1.0 and

8.335.
(A)

will be fairly close to c


998 since Q W 5QToluene (see Equation (13-53)).
W
The series of messy terms for Equation (13-56a) can be calculated. Since the tank dimensions and
physical properties are the same as in Example 13-5, the only term on the RHS of Equation (13-56a) that
is different is P. Thus the result in the same as Equation B in Example 13-5, d 0.0576 P 0.3
(B)
In addition to Equations A and B, we need to solve Equation (13-53)
(C)
1 d c 865 d 998 1 d
M
d d
M

Solving equations A, B and C with Goal Seek we obtain


Then solving Equation C,

0.146 and

0.146 865

d,feed

0.874.

0.854 998 978.6

Equation (B) P 0.3


2.876
P 33.84 W.
d 0.05076
nd
13.D35 (was 14.D11. in 2 edition)
From Eq. (12-46),
E1 K 1
E2K 2
B1 1
, C1
, D1 R 0 x 0
R1
R2
(Eq. 6-6) For 1 < j < N A j

1, B j

(Eq. 12-48) For Stage N A N


Example 13-4: R 0

1000, x A0

For Acetic Acid, K A j

1, B N

0.35, x D,0

E jK j

Rj

ENKN

E j 1K j

, Cj

RN

Rj

, DN

6 , EN

0.65, N

FN z N
1

, Dj

Fjz j

E N 1y N

1475, yA,N

0, yD,N+1

y Aj x A j : Use Fig. 14-4 to estimate K A, j .

K A1
K A4

0.03
0.1
0.12

0.5

0.3, K A 2

0.15
0.14

0.33, K A 3

0.09
0.21
0.16

0.43

0.5, K A 5
0.5, K A 6
0.5,
0.24
0.28
0.32
For first guess assume constant E 1475 and R 1000.
Then
C1
D1

B1

E 1K A1
R1

E 2K 2

1475

R2

1000

R D x A ,0

0.33

1000 0.35

1475 0.3
1000

1.4425

0.48675
350

349

and so forth with D6


1

DN

475 0

0 . Thus matrix for acetic acid is,

-0.48675

1475

1475

1.4425

-1

-1

-1

1000

.33

.43

1000
1

-1

1475
1000

1475

.43

0.5

1000

1475
1000

1475

0.5

1000
1475

1000

0.5

1475

0.5

1000

-1

1475
1000

0.5

0.5

13.D.36. Part a. New problem in 3rd edition. See figure

Forg

C Aq ,0

FAq

*
org ,1

Min

Forg,Min

Forg

b.

0.736 FAq

1.4 147.2

Operating line goes through CAq,N

Corg,1
See Figure.

C Aq ,N
C

*
org ,N 1

0.10 0.008
0.133 0.008

0.736 200 L h

206.08 ,

147.2

Forg

206.08

FAq

200

0.008 and Corg,N

0.736
L
h

1.0304

0.008 with slope 1.0304.

0.097

3 stages more than enough.

~2

3
4

stages needed.

350

Part c.

MW Zr NO3

91.22 4 14.0067 3 15.994

MW water

2 1.00797

15.994

18.00994

351

Basis 1 liter 0.10 mol Zr NO3

Have

33.917g Zr NO3

and

1000 g

33.917

966.083 g water

966.083 18.00994 53.64 mol water


.1
Mole frac.
Zr NO 3 4
0.00186
53.64 .1
33.9179
Mass frac.
Zr NO 3 4
0.033917
1000 g
System is dilute if consider mole fraction, less so if use mass fractions. If densities are constant,
then constant flow rates is valid. Even with variable density, solving problem with mole fractions and
constant molar flow rates would be accurate. This would require converting equilibrium data to mole
fractions. Use of fractions with concentrations in mol/L is NOT correct.
which is

13.D.37. New problem in 3rd edition.


Part a. HETPlarge-scale = HETPpilot (Dlarge/Dpilot)0.38 = (0.24 m) (1.1 m/.05 m).038 = 0.78 m
flarge-scale = fpilot (Dpilot/Dlarge)0.14 = (1.4 s-1)(.05 m/1.1 m)0.14 = 0.91 s-1
Part b. HETPlarge-scale = HETPpilot (Dlarge/Dpilot)0 = HETPpilot = 0.24 m
flarge-scale = fpilot (Dpilot/Dlarge)0 = fpilot = 1.4 s-1
c. Use of the more conservative design developed for difficult systems (n 1 = 0.38, n2 = 0.14) results in a
much higher HETP and thus a much taller column and more expensive column than use of the design
procedure for easy systems (n1 = 0, n2 = 0). Considerably more data is needed for a large variety of
systems to determine best design practice. If a variable speed motor is used in the large-scale system the
difference in predicted optimum frequency is not as serious because the system can tuned to find the
optimum frequency.
New problem in 3rd edition.

13.D.38.

MWwater

18.02,

F 1.0 kmol hr ,

MWtoluene

92.14 , m

0.06 kmol hr.

C toluene

C raffinate

C water

0.00023 ,

x IN

20.8

y IN

x out Fx in / F Sm
Note m m. m is equilibrium in mole fraction units. Assume extract has properties
toluene and raffinate properties of water.
F x IN

Fx out

Sy out and

C extract

kmole benzoic
m 3 extract
20.8
kmole benzoic
m 3 raffinate

Units on m are

y out

m x out

1
865 kg tol m 3

92.14 kg toluene
kmol toluene

1
998 kgW m 3

18.02 kgW
kmol W

122.71

kmol benzoic kmol extract


kmol benzoic kmol raffinate

352

1.0 0.00023

x out

1.0

0.0000275 , y out

0.06 122.71

If use m

20.8 find x out

13.D.39.
Feed is 0.1
1 equil. stage

1 .00023

122.71 0.0000275
1 .06 20.8

0.00337

0.000102, WRONG!

New problem in 3rd edition.


CC 4 , 0.9 AA. F 10 kmol h . Solvent pure. S 10 kmol h.

Lever arm:

10

10

FM
SM

x F,CC

, Alternatively

x M ,CC

x M ,CC

x S,CC

S
F

Then x M,CC 4 0.05


Find Mixing Point M.
[The figure is shown at the end of problem 13D39 as the single stage mixing line.]
Phases split along the line TE to find the line through M
Rafinate: x CC 4 0.041, x AA 0.54 . Extract: yCC 4 0.095, y AA 0.07
Overall Balance:
E+R+=F+S+=20
CC4 Balance:
.095E+0.041R = (0.0) S+0.1 for F=1.0
Solve simultaneously, R 16.6667,
E 20 R 3.3333
NOTE: Since CC4 mole fracs can be read more accurately, the CC4 balance is
probably more accurate than the acetic acid balance equations.

13.D.40.

S1

S
2

CC4
1

E1

R1 = R2 single stage = 16.6667


Mix with S2 = 10 (pure)

E2

x M 2 ,acetic

16.6667

R1

SM 2

10

R 1M 2

x M2,AA

R2

x R 1 ,acetic

x M 2 ,AA

x Sacetic
x M 2 acetic

0.54

x M 2 ,AA

.54 1.6667 2.6667 .3375

Find M2 and by trial and error find a tie line though M2. See figure on next page.
Extract 2,
yCC 4 0.046
y AA 0.065
Raffinate 2,

R2
CC

E2

x CC

x AA

R 1 S 16.6667 10

balance

Substitution

R2

0.018

19.40 and E 2

0.018R 2
0.018 R 2

0.57

26.6667

0.0046E 2

0.041 16.6667

0.046 26.6667 R 2

0.0 10

0.68333

7.16 kmol h .

353

354

13.D41. New problem in 3rd edition.


R N 1 F 10,
x CC 4,N 1 0.1,
x AA,N

E0

S 14.5,

1.0 , y CC

y TEA,0

Mixing. Use lever arm rule.

1.45

x M ,CC

14.5

FM

10

SM

xN

1,CC

0.9
0.091

4,N

xN

1,CC

x M ,CC

S
y S,CC
F
SF 1

x M ,CC

y S,CC

.1

1.45 0

0.041

2.45

Find M. Draw E N MR 1 line. See figure on next page.


Raffinate:

x1,CC

0.008

x1,AA

.58

Passing Streams
E N R N 1 & E O R 1 intersect at .
Very close to parallel. Use parallel lines to step off stages.
Estimate # Stages = 3.
Flow rates
24.5 F S E 3 R 1

CC

balance. F .1
E3

S 0

1.0

E 3 .091

R1 0.008

1.0 24.5 0.008

9.69 kmol h , R 1 24.5 9.69 14.81


.091 .008
Can compare to 13.G.2 Part c.
Extract 10.066 and Raffinate 14.433
Extract Mole fraction y TEA 0.841 x CC 4 0.0913 y AA 0.067
Raffinate Mole fraction

x TEA
Two results are reasonably close.

.418

x CC

0.0056

x AA

.577

355

356

13.D42. a. First, plot points EN and R1 on the saturated extract and saturated raffinate curves,
respectively.
Second, Find point at the intersection of lines FEN and R1S.
Third, step off equilibrium stages. Need about 3. See graph.
Part b. Easiest: use the lever-arm rule. Find mixing point M at the intersection of lines FS and ENR1.Then
S FM
0.81 F
1235kg / h
F SM
Can also write 3 mass balances (overall, pyridine, and water) and solve for the unknown flow
rates F, EN and R1. Unfortunately, this will not be very accurate because it is difficult to read the water
values accurately.
13D.43 (was 14.D5. in 2nd ed.)

Plot points for F, S

Use lever-arm rule to find point M.

E 0 , and R 1 (on saturated raffinate line)

E0M

1000

S 1371
FM
Line R 1M intersects the saturated extract curve at E N . x acetone

0.067 .

Lines FE N and R 1E 0 intersect at (a second piece of paper was attached to find


accurately). Step off stages. 3 is more than sufficient. Need about 2 & 2/3 stages. This
is close to the 2 + estimated in problem 13.D19 with a McCabe-Thiele analysis.

357

358

K Dm

13.E1.
Since

K Do

y m,N
Estimate: E

0.05, K Do
R
1

20, E

200, F 1

1 ortho goes up column and since K D M

yo,N

0, x m,0

E .52F and R

200.52 and R

Recoveries:

0.15, R

20.48,

.92 .52 1
x ortho,N

x o,0

R .48F
R
20

E 200.52
E y ortho,1 or y ortho,1

0.09974 and

E
R

1 meta goes down.

20.48

200

0.1024

0.002386

0.00203

.94 .48 1 Rx meta,N or x meta,N .02179


Plot equilibrium curves and operating lines (see Figure)
Feed cannot be 3rd stage since cannot get x m N desired. Cannot be 5 as will be past
intersection of R E and meta op lines.
Thus feed must be 4th stage. Do not get match of total number of stages.
Need 8 1/3 for ortho and ~ 5 2/3 for meta.
A very slight adjustment of recovery meta will change this. (Meta is approaching a pinch
point at feed stage). 93% recovery was not enough. Therefore, need ~ 93.5% recovery
with ~ 8 stages.

359

13.E.2. New problem in 3rd edition. Part a.

x N,p
Part b.

xy

0.04 .004

Paraxylene:

96% recovery. 4% p-xy left in diluent

0.00016 wt. frac.


y

Ka

0.080

m, E

20, 000, R

x
Eq. (12-28) converted to extraction notation is convenient. L

x0
xN

R
mE

x0
n
N
Part c.

x *N
x *N

mE
R

0.00016,

0.004
0.00016

ortho-xy m

.625

xN

xN

x *N

x0

*
N

yN

x *N

1000

mE

0.080 20, 000

2.3025

0.

Thus

0.006, x *N

0,

0.625

4.899

0.470

0.150, x 0

1000

R
mE

n 1 .625

Eq. (12-31) Converted:

Part d.
m-xy

0.004, x N

.375

mE

0.150 20000

1000

1 mE R
1

mE
R

N 1

1 3

0.006

1.842 E - 5
1 35.899
Alternative Solutions are presented below for meta-xylene.
m 0.050, x 0 0.005, x*N 0, N 4.899 E 20,000,

mE R

1000, b

0.05 20, 000

1
1000
Must use special form. But the L mV 1 form in terms of x is not available. Thus, need to
derive, or translate or find in another source. Looking at development of Eq. (12-12).
N x x0 x N
Solving for N,

x0

xN
x

Where x is determined in same way y was determined for Eq. (12-12),


L
y1
x0 b
y1
V
x x j x j 1 const
x0
L V
L V
Alternatively,

yN

L
xN
V
L V

yN

L V

xN

360

Translating to this extraction problem,

x0

N
And solving for xN, x N

xN

L V
x0

x
x0
0.005
N 1 5.899

R E , yN

0,

xN

xN
xN
0.0008476

Alternative Solution: Redefine terms to match Eq. 12-12 [Relating y to solvent and x to raffinate is
arbitrary. Switch these definitions.]
y N 1 meta xylene in hexane 0.005

y1
m

Kd

0.05

mxy out is unknown

20, b
yN

y1

0, L
N y1

x 0 is now inlet solvent

20, 000;
L
V

x0

20, 000

mV

20 1000

4.899 y1

20 0

1 , V 1000
0

361

Solve for y1,

yN

y1

0.005

5.899

5.899

0.0008476

This is actually x N in normal notation.


Part e. Shown for normal notation.

pxy equil slope = 0.080

y EQ

0.080 .004

0.00032

yN

0
x 0,pxy

x N ,pxy

0.004

0.00016
0.00032 0

Slope Operating line

Slope

E MIN

0.004 0.00016
R
0.08333 , E MIN
0.08333

0.08333
1000
0.08333

12, 000 kg h

13.E.3. New problem in 3rd edition. Part a. Plot the equilibrium data and points F and S. Straight line
from power F to point S passes through mixing point M. Since amounts of F and S are equal,
M is at the half-way point of the line. Find tie line through M by trial-and-error. This is
difficult since tie line is very sensitive.
Approximately, raffinate x AR
and extract y AE
Mass Balances:

0.326

x DR

0.575

0.046

y DE

0.058

E R S E 40
R 40 E
Ey AE Rx A,S Sy A,s Fx AF 20 0

Solve simultaneously, E

18.0 kg , R

20 .4

22.0 kg.

Part b. First add solvent until reach saturated raffinate curve at intersection with FS line.
Initial Raffinate x AR 0.36,
x D 0.54

R INIT x AR
R init

Fx AF SINIT x AS

8 x AR

8 0.36

20 .4

SINIT 0

22.22 kg

SINIT R INIT F 2.22 kg


Second, use Eq. (13-27) for the continuous solvent addition batch extraction.

362

x t ,final ,A

S
R

x t ,feed ,A

dx t ,A
yA

x t,feed,A is the raffinate after solvent addition to form two phases


x t,feed,A

0.36 , x t ,final,A

x A,initial raffinate

0.292

From equilibrium find values y A (extract), Approximate values are:

x A,t

yA
0.048
0.046
0.045

0.36
0.326
0.292
0.292

1y
20.8
21.7
22.22

dx A

0.36 0.292

yA

0.36

Sadded

1.47R t

20.8 4 21.7

1.47

Eq. A

In the derivation R t is assumed constant, R t

Sadded

22.22

R t,INIT

22.22 kg

32.66 kg

With this approximation E Sadded . 32.66 kg


Solute mass balance
R t x A,INIT Sadded y A,added R t x A,final

y A,added

y A ,Avg

0, x A,INIT

Ey A,Avg

0.36, x A,final

22.22 0.36 0.292


32.66

0.292

0.046

If we do not assume R is constant, then Eq. (13-27) is


x t ,A

Sadded

d R x t ,A

dS added
0

yA

x t ,INITIAL ,raf

We would need to do a numerical integration with a calculation of R x t,A versus y A . this


can be done, but is challenging.
13.G.1. New problem in 3rd edition.
Extract 1: flow 3.90769, xTRA
Raffinate 1: flow
Extract 1:
Raffinate 2:

0.84986, xcarbontet

16.03923, x TEA

flow
flow

0.085102, xAcetic acid

0.41361, x carbontet

11.63396, x TEA

0.91426, x carbontet

14.40527, x TEA

0.065042

0.041332, x Acetic Acid


0.036586, x Acetic Acid

0.54506
0.049149

0.41633, x carbontet =0.016472, x Acetic Acid =0.56719 .

Entering carbon tet 0.10 10 1.0 kmoles hr


Leaving in raffinate

0.016472 14.40527

In Out in Raffinate
Extracted
% extracted = 76.27%

0.23728

0.7627

363

13.G.2. New problem in 3rd edition.


Part a, 3 stage cross-flow. All flow rates are kmol/h
Total Flow rate
TEA flow
CCl4 flow
Acetic flow
Extract 1
3.961
3.366
.3371
.2576
Extract 2
11.634
10.637
.4256
.5718
Extract 3
11.052
10.419
.1554
.4777
Raffinate 3
13.353
5.580
.0819
7.693
Carbon tet remaining in raffinate 3 is 0.0819 kmol/h. Since carbon tet feed was 1.0 kmol/h, 0.9181
kmol/h was extracted. Fraction extracted = 0.9181/1.0 = 0.9181.
Part b. 3 stage counter-current with S = 10 kmol/h.
Extract 1
4.9142
3.723
.7242
Raffinate 3
15.086
6.277
.2758

.4672
8.533

Carbon tet remaining in raffinate 3 is 0.2758 kmol/h. Since carbon tet feed was 1.0 kmol/h, 0.7242
kmol/h was extracted. Fraction extracted = 0.7242/1.0 = 0.7242.
Part c. 3 stage counter-current with S set to give same fraction extracted as in part a (0.9181) and outlet
raffinate carbon tet flow rate of 0.0819 kmol/h. This is trial-and-error.
First trial: S = 20 and CCl4 raf 3 flow rate = 0.0289
Second trial: S = 18 and CCl4 raf 3 flow rate = 0.04045
Third trial: S = 16 and CCl4 raf 3 flow rate = 0.0590
Fourth trial: S = 14 and CCl4 raf 3 flow rate = 0.0908
Fifth trial: S = 14.5 and CCl4 raf 3 flow rate = 0.08104
This is close enough.
Final Results:
Extract 1
10.066
8.469
0.9189
.6786
Raffinate 3
14.433
6.031
0.0810
8.321

364

SPE 3rd Edition Solution Manual Chapter 14


New Problems and new solutions are listed as new immediately after the solution number. These new
problems are:14.A3, 14.A4, 14.C5, 14.D6, 14.D9, 14.D11, 14.D15-14D17, 14.E2, 14.E3. Chapters 13
and 14 from the 2nd edition were rearranged to place all the extraction material into chapter 13 and the
material for other separations in Chapter 14. Thus, the numbers of many problems have changed.
14.C.5. New problem in 3rd edition. Part a. y
y, x
x, m 1, F
U, S
where
F, U, S, O, R, E are kg
Eq. (13-27b) becomes
U
U
y
x
x F y IN
O
O
U
and (13-21)
x
x IN y IN
1 U O and y = x
O
Part b.
Eq. (13-29b) becomes
O 1
n x t ,final x t ,feed
U K
Where K y x at equilibrium = 1.0 in washing.

n
14.D1. (was 13D29 in 2nd ed.)

a) Translate eq. (12-28),

U
mO

O, R

U, E

x *N
x *N

x0
xN

U
mO

n mO U

Note: x in wt frac. translates to x in kg m 3 if densities are constant. Densities cancel. For


washing equilibrium is equal overflow & underflow concentrations. Thus, m = 1, b = 0
yN 1 b
H 2SO 4
x *N
y N 1 0, x 0 1.0, x N 0.09
m
U
40
mO
1
0.8 and
1.25
mO
1.0 50
U
0.8

1.0 0
0.09 0

1 0.8

0.8
4.96

n 1 0.8

b) HC Use Eq. (12-31) or (14-8)

xN

x0
N

1
1

xN

x *N

x0

*
N

mO U
mO U

4.96,

N 1

1
1

0.75

mO

1.0 50

40

mO U
mO U

N 1

1 1.25
1 1.255.96
1.25, x *N HC

0.0674 kg m 3
yN

1HC

Alternative:
363

xN

Note:

x0

xN
x0

HC

=0.09
H 2 SO 4

Thus, if one is clever and realizes change will be same for HC & H 2SO 4

since

mO
U

& N are identical , dont need to use Kremser eqn for part b.

14.D2. (was 13.D22 in 2nd ed.)


a. 1000 cc sand = 400 cc underflow liquid. This is about 400 g = 0.4 kg
liquid. Equil: y = x. Use nomenclature of Table 13-4.
U
U
Operating Eq. y j
xj 1
y in
x out
O
O
U .4
Slope
0.8. Goes through point (y = 0, x = 0.002)
O .5
Overall bal.
O yin U x in U x out O y out

.4 0.035

.4 .002

.5 yout y out

.0140 .0008

Need 6 2/3 Stages See Graph (Can also use Kremser eq.)

b. Mass Balance:
Op. Eq.:

U xj
yj

O j y jin
U
Oj

xj

U xj
y jin

0.0264

O jy j
U

Oj

xj

2 slope , x out 0.002 (see graph)


O
Obtain approximately same separation, but use much more wash water.
(was 13D23 in 2nd ed.)
U

14.D3.

.5

0.4, O

0.2,

364

y
1

U=3

U=3

O
y

Basis:

O
2
3
y
0
in 3

in 4

O
y

in 2

2
0

O
2
1
y
0
in 1

1 kg CaCO 3 solids

Feed Mole frac. can be arbitrary. Pick x 0

U x iN

M.B.

O yiN
U

y out

O y out

x iN y iN
O
O
y out , x out at Equil (y = x) line

x in , yin

Point
Slope Op line
See graph.
Find
Recovery

x out

U x out

0.01 as basis

x in , 0 is on op line

x4

0.00127

x0

0.01

0.127

x4

1 0.127 0.873
x0
Recovery is significantly better with counter-current process.

365

14.D4.

(was 13D24 in 2nd ed.)

0.8 0.8 0.2

0.8,

4, O

4000 kg/h

366

U F1

1000

kg
h

dry solids

U F2
UT
In section 2:
Slope

U F1

UT

yj

Intermediate feed at x

Slope

xj

L liquid
L solid

kg

.8

.2

U F2
1

1.0

3200

1600

U F1
O

xF
xj

kg liquid
h

kg liquid

2.5
kg liquid
4800
h
UT
y0
x1
O

4800 4000 1.2 Goes through point y0 , x1

UT O

In Section 1: y j

2000

kg liquid
L liquid
kg solid
2.5
L solid

1.0

0, 0.006

0.02
U F1

yN

U F1 O 1600 4000

xN

y N , x N+1

0.4 . Goes through point

Also intersects Section 2 op. line at feed line. (Or calculate y N from mass balance). Equilibrium
is y = x. Step off stages (see Figure). Need 5.4 equilibrium stages. Opt. Feed is 4 th.

14.D5.

(was 13D25 in 2nd ed.)

F1: 1000

kg dry

0.8, 1

0.2

1
1
kg dry
L solid
h
2.25
0.2
L
L under flow
kg dry 1
1
F2 : 2000
4000
h
2.5 0.2

2000 L
h

L
h

F1 5 wt %

F2 2 wt %

367

2000L

Liquid Volumes:

total 0.8 liq

3200 L liq

4000 0.8 liq


underflow

U0

ON

4000 kg h , y N
x0

144 4800
Ext. MB,

0,

4000

1.2,

4800

ON 1 y N
y1

U0 x 0

1600

U0x0

xN

mx 0

n 1 0.8333
N

5 wt %

where

kg liq
h

1.0 kg L

0.05

kg N a 0H
kg liq

3200 0.02

0.006

UNx N

Convert to Kremser
O
V, U
L, m 1, y1*
Eq. (12-30)

kg liquid

U 0x 0

0.030 , x N specified

2 wt %

h
4800

FT : Total liqd h

L liq

1600

O1 y1
x0

x0

xN

4800
4000

0.030 0.006

0.030, mV L

0 0.030
0.0288 0.030
n 1.2

4000 4800

0.0288

0.8333

0.83333
8.83 or 9 stages

Use 2 feeds!
14.D.6. New problem in 3rd Edition. 2.5 kg wet is 1 kg dry solids-insoluble, and 1.5 kg underflow liquid.
1 kg dry solids
Part a.
10 kg total
4 kg dry insoluble solids
2.5 kg total

1.5 kg liquid

6 kg liquid. , Ov 10 kg liquid.
kg dry solids
Before 1st mixing: 0.05 frac BaS
6 kg liquid 0.3 kg BaS
0.3 kg BaS
0.01875 mass frac in U & Ov.
1st Mix:
16 kg liquid total
U

4 kg dry solids

Settle (6 kg liquid in U)
2nd Mix Pure Water

0.01875

0.1125 kg BaS

0.1125 kg BaS

0.00703 mass frac in U & Ov.


16 kg liquid
Settle (6 kg liquid in U) 0.00703 0.0421875 kg BaS
0.0421875kg
0.00264 mass frac BaS in U and Ov.
3rd Mix Pure Water
16 kg liquid
Part b.
Result is same. Can also be done graphically.
Part c.
Countercurrent. Easiest solution approach is to use Kremser equation.
x N x *N
1 m Ov U
N 1
*
x 0 x N 1 m Ov U

368

External M.B.

3, x 0

xN

x0

x0U

y N 1Ov

y1
14.D7.

0.05, m 1, Ov

30 6

30 6

U x0

(was 13D27 in 2nd ed.)


Operating Eq.:

xNU

xN

Ov

Basis 1000 cc wet sand.


.4

xj

0.0003205

6 0.05 0.0003205 30

xj

y j,in

U
O

vol water

xj

y j wt. fractions
1

.4

O = 0.2 kg. Thus, each operating line has slope

y j,in , x j

x0

0.035, y N,in

yN

xN

0.002, yS,in

y N , y 2,in

1,in

0.009936

1000 cm 3 wet sand

vol sand wet

Each op line goes through pt.

yN

y1Ov

Equilibrium:

yj

0.05 0.00641

6, x *N

30, U

.2

1.0 g

kg

cc

1000 g

0.4 kg

2.

yN

2,in

y N 1 , y1,in

yN

Start at stage N where x N = 0.002. Find y N then work backwards to stage N-2. This
gives inlets for first 3 stages so can then work forward (see Figure). Note: that stages 5 and
N-2 are not connected. 8 stages gives more than enough separation, but 7 is not enough.

369

14.D8.

(was 13D28 in 2nd ed.)


Use Kremser equation
Fsolv Fsolid .95, y mx is equilibrium with m = 1.18, and N = 11.
Recovery is 1 x N

x0 .

Eq. (12-31) becomes x *N

m Fsolv

yN
*
N
*
N

xN

xN

x0

x0

0 .

m Fsolv
Fsolid

1
1

xN

1.18 .95 1.121. Then

Fsolid

N 1

m Fsolv
Fsolid

1 1.121

x0

12

1.121

0.041

Thus Recovery = 0.959


14.D.9. New problem in 3rd Edition. Assume FSolid and Fsolvent are constant despite removal of sugar from solid.

FSolid

FSolid
xF
Fsolv

Eq. (13-21) becomes x


a.

Fsolv

1.0, x F

3.0,

0.055, y solv,IN

y solv ,IN

mE

FSolid Fsolv

13

0.055
1
1.18
0.01211 , y
3
3
b. x = 0.004. Solve for Fsolv .

xm E

Fsolv
Fsolv

FSolid

mE x

L Slurry stream

0.055 0.004

1.0

ySolv,IN

14.D10. (was 13D30 in 2nd ed.)

1.18 .004

G H 2 stream

120 lb h

x
1 x

Yin

G Yin
X out

lb CH 4
lb H 2

Yout
120

10.805 kg

CONSTANT

CONSTANT

100

100 lb h of H 2

Operating Line. Must work in weight ratios. Y

0.0143 wt frac.

mE x

1.18 0.01211

y solv,IN

xF
xF

1.18,

FSolid Fsolv

FSolid

0, m E

1 y
in

in

30
100

X in

L
G

X
x in

1 x in

0.30, Yout

L
G

X in

Yout

0
lb CH 4
lb H 2

out
out

.05
.95

.0527

L X out

0.30 0.0527

0.206 , x out

.206

1 X

1.206

0.171
370

Operating line becomes,


L
Y
X Yout where
G
Equilibrium Curve:

120

100

y = 1.2 x becomes

x
0
.05
.10
.15
.20
.25

Y
Y 1

1.2

X
X 1

1.2 X
1 .2 X

Plot Y vs X
X
Y
0
0
.0526 .0038
.1111 .1364
.1765 .2195
.2500 .3158
.3333 .4286

See Figure for Plot. Need 5 1/8 stages.


14.D.11. New problem in 3rd Edition. 10,000 kg h wet solids,
1
frac vol dry solids.
Basis 1 m 3 wet solids :
Weight liquid + weight solids

1.20 and goes through X in , Yout .

1.0

frac. vol. liquid ,

1000 kg m3 1.0 1

1500 kg m 3

0.4

400 kg
Thus

400
1300

900 kg 1300 kg total m 3 wet solids.

of weight is underflow liquid, U

14.D12. (was 13D32 in 2nd ed.)

Fsolv Fsolid

400
1300

10, 000

kg
h

3076.9 .

1.36

371

Op. Eq.:

Fsolid

Fsolv

Fsolid

y1

Fsolv

Where y and x are kg m 3 . y

xF

x0

m E x is equilibrium.

x 0 , x N 1 .975 x F .025 x 0 , y N 1 0, x *N 0, N 5, Fsolv Fsolid


Can use any of Kremser equations such as Eq. (12-31).
m Fsolv
1
*
xN xN
Fsolid
1 1.36 m
0.025
N 1
6
*
x0 xN
1 1.36 m
m Fsolv
1Fsolid

1.36

Which becomes: 0.1582 m 6 1.36 m 0.975 0 Find m = 1.313


1 1.313 1.36
.025
0.025005 which is OK.
Check:
6
1 1.313 1.36
14.D13. (was 13D33 in 2nd ed.)
a)

Use Eq. (13-21),

R S 10 12.5

R S x 0

0.8, m E

1.18

y in

R S

g L in liqd

, x

g L in solid
Frac. Rec. 1 0.404040 0.5959596
x t ,final
S
1
n
b) Eq. (13-29b)
mE
x t ,feed
R

x t,final

x F exp

1.25 1.18

, equil. y

m E x, y in

0.8 x F

1.18 0.8

0.8
1.98

xF

0.4040 x F

0.228779 x F , Frac Rec = 1 0.228779 = 0.7712

14.D14. (was 13D34 in 2nd ed.)

BaSO 4 coal BaS 2 CO 2


Equil: Soln conc in underflow = soln conc in overflow. Thus really washing
Equil : y x, m 1, b 0

350

2075

kg
h

in sol.

kg
h

, y in

Eq. (12-29)
14.D.15.

1.5

kg solution
kg insoluble solid

0.0, x *N
N

0, y1
n

xN

U0 x0

525 kg soln., x in
xN

O
x *N

x0

n L mV

525
2075

x *0

0.20, x out

0.2 0.00001

0.00001
0.0506, x *0

n 0.00001 .2 .0506
n 525 1.0 2075

New problem in 3rd Edition. With 1000 kg/h dry solids U 1.5 1000

0.0506

6.99 or 7.0

1500 kg h

a) Can use Kremser eq. with large N to find Ov Min or a sketch

372

y1*

Equilibrium is y

y1*

y
yN

U
0

Ov

b. Ov 1.2 Ov Min

1500

Ov

1782

Kremser:
Eq. (12-28)

x0

Min

x0

xN

Min

1 .99 .15

0.15

xN

0.15 0.0015

1500

Ov Min

.15

10101

1.0101

1485

0.0015

1782

0.84175

Ov

xN

0.0015, x *N
n

0, x 0

.15, m 1, U Ov
x0
xN

U
m Ov

x *N
x *N

0.84175

U
m Ov

m
U Ov

0.15 0
.84175
.0015 0
2.81
N
16.33
1
.17227
n
.84175
In theory, can use McCabe-Thiele, but it is difficult to accurately step off this large number of
stages.
U
1500
c.
Ov 2000,
.75
m 1
Ov 2000
n 1 .75 100 .75
N
11.29
1
n
.75
n 1 .84175

N act

15

E overall

N eq

11.29

N sub actual

15

0.753

For m E use N = 15 and change mE with same equation

n
N

.75
mE
n

100

.75
mE

mE
.75
373

Vary mE until N = 15. m E .911


On a McCabe-Thiele diagram this is trial and error. Kremser is much easier.
14.D.16. New problem in 3rd Edition. Part a. U 2 kg, O 2 kg, x IN 0.06, y IN 0
Solution (translation of Eq. (13-21)) is
U
x
x IN y IN
1 U O
1 .06 0 2 .03
O
Part b. Want x 0.005 O is unknown, x IN 0.06, y N 0, U 2 Solve for O

x IN

y IN ,

0.06 0.005

y IN

x IN

14.D.17. New problem in 3rd Edition. K = 1 Eq (1328) becomes

x t ,final

x t ,feed exp

Part b.

2, U

x t,final

O
14.D18.

O
U

2, x t,feed

O
Part c.

2, x t,feed

0.06 e

x IN
x

x
y IN

22 kg water

0.005 0

Part a.

0.06
1

n x t ,final x t ,feed

0.02207

0.06, x t,final

0.005

U n x t ,final x t ,feed

2 n

0.005
0.06

4.97 kg

x in Part a.

O normal batch in Part b.

One equilibrium stage. F 1000, x A N+1

E 0 y A,0

.2, S

662, y AS

y DS

F x A,N+1

0.12 (same as Example 14-2)


E0 F
Plot M. By trial and error find tie line through M (Final result shown in Figure).
y A1 .238, y D1 0; x A1 .078, x D1 .656
x A,M

Flow rates: Diluent balance:

R1x D1

R1

F x D,N+1 x D1

E1

M R1

F x D,N+1

1219.5

1662 1219.5

442.5

374

14.D19.

This problem is essentially a repeat of Example 14-2, except using exactly 3 stages.
Clearly, x A1 0.04 since now have more stages. F, E 0 and M are unchanged. Problem
is trial-and-error. Guess location of R 1 . Find E N and . Step off 3 stages and see if
have correct location of E N .

x A1

14.D20.

0.026 and y A3

The third and final trial is shown in the figure.

0.38.

Although this is leaching, this cross-flow problem is very similar to cross-flow extraction.
We can derive
R j 1 x A j 1 E j,in y A j,in
x A Mj
R j 1 E j,in

M
Stage 1:

Rj

R0

E j,in where R j

1000, E1,in

M j x A Mj

421, x A0

xAj

yA j
yA j

.2 y a1,in

0 , x AM1

200 1421 .1407

Find M on line SR 0 at x AM1 (see Figure). By trial-and-error find tie line through M.
375

This gives E1 and R 1. Find y A1

R1
Stage 2: x A M 2

y A2

.18, x A2
R2

Stage 3: x A M 3

R3

.35, x A1

1421 .1407 .35

0.085
1254.9 421
.058, from tie line , M 2

1675.9 0.085 .18


.058 .18
1305 .058

1421

1254.9

.113 .35

1254.9 .113

.113, M1

1726

1675.9

1305.0

0.044 , y A,3

1726 0.044 .09


.03 .09

14.D21. a. Basis 1 kg mix in underflow: x NaC values

.09, xA3

.03, M3

1726

1323.3 kg/h

0.8 1.0
crystals

0.2 yNaC

Since crystals are pure NaC, NaOH is in liquid only. Since 20% of the underflow is liquid,
x NaOH 0.2 y NaOH . Generate equilibrium table.

376

x NaOH

Soln (y)
Mass frac NaOH

0
0.004
0.008
0.012
0.016
0.020
0.024
0.028
0.032
0.036

0
.02
.04
.06
.08
.10
.12
.14
.16
.18

y NaC

x NaC

.270
.253
.236
.219
.203
.187
.171
.156
.141
.126

.854
.8506
.8472
.8438
.8406
.8374
.8342
.8312
.8282
.8252

Feed is 45 wt% NaC crystals. x values: NaC (soln) = 0.5193, NaOH (soln) = 0.099, water 1-0.51930.099 = 0.3817. Since feed is 55% liquid,
x F,NaOH 0.55 y NaOH 0.099 y NaOH 0.099 0.55 0.18
,
y NaC 0.126
From the equilibrium data
F = 100, S = 20, Plot F & S and find M.
FM
20
,
SM 100 Tie line through M gives E & R.
E RM
1.119
R EM
(measured on figure)
E R 120 1.119 R R 120
R 56.63 kg/min, E 63.37 kg/min

R : Raffinate

0.833

E : Extract y NaC

x NaC , 0.026

0.16, y NaOH

x NaOH

0.135

The underflow is z wt frac crystals (Pure NaC) + (1-z) wt frac solution


y NaC 0.16 is soln in equil

z 1.0

Thus,

1 z 0.16
z

0.333 0.16

0.84
was 80% solids in problem statement.
c.

Same M. Plot

R1

draw line

R1

0.833

M to

EN

EN

R1

80.1% OK

2 stages more than sufficient

120

1.137 R 1

EN

R 1M

103.5

R1

ENM

91.0

R1

120

1.137

377

R1

56.14

R1 : x1,NaC

E N : y NaC

E
63.86
kg/min, N
kg/min
0.845, x1,NaOH 0.01
0.152 y NaOH

0.147

378

379

14.E1a.

This is difficult part converting data


Basis 1 lb oil-free solids

y oil

ysolvent
1.0
0.9
0.8
0.7
0.6
0.5
0.4
0.35
0.3
0.28

0
0.1
0.2
0.3
0.4
0.5
0.6
0.65
0.70
0.72
Note:

ysolids

1.0

x solids

z
0.20
.242
.283
.339
0.405
0.489
0.600
0.672
0.765
0.810

1 z

0.830
0.80515
0.7794
0.74683
0.71174
0.67159
0.625
0.598086
0.56657
0.552486

y oil z

x oil

1 z
0
0.01948
0.044115
0.07595
0.1153
0.16420
0.2250
0.26124
0.303399
0.3222

0 for all streams, Z = lb solution/lb oil free solids.

Plot data on triangular diagram. See Figure 14.E1a, b, c, d, e.


b&c. F + S = M1 = 1500

F x oil,F

S yoil,S

1000

M1x oil,M , x oil,M1

0.252

1500

0.168

See Figure 14.E1a, b, c, d, e.


Check: Lever Arm
Extract E1 ,

y oil,1

Mass Balances:

M 1S

M 1F

0.34; Raffinate: x oil,1

0.092 and x solids,1

1500

0.34 E1 +0.922 R 1

R1

E1

Extract:
MB:

R 1 , 252

1040.3,

Finish step c) Stage 2: R1 S2

x oil,M 2

. Find tie line through M1.

M2

E1

459.7 lb

1540.3

, R1 0.092

M2 xoil,M

0.062 . Plot M 2 and find tie line through M 2 .

yoil

1540.3

0.115; Raffinate: x oil,2


E2

R 2 , 95.7

0.025 and x solids,2

0.115 E 2

0.80 .

0.025 R 2

R 2 904.8 lb, E 2 635.5 lb


d & e Same answer as b & c but R & E are flowrates.
f.

0.730

See Figure 14.E1f. 3 stages is more than enough. Need ~ 2

1
3

equil stages.

380

Lines E N R N

and E 0 R 1 intersect at .

381

382

14.E.2. New problem in 3rd Edition. Converting data is the difficult part, but is obviously identical to
Problem 14.E.1.
Basis 1 kg oil-free solids

x solids

y oil

ysolvent

z
0.20

1.0

x oil

1 z

0.830

y oil z
1 z

1.0

0.1

0.9

.242

0.80515

0.01948

0.2

0.8

.283

0.7794

0.044115

0.3

0.7

.339

0.74683

0.07595

0.4

0.6

0.405

0.71174

0.1153

0.5

0.5

0.489

0.67159

0.16420

0.6

0.4

0.600

0.625

0.2250

0.65

0.35

0.672

0.598086

0.26124

0.70

0.3

0.765

0.56657

0.303399

0.72

0.28

0.810

0.552486

0.3222

Approximate solution, use Eq. (13-29a) Oil balance:

S
Rt

x t ,final

x c ,feed

dx t
y

S = Mass Solvent,
R t Mass raffinate (solids + solute)
x = Mass frac. solute (oil) in raffinate
y = Mass frac. solute (oil) in raffinate in extract (solvent)
a) M is now at saturated raffinate curve. x oil,M

0.21, x solids,M

0.63

Mass balance F + S = M
Solids .748F + (0) (S) = 0.63M

0.748

F 1187.3 kg R initial
0.63
S 187.3 kg
b) Now mixing is from S to a point on raffinate curve.
From equilibrium curve in solution to 14.E.1.
383

x oil

y oil

0.21
0.1625
0.115
0.0675
0.02

1 y oil

.54
.498
.40
.28
0.1

Insoluble Solids M.B.


Initial 0.748, F = 100, Final 0.81,

1.852
2.0080
2.50
3.57
10.0

R t final

.81 R t final 748


R tfinal 923.5 kg
Raffinate is 0.81 solids, 0.02 oil and 0.17 solvent
Solvent remaining in raffinate is 0.17 923.5 157.0 kg
Needs to be recovered by evaporation.
Do Simpsons rule in 2 parts.
0.21 0.115
1.852 4 2.008 2.50
6
1

0.115 0.02
6

Sadded

2.50 4 3.57

.1961

0.4241

10

0.6202

0.6202 R t , but what is R t ?

Eq. (13-29a) assumes R t

Const.

Use average value of R t .

R t ,avg
or

Sadded
Stotal

R t init R t ,final
1187.3 923.5 1055.4
2
0.6202 R t,avg
0.6202 1055.4 654.6

Initial addition + Sadded

Extract amt S

Stotal

Sremain in raffinate

Oil in extract

x F,0.1 F x final,oil R t,final

187.3 654.6

841.9 157.0

841.9

684.9 by solvent

0.252 1000

0.02 923.5

233.5

Total wt extract 684.9 233.5 918.4


yoil 233.5 918.4 0.254

ysolvent

0.746

14.E.3. New problem in 3rd Edition. Solid Matrix is insoluble. Solids = (.748) 1000 = 748 kg. R t not
Constant, but Solid is.
Solids
Rt
x Solids

ydS

d R xA

Solids d

xA
x Solids

384

x final ,A x Solids

S
Solids

d x A x Solids
y

x A ,raf ,init
x Solids ,raf ,init

Changes limits integration.


x oil 0.21, x Solids

x oil

0.63, x oil x Solids

0.115, x Solids

0.21 0.63 .3333

0.705, x oil x Solids

0.115 .705

0.163

x oil 0.02, x Solids 0.81, x oil x Solids 0.02 0.81 0.0247


Numbers for use in Simpsons rule are from Solution 14.E.2.

.3333 0.163
6

0.163 0.0247
6

Sadded
Stotal

1.852 4 2.008

2.50

2.50 4 3.57

Solids total integral

10

0.3515

0.6173

Total 0.9688
748 0.9688 724.6 kg

initial added 187.3 724.6

911.9 kg

Extract Amount Solvent

Stotal Sraf ,final 911.9 157


Oil in extract = 0.252 (1000) 0.02 (923.5) = 233.5
Total weight extract 754.9 233.5 988.4

754.9 kg

wt frac solvent = 0.764, wt frac oil = 0.236

385

Chapter 15 Solution Manual


Since this is a new chapter, all problems are new.
A. Discussion Problems.
15.A1. The mole fraction water is constant but since the temperature within the vessel varies the total
molar density Cm varies and the water concentration = Cw = ywCm also varies. Thus, Eq. (1510a) incorrectly predicts molecular diffusion. Equation (15-10b) predicts no molecular
diffusion because dyw/dz = 0.
B. Generation of Alternatives.
15.B1. For example, one could operate with both inflow and outflow at the bottom of the tube. If flow is
controlled with a constant head tank, the height of liquid in the tube will be very close to
constant.
C. Derivations.
15.C4. Substitute in q = ( Re)/(4) into Eq. (15-35d) and obtain = [(32Re)/(42g)]1/3.
15.C5. Start with Eq. 15-52a), set vB=0 and solve for yAvA. Then NA = Cm yAvA. Substitute in the
expression for yAvA and Eq. (15-52e) for JA. This gives the desired result.
15.C6. This problem is included to show that one can derive the expressions in books. There is a lot of
algebra, but the derivation works. First, can expand the derivative,
1
AB 2 (1 2 x1 x12 )

x1 x [ B ( A B ) x1 ]2
Then take the derivative and expand all terms. The denominator becomes
[ A ( A B) x1 ]3 [ Bx2 Ax1 ]3 and the numerator simplifies to 2 A2 B 2 x2 . Multiply by x1. Q.E.D.
*
15.C7. With CMO and y as mole fraction, vmol
y Av A yB vB y Av A (1 y A )vB 0 . Since NA = -NB,
CAvA = -CBvB and for an ideal gas Ci = yi Cm. The total molar concentration Cm is constant.
Then, vA = -(1-yA)vB/yA (Eq. A)
In terms of mass fractions yA =(yA,mass/MWA)/[yA,mass/MWA + (1 yA,mass)/MWB]. (Eq. B)
Substitute Eq. B into Eq. A and simplify.
(1 y A, mass ) / MWB
vA
vB (Eq. C)
y A, mass / MWA

*
Then in mass terms vmass
y A,mass vA yB ,mass vB which after substituting in Eq. C and simplifying
*
vmass
(1 y A,mass ) ( MWA / MWB )(1 y A,mass ) vB . (Eq. D)
*
If MWA = MWB, vmass
= 0. We can write
vB = NBCB = NByBCm = NBCm(1 yA) (Eq. E)
where the y are mole fractions. Substituting Eq. B into Eq. E and then substituting this into Eq. D,
we obtain
1 y A , mass MW A

Cm N B
(1 y A , mass ) (1 y A , mass )

MW B MW B

vmass
(Eq. F)
y A , mass / MW A (1 y A , mass ) / MW B
*
Since yA,mass varies throughout the distillation, vmass
is different for each stage.

386

D. Problems.
15.D1. Dprop,water = 0.87E-9 m2/s. Eq. (15-9), J A , z D AB

dC A

( D AB / L )(C A , L C A ,0 ) . If C A,0 = 1.2


dz
kg/m3 is the known value, C A, L can be larger or smaller than C A,0 . For smaller C A, L we have

C A, L =1.2 (0.2E-5)(0.0001)/0.87E-9 = 0.9701


If it is larger value, then C A, L =1.2 +(0.2E-5)(0.0001)/0.87E-9 = 1.430
15.D2. Taking the ratio of Eq. (15-23c) at the unknown T and at T =298.16,
exp[ Eo / (TR )]
= 1.52E-09 for T = 335.18K. Flux
D (T ) D (298.16)
exp[ Eo / (298.16 R )]

J A , z D AB

dC A
dz

( D AB / L )(C A , L C A ,0 ) (1.52 10 9 / 0.0001)(0.9701 1.2) 0.35 10 5

The temperature can be found with Goal Seek from a spread sheet, but one has to trick Goal Seek
into working. Multiply the desired and the calculated fluxes by 1,000,000 and have Goal Seek
match these two values.
15.D3.a. 0.181cm2/s, b. 0.198 cm2/s, c. 0.0725 cm2/s, d. 0.198 cm2/s
15.D4. a. 0.0875 cm2/s, b. 0.096 cm2/s, c. 0.175 cm2/s, d. 0.096 cm2/s.
15.D5. Use Arrhenius form in Eq. (15-23c) but for mole fraction 0.0332 instead of infinite dilution. Write
the equation for both known temperatures and divide one of these equations by the other. The
constant Do divides out. Take the natural log of both sides and solve for E/R. The result is
D (T ) 1 1
E / R ln AB 1 /
D AB (T2 ) T2 T1
The constant Do can be found from the known conditions at T 1
Do DAB (T1 ) / exp[ E / ( RT1 )]
Or from the known conditions at T 2. The results are: E/R = 1348.3, E = 2677.6 cal/mol, DAB
(x=0.0332, T=300) = 1.31310-9m2/s.
15.D6. Same equations as in 15.D5. At 298.16 K for the infinite dilution value set C sucrose = 0. Final
results are Eo = 4953.8 cal/mol, DAB(infinite dilution, T = 320K) = 0.92510-9m2/s.
15.D7. For an ideal solution the term in brackets in Eq. (15-22) is equal to 1.0. Write this equation for
two of the xA values with the corresponding diffusivities (e.g., x = 0.0332 with D = 1.00710-9
m2/s and x = 0.7617 with D = 1.22610-9m2/s). Then have two equations with the two unknowns:
o
o
o
o
and DBA
. Solve for the two unknowns. Results are DAB
= 0.99810-9 m2/s and DBA
=
DAB
-9 2
1.30810 m /s. Check results with the other two mole fractions and find that the fit is good.
15.D8. From http://www.engineeringtoolbox.com/ the density of methanol at bp is 750.5 kg/m3 (used a
linear interpolation), which means partial molar volume = 1/(density/MW)= 0.0426 kg/m3.
Viscosity of water is 1.0 cp = 0.001 Pa s = 0.001 kg/(ms).
a. With B = 2.26, DAB = 1.4310-9 m2/s.
b. With B = 2.26, DAB = 1.5610-9 m2/s.
387

2
15.D9. Combining Eqs. (15-35b) (15-35d), vvertical ,max,liq 0.5 9 gq / Assume that the bulk is pure
1/3

water with infinite dilution of ethanol. From Perrys Chemical Engineers Handbook, 8th edition,
(p. 2-305) at 1.0 bar (0.1 MPa) water has W,m,liq = 55.212 kmol/m3 W,liq=994.64 kg/m3 and
W,m,vapor = 0.032769kmol/m3 W,vapor = 0.5903 kg/m3. The water boils at 372.76K. At this
temperature, from p. 2-432, the viscosity of liquid water in Pas is,
W ,liq exp[52,843 3703.6 / T 5.866 ln T (5.879 1029 )T 10 ] 2.807 104 Pa s
The viscosity of the vapor at 372.76K is (p. 2-426)
W ,vapor (1.7096 108 )T 1.1146 1.2561105 Pa s or kg/(ms).

Now we can calculate the vertical velocity of the liquid water for q = 7.510-6m2/s (remember to use
liquid properties).
1/3

9(994.64)(9.81)(7.5 E 6) 2
vvertical ,max,liq 0.5 9 gq / 0.5
0.130 m / s
0.0002807

A check of the units show they work. The modified Reynolds number (using gas properties) is,
2

Re

d tube ( v gas vliq , y ,max )

1/3

(0.10)(0.5903)(0.81 0.130)
1.256 10 5

3195.6

/
The gas phase Schmidt number is Sc gas
The viscosity and density were found earlier.
D EW gas
The diffusivity of ethanol and water in the vapor phase at 372.76K and 1.0 bar = 0.98717 atm can be
estimated from the Chapman-Enskog theory with the parameters in Table 15-2. This value of DEW =
1.65810-5 m2/s. Then Scgas = 1.283. Since both Re and Scgas are in the range for Eq. (15-47a), the

modified Sherwood number is,


k p d tube ( pB )lm
0.0328(Re) 0.77 Scgas 0.33 0.0328(3195.6).77 (1.283).33 17.79
DAB ptot
15.D10. From the Chapman-Enskog theory DNH3-air = 2.0510-5m2/s at 318.16K and p = 1.2 atm.
Problem 15.D10a, 3rd ed.
MW A
28.9 MW B
17 const
1.86E-07
T
318.16 p
1.2 T^3/2
5675.033
sigma A
3.711 sigma B
2.9 sigma AB
3.3055
eos A/kB
78.6 eps B/kB
558.3 eps AB/kB 209.4812
kT/EpsAB
1.5188 Col integ
1.197 Linear interpolation table 15-2
D AB
2.05E-05
D, cm^2/s 2.05E-01
The concentration at z = L is CNH3 (L) = CNH3 (z = 0) + JNH3L/DNH3-air. Results are 0.0002483 kmol/m3
and 0.0002117 kmol/m3.
15D11. D = JL/C = 4.11410-5m2/s. Set up spreadsheet to obtain this value. Since the collision integral
was entered manually, had to do several iterations. After 6 iterations T = 396.2K (see
spreadsheet, and note that collision integral does not exactly match the value of kT/AB.

388

Problem 15.D11, 3rd ed.


MW A
28.9 MW B
T
396.1642 p
sigma A
3.711 sigma B
eos A/kB
78.6 eps B/kB
kT/EpsAB 1.891168 Col integ
D AB
4.11E-05
D desired 4.11E-05 chkB7-B8
chk x E5
Problem
MW A
T
sigma A
eos A/kB
kT/EpsAB
D AB
D desired

17 const
1.86E-07
0.9 T^3/2
7885.202
2.9 sigma AB
3.3055
558.3 eps AB/kB 209.4812
1.1069 Linear interpolation table 15-2
1.90E-10
1.90E-05 Goal seek to zero changing B3

15.D11, 3rd ed.


28.9
396.164199034186
3.711
78.6
=B3/F5
=F2*F3*SQRT(1/B2+1/D2)/(D3*F4*F4*D6)
0.00004114

MW B
p
sigma B
eps B/kB
Col integ

17
0.9
2.9
558.3
1.1069

const
T^3/2
sigma AB
eps AB/kB
Linear interpolation
in Table 15-2

0.0000001858
=B3*SQRT(B3)
=0.5*(B4+D4)
=SQRT(B5*D5)

chkB7-B8 =B7-B8
chk x E5 =100000*D8 Goal seek to zero
changing B3

15.D12*. From http://www.engineeringtoolbox.com/ viscosity is 1.0 cp = 0.001 Pas = 0.001 kg/(ms).


At 298.16 K, DAB = 1.114 m2/s. Density water = 998.3 kg/m3, viscosity water = 0.001 kg/(ms).
Calculate = 0.000115282 m, vy,avg = 0.04338 m/s, Re = 19.966. This is a long residence time
with Re< 20 so there are no ripples. Shavg = 3.41 and kavg = 3.295E-05 m/s, and 0.000168 kg/s
carbon dioxide are absorbed.
15.D13. From http://www.engineeringtoolbox.com/ viscosity is 1.0 cp = 0.001 Pas = 0.001 kg/(ms). At
298.16 K, DAB = 1.114 m2/s. Density water = 998.3 kg/m3, viscosity water = 0.001 kg/(ms).
Calculate = 0.0001663 m, vy,avg = 0.090241 m/s, Re = 59.898. This is a long residence time,
laminar flow, with no surfactant so there are ripples. Shavg = 5.8 and kavg = 3.89E-05 m/s, and
0.000198 kg/s carbon dioxide are absorbed
15.D14. From http://www.engineeringtoolbox.com/ viscosity is 1.0 cp = 0.001 Pas = 0.001 kg/(ms). At
298.16 K, DAB = 1.114 m2/s. Density water = 998.3 kg/m3, viscosity water = 0.001 kg/(ms).
Calculate = 0.0007717 m, vy,avg = 1.9441 m/s, Re = 5989.8. This is turbulent flow with 1300 <
Re < 8300. Scliq = 899.2, Shavg = 255.5 and kavg = 0.0003689 m/s, and 0.00188 kg/s carbon
dioxide are absorbed
15.D15. From http://www.engineeringtoolbox.com/ viscosity is 1.0 cp = 0.001 Pa s = 0.001 kg/(ms). At
298.16 K, DAB = 1.114 m2/s. Density water = 998.3 kg/m3, viscosity water = 0.001 kg/(ms).
Calculate = 0.000115282 m, vy,avg = 0.04338 m/s, Re = 19.966. This is a short residence time
with Re< 20 so there are no ripples. Shavg = 9.942 and kavg = 9.61E-05 m/s, 7.851E-09 kg/s
carbon dioxide are absorbed.
15.D16. From http://www.engineeringtoolbox.com/ viscosity is 1.0 cp = 0.001 Pas = 0.001 kg/(ms). At
298.16 K, DAB = 1.114 m2/s. Density water = 998.3 kg/m3, viscosity water = 0.001 kg/(ms).

389

Calculate = 0.0001663 m, vy,avg = 0.090241 m/s, Re = 59.898. This is a long residence time,
laminar flow, with surfactant so there are no ripples. Sh avg = 3.41 and kavg = 2.28E-05 m/s, and
0.0001165 kg/s carbon dioxide are absorbed.
15.D17. Used a spreadsheet set up to solve Example 15-6. For = 0.001 meter one obtains xNH3 =
0.04988, yNH3,surface = .21593, Nwater = 0.5393, NNH3 = 0.0228307. The concentrations are the same
as in Example 15-6, but the fluxes are 10 larger.
15.D18. Part a. For two part solution need values at xE = 0.25 and 0.35. The average molecular weights
are calculated as in Example 15-5, and are used to determine the average molar densities. The
Fickian diffusivities are estimated by interpolating between values given in the Table in Example
15-5. The activity coefficients are determined in the same way as in Example 15-5. Then the
Maxwell-Stefan diffusivities are found by the same method. The values are listed below
MWavg

DEW, m2/s

m ,kmol/m3

DEW , m2/s
XE = 0.25
25.0
36.28
0.63310-9
1.9028
1.49510-9
-9
XE = 0.35
27.8
31.62
0.62510
1.5553
1.60910-9
Write Eq. (15-61c) for both intervals. For z from xE = 0.2 to 0.3 we obtain (values at xE = 0.2 and 0.3
are in Example 15-5),
36.28(1.495 10 9 )[1.7083(0.3) 2.1582(0.2)]
zN E
9.3445 10 9
1.9028(0.25)
From xE = 0.3 to 0.4 (interval is over length - z) we obtain,
31.62(1.609 10 9 )[1.4338(0.4) 1.7083(0.3)]
(0.00068 z ) N E
5.7027 10 9
1.5553(0.35)
Adding the two equations to remove the unknown z and then solving for NE and z,, we obtain
NE = -2.12810-5kmol/s and z = 0.0004223m
Part b. Since the interval z is greater than the interval z = 0.0002577m, we subdivide the interval
from xE = 0.2 to 0.3 into 2 parts. The values needed are given below.
MWavg
XE = 0.225
XE = 0.275

DEW, m2/s

m ,kmol/m3

24.3
25.7

37.625
34.99

0.65910-9
0.62410-9
z1 N E 5.5371 10 9

E
2.01976
1.79959

DEW , m2/s
1.48210-9
1.53210-9

Equation (15-61c) is now written 3 times: z 2 N E 3.9846 10 9

( z1 z 2 ) N E 5.7027 10 9
and solved for the 3 unknowns z1, z2, and NE. Obtain NE = -2.238910-5kmol/s, z1 = , 0.0002473, and
z2 = 0.0001780m.
15.D19 (Optional, Unsteady diffusion) At the average C = 0.001 mol/L Dsucrose 0.5228 105 cm 2 / s .
Equation becomes

CA
C A,0

1 erf

z
4(0.5228 10 5 )t

Numerical values of C A / C A,0 are easily obtained with a spreadsheet or with the use of Table 17-7.

390

z, cm
0
0.01
0.05
0.1
0.2
0.3
0.4
0.5
0.6
0.7
0.8
0.9
1.0
1.2
1.4
1.6
1.8
2.0
3.0
3.5
3.56
4.00
5.0

t = 1000 s
1
0.9221
0.6249
0.3281
0.0505
0.00335
9.16E-05
1.009E-06
7.61E-12

t = 10000 s
1
0.9753
0.8771
0.7571
0.5362
0.3535
0.2161
0.1220
0.6352
0.0304
0.0134
0.00538
0.00198
0.000206
1.49E-05
7.49E-07
6.2E-10

t = 100000 s
1
0.9922
0.9610
0.9221
0.8449
0.7692
0.6957
0.6249
0.5574
0.4936
0.4346
0.3788
0.3281
0.2406
0.1710
0.1177
0.0784
0.0505
0.00335
6.20E-04
4.99E-04
9.16E-05
1.01E-06

Part b. C 1.0 10 6 when C / C0 5.0 104 , which for t = 100000 s occurs for a thickness of <3.56
cm (Goal Seek gives 3.559 cm). Thus, at this time a layer 3.56 cm or thicker appears to be
infinitely thick.
Part c. C 1.0 10 6 when C / C0 5.0 104 , which for =0.10 cm occurs at t = 78.938 s (done with
Goal Seek on spreadsheet).
H. Spreadsheet Problems
15H1. Let A = air, B = hydrogen, and C = ammonia. Then NC = -NA NB. Substitute this expression
into Eqs. (15-65a, b)

m y A
z

m y B
z

y
y
y
B C A
D AB D AC D AC

yA
y
A
NA

D AB D AC

NB

y
y
y
y
y
B B NA A C B NB
DBA DBC
DBA DBC DBC

Determine NB from the 1st equation and NA from the second.

391

m y A
NB

m y B
NA

y
y
y
B C A
D AB D AC D AC
yA
y
A

D AB D AC

NA

y
y
y
A C B
D BA D BC D BC
yB
y
B

D BA D BC

NB

Put these equations and the values for mole fractions at the boundaries, diffusivities, m, and z = into a
spread sheet. Guess a value for NA,guess, calculate NB and NA,calc, and use Goal Seek to make
NA,guess - NA,calc = 0 by changing the value of N A,guess.
Results Nair = -6.209E-5, NH2 = 14.026E-5, and NNH3 = -7.817E-5 kmol/s. As expected hydrogen
diffuses in the positive direction and ammonia in the negative direction. The surprise is the
substantial negative diffusion rate of air. (Spreadsheet shown in 15.H4, but with different
numbers.)
15.H2. Used a spreadsheet set up to solve Example 15-6. For a bulk gas that is 40% air, 15 % NH3 and
45% water obtain xNH3 = 0.05596, yNH3,surface = .24225, Nwater = 0.032964, NNH3 = 0.001955 kmol/s
15.H3a. See solution to 15.H1 for procedure and 15.H4 for example spreadsheet.
Results: Nair = -1.87E-8, NH2 = 2.98E-7, NNH3 = -2.80E-7 kmol/s.
b. For ammonia Deff = 1.5656E-5 m2/s. Estimating dC/dz with the difference approximation for a very
dilute solution, N = J = -Deff m y/(z)= -(1.5656E-5)(0.08928 kmol/m3)(.002)/(.01m) = -2.80E7 kmol/s. Thus, this is accurate. For hydrogen and air Dair-H2 = 3.0550E-5 m2/s. Then
Nair = Jair = (3.0550E-5)(0.08928 kmol/m3)(-0.001)/(.01m) = 2.73E-7 kmol/s. The same value is
obtained for hydrogen. The hydrogen value is close, but the air value is not close. Conclusion:
Use the Maxwell-Stefan approach.
15.H4. See solution to 15.H1 for procedure. Results: Nair = -5.903E-5, NH2 = 14.069E-5, NNH3 = 8.166E-5 kmol/s. Note the substantial negative diffusion of air despite the zero driving force.
The air is dragged along with the ammonia. The spreadsheet is given below (labeled as 15.D22),
first with the numbers, and then with the formulas.

392

HW 15-D22,
A=air, B=H2,
T

3rd Ed. SPE


C = NH3
273.000000000 p

D AB 1 atm
D AB

0.000061100 D AC 1 atm
0.000030550 D AC

NA guess

2.000000000

0.000019800 DBC 1 atm


0.000009900 DBC

0.000074800
0.000037400

-0.000059028

del z

0.010000000

yA z=0
yA z=
yA avg
yA/z

0.520000000
0.520000000
0.520000000
0.000000000

NB
NA calc
chk NA-Nacalc
100000 chk
NC

0.000140693
-0.000059028
0.000000000
0.000000000
-0.000081665

y B z=0
yB z=
yB avg
yB/z

0.089278949
0.480000000 y C z=0
0.000000000 yC z=
0.240000000 yC avg
-4.285389534

0.000000000
0.480000000
0.240000000

kmol/s
kmol/s
Goal seek to zero
change B10

HW 15-D22,
A=air, B=H2,
T

3rd Ed. SPE


C = NH3
273

D AB 1 atm
D AB

0.0000611
=B7/D3

D AC 1 atm
D AC

0.0000198
=D7/D3

NA guess

-0.0000590279043468439

del z

0.01

=D3/(0.0820575*B3)

yA z=0
yA z=
yA avg
yA/z

0.52
0.52
=(B14+B15)/2
=-D12*(B14-B15)/B12

y B z=0
yB z=
yB avg
yB/z

0.48
y C z=0
=B9*B11/D3
yC z=
=(D14+D15)/2
yC avg
=-D12*(D14-D15)/B12

NB
=(B17+(D16/B8+F16/D8+B16/D8)*B10)/(B16/B8-B16/D8)
NA calc
=(D17+(B16/B8+F16/F8+D16/F8)*B19)/(D16/B8-D16/F8)
chk NA-Nacalc =B20-B10
100000 chk
=100000*B21
NC
=-B19-B20

DBC 1 atm 0.0000748


DBC
=F7/D3

=1-B14-D14
=1-B15-D15
=(F14+F15)/2

kmol/s
kmol/s
Goal seek to zero
change B10

393

SPE 3rd Solution Manual Chapter 16


New Problems and new solutions are listed as new immediately after the solution number. These new
problems are: 16.A1, 16.A7, 16.A8, 16.C2 16C4, 16.C5, 16.D3, 16.D9, 16.D16 to 16.D22, 16.G116.G3, 16.H1-16.H2. Chapter 16 was chapter 15 in the 2nd edition. Problems from that edition
have the same problem number, but the chapter number is now 16 (e.g., problem 15.D6 is now
16.D6).
x
, Operating y = x,
16.C3. (was problem 15C3 in the 2nd edition.) Equilibrium y
1
1 x
y out

n OG
y in

dy
y

. Substitute in for y* and let x = y (total reflux operating line)

dy
y
1 y

n OG
1
y out

n OG

y y 1

y in

y out 1

y2 y2
1 y

y y
1

dy

n OG

dy

y out

y out

1 dy
y 1

y in

y in

y in

1 n

y out

y in

which becomes Eq. (16-81).

16.C4.

New problem in 3rd edition. With extract dispersed,


E MD

Since

y IN

y IN

y M,OUT

y IN

y*M,OUT
y M,OUT

0, E MD

*
M,OUT

Mixer mass balance with y IN


Solving for x M,OUT :
Then,

y*M ,OUT

E M ,D

0, Fx IN
F x IN

x M ,OUT
m

F x IN

m x M,OUT

S yM,OUT

F x M,OUT

S y M ,OUT
F

S y M ,OUT
F

y M ,OUT
y

where y*M,OUT

*
M ,OUT

y M ,OUT
m F x IN

S y M ,OUT
F

y M ,OUT

E M ,D m x IN
1

mS
E MD
F

395

External MB y IN
E S,D

0 F x IN

yS,IN

yS,OUT

yS,IN

*
S,OUT

S yS,OUT

Equil : y*S,OUT

Substituting: y*S,OUT
y S,IN

E S,D

Substitute in

y S,IN

y S,IN

y S,OUT

E MD m x IN
mS
1
E MD
F
E MD

y S,OUT

m x IN

m x S,OUT
S

m x IN

y S,OUT

S
y S,OUT
F

x IN

y M ,OUT

F x S,OUT

yS,OUT

y S,IN 1 E SD
1

m E SD x IN

mS
E SD
F

and do some algebra to obtain,


mS
1
F
mS
1
E SD
F

E SD 1 E MD
mS
E MD
F

The definition of the total stage efficiency is,


E total,D

Equilibrium:

x D IN

x D ,S,OUT

x D IN

*
D ,S,OUT

y*S,OUT

E total,D

x IN

y IN

y S,OUT

y S,OUT

y IN

*
S,OUT

y *S,OUT

S
F

y S,OUT
m x IN

with y IN

yS,OUT . Substitute into definition.


y S,OUT x IN

S
y S,OUT
F

m 1

S y S,OUT
F x IN

Which after substituting in yS,OUT and doing some algebra, becomes.


E MD
E total
1

m SE MD
F

E SD 1 E MD

m S E MD
F

mS
F

mS
1
F
E MD

E SD 1 E MD

mS
1
F

16.D1. (was problem 15.D1 in the 2nd edition) The corrected value of H G,E

1.41 ft is given in
Example 16-2. From the results of Examples 16-1 and 16-2 and from Eq. (16-38),
396

2.61

0.15

0.15

2.61

0.83 0.85 ft
2.2
2.2
mV
Eq. (16-27a)
H OG
H L H G where V L 8 5
L
The value of m (the slope of the equilibrium curve) varies throughout the enriching section.
From the McCabe-Thiele diagram used to prepare Figure 16-4A values of m were found from
y 0.442 y* .63 to y x D 0.8 y* 0.82 .
H L,E,cor

y
y*
m
H OG
y* - y
1

y* y
Arithmetic Average:

H L,E,Initial

0.442
0.63
0.441
2.01

0.5315
0.598
0.406
1.96

0.621
0.66
0.449
2.02

0.7105
.727
.5144
2.11

0.8
.822
.745
2.42

0.188
5.32

0.0665
15.04

0.039
25.641

0.0165
60.606

0.022
45.45

H OG

2.01 1.96 2.02 2.11 2.42

Geometric Average: H OG

2.01 1.96

2.02

15

2.11 2.42

2.10
2.10

No difference!
To find n OG do Simpsons rule in 2 parts because of the unusual shape of 1 y*

n OG ,1

0.179
6
0.179

n OG ,2

5.32 4 15.04

25.641

25.641 4 60.606

y vs. y.

2.718

45.45

9.353

n OG n OG,1 n OG,2 12.072 , h E H OG n OG 25.35


This is reasonably close to the 26.1 ft estimated in Example 16-1.
The physical properties and use of Figure 10-25
16.D2. (was problem 15.D2 in the 2nd edition) a.
calculated in Example 16-2 are unchanged. Now F = 48.

G flood

141,

F 1

55, Dcol

12

48

14

F 2

20

G flood 2

F 1

Dia 1
b. In Eq. (16-37)

12

F 2

48

Dia 2

20

1.16

0.749 lb ft 2

4.68

5.83 ft

14

2 same .

Estimate

h p ~ 10 ft (we know it will be less than before), SC,V is unchanged, terms in


denominator are unchanged.
H G ,E 1

hp 1

10

hp 2

10

13
13

H G ,E 2

55

10

141

22

13

1.33

0.40 ft

397

In Eq. (16-38),

0.045 ,

0.07 , h p 1

10 , h p 2

22 .

Cf ,L , and SC,L unchanged.


H L,E 1

hp 1

hp 2

0.15

H L,E 2

0.045

10

0.07

22

16.D3. New problem in 3rd edition. In the enriching section,


HETP H OG ln(mV / L) / [(mV / L ) 1] and H OG

0.15

0.83

0.47 ft

H L mV / L H G

With H L 0.827, H G 1.33, m / ( L / V ) 0.63 / (5 / 8), H OG 2.16, and HETP = 2.15 (from
Example 16-2). With the same mass transfer coefficients, but m = 0.577,
H OG [0.577 / (5 / 8)]0.83 1.33 2.10 and HETP 2.10 ln(0.9232) / (.9232 1) 2.19 ft
A 24.4% increase in both mass transfer coefficients gives H L

HG

1.244(0.83) 1.033 and

1.244(1.33) 1.65 .

For m = 0.63, H OG

[0.63 / (5 / 8)]1.033 1.65 2.69 and


HETP 2.69 ln[0.63 / (5 / 8)] / [0.63 / (5 / 8) 1] 2.68 .
With the same mass transfer coefficients, but with m = 0.577, H OG

2.61 and HETP = 2.71.

A 24.4% decrease in both mass transfer coefficients gives:


For m = 0.63, H L 0.63, H G 1.01, H OG 1.65, HETP 1.64
For m = 0.577, H L

0.63, H G

1.01, H OG

1.59, HETP 1.65 .

Clearly, the variation in mass transfer coefficients results in a large range for HETP (from 1.64 to
2.69 feet for m = 0.63) while the small change in m had little effect. To be safe the larger value
of HETP = 2.69 would be used. This is a safety factor of 1.20. Note that Bolles and Fair (1982)
recommend a safety factor of up to 1.70.

16.D4. (was problem 15.D4 in the 2nd edition)

Feed line:
Top: y

L
V

.6,

.6

q-1

.4

L
V

Since we have total reboiler, ys,in

1.5 . Plot this and operating lines (see figure).

x D where
y intercept

Bottom goes through

0.04

xB

L D

0.9

1 L D

1.9

L
D

xD

0.474

0.484

0.04 .

398

dy

nG
For
L HG

V HL

enriching

0.474

y AI y A
from

2.568

V HL
From figure generate following table.

yA
Stripping: .04
.3225
.605
Enriching: .605
.7625
.92
Simpsons rule, n G ,S

.315
6

.565
6

.605, y E,out

.92

33.33

1 2.26

slope

4.17

.8

yAI
yAI-yA 1/(yAi-yA)
.13 0.09
11.111
.46 .1375
7.2727
.63 .025
40.00
.62 .015
66.66
.80 .0375
26.66
.95 .03
33.333

16.D5. (was problem 15.D5 in the 2nd edition) a)


1

1.3

11.11 4 7.2727

66.66 4 26.66

Eq. (16-81), n OG

y E,in

section,
Eq.
(16-16)
draw
line
of
1.3
.770 to get y I and x I . For stripping section slope is
0.8

L HG

n G ,E

, where y S,out

7.55 , h strip

40

10.8 , h E

HG n G

Total Reflux. y out

.956 1 .65
.65 1 .956

H G n G,S

1.3 10.8
0.956, yin

9.8 ft.

14.1 ft
0.65

.65 1
.956 1

399

n OG

1.4904
4.0257
b) Finite reflux. Plot op. line. Find y & y* (see graph). Use Simpsons rule in 2 parts.
y

y*

0.783
0.82225
0.8615
.90075
0.94

0.842
0.8795
.9125
.943
.973

y* y
0.059
0.05725
0.051
0.04225
0.033

4.0257, H OG

1
y* y
16.949
17.467
19.608
23.6686
30.303

n OG1

n OG 2

0.8615 0.783
6
0.94 0.8615
6

16.949 4 17.467

19.608 4 23.6686

19.608

30.303

1.3924
1.89166
n OG

3.2841

c) Changes in L/V in equation connecting HETP and H OG

400

401

16.D6. (was problem 15.D6 in the 2nd edition) a.

n OG

11.11 3.56 14.67

H OG h n OG 7.47 14.67 0.509 m


b. From McCabe-Thiele diagram we find the following
y
y*
y* - y
1/(y* - y)

0.016
.0267
0.017
93.46

0.066
0.1067
0.0407
24.57

0.116
0.1815
0.0655
15.27

.494
.623
.129
7.752

.872
.9201
0.0481
20.79

.922
.9523
.0303
33.00

.972
.9832
.0112
89.29

Do integration with Simpsons rule in three parts.


.1
.756
n OG
93.46 4 24.57 15.27
15.27 7.752 4 20.79
6
6
.1
20.79 4 33.0 89.29 15.97
6
The difference is because of inaccuracies in Simpsons rule.
16.D7. (was problem 15.D7 in the 2nd edition) Bottoms x A

xA

0.1 Distillate

.9
0.1 , equil. y

From equilibrium data in 4.D7. @ x A

y out

partial reboiler

xD

0.262

yin With total condenser,

0.9

yA x A

Find average
Bottom

1 yA
0.262 0.1

3.195 ,

0.738 0.9

Geometric Avg.,
Eq. (16-81),

1 xA

.929 0.9
dist

3.195 1.454

avg

n OG

. From data in 4.D7., y*

1
1 2.155

H OG

h n OG

16.D8. (was problem 15.D8 in the 2nd edition)

.071 0.1

12

0.9.

1.454

2.155

0.9 1 0.262

0.929 when x

0.262 1 0.9

0.262 1
0.9 1

4.80

0.42 m
m

H Ptot

22500 855

26.3

402

26.3 .0011

max

.0011 .0001
L
y out
x in
V
x *A ,out

0,

28.93 ,
x out

26.04 . Basis V = 1, then L = 26.04.

act

26.04 .001

26.04

mV

26.316

0.9895

Use Colburn Eq. (16-34b),


1
0.0011 0
n OL
n 0.010488
0.010488
0.0001 0

0.84 9.51

0.02604

.9895

9.51

7.99 m

Can check with Eq. (16-63) and get same n OL


16.D9.

New problem in 3rd edition.


For example, if nO ED 1, we have for the perfectly mixed model,

nO

EMD

ED
1/ 2
1 nO ED
For the plug flow model,
EMD 1 exp( nO ED ) 1 exp( 1)

For same value of nO

ED

0.632

the plug flow model always predicts a higher stage efficiency.

16.D10. (was problem 15.D10 in the 2 nd edition)


y in

n OG
y out

dy
y

. For both cases y*


y in

dy

n OG

y out

For both countercurrent and cocurrent

0, and y y*

y in

y out

0.01
0.0001

y , then
4.6

Note that n OG is same because of irreversible reaction.


c. Flow rates enter into solution only as a check that at least the stoichiometric amount of sulfuric
acid is available to neutralize the ammonia.
16.D11.

(was problem 15.D11 in the 2nd edition) From Eq. (16-72) with irreversible reaction,

n OG

y A in

y A out

50.0 ppm
0.01 ppm

8.517 for both cocurrent and countercurrent.

16.D12. a. (was problem 15.D12 in the 2nd edition)

.013 .00004

G min
L

.0053

G
Use Eq. (16-57),

15 L G

yin

min

0.013, y*in

H p tot

2.7 1.1 2.4545, y

mx

2.4453 (see Figure)

36.679, x out
m x out

y in

y out

L G

0.0003533

0.0008672, yout

0.00004, y*out

0
403

y*

.01296

n OG

b.

0.0121328,

in

y*

out

.0121328

0.0120928

.00004

6.1246 , h

.00004

4.59 ft

Cocurrent. Operating and equilibrium lines shown in figure. Lowest y out is at intersection point
= 0.00081. For y out

x out

y in

y out

L V
2.4545

L V

36.679

n OG

0.00085 ,

.013 .00085
36.679

0.00033125, y*Aout

mx out

0.000813

0.0669184 . Use Eq. (16-20),

0.013 0.000813

1.0669184

1.0669184
h 1.98 ft

0.0669184

0.00085 0.000813

5.496

16.D13. (was problem 15.D13 in 2nd edition) a. Same conditions as Example 16-3. Assume same H OL .
If operation is possible, find n OG & y out .
Dilute Use Eq. 16-70. m = 30.36 (Example 16-3)
y*out m x out b 30.36 0.001 0.03036

mV
L

30.36
15

1
mV
1
L

2.024 , n OG

Lx in

Mass bal.

y out

n OG

L
V

Vyin
x in

y in

Lx out
L
V

mV
L

y in

y *out

y out

*
out

mV
L

Vy out

x out

n 3.024

15 0 0.03082 15 0.001

0.03082 0.03036

0.01582

mV

3.024
0.01582 0.03036
L
Not possible, term inside brackets is negative.
b) Same conditions as Example 16-3 except x out 0.002 . Assume same H OL . If operation is
possible, then
404

y*out

m x out

30.36 .002

y out

15 0

0.03082

n OG

c) Same conditions, except L V


*
out

15 0

n 1.759

1.759

1 exp

Then:

0.8,

0.03082 0.0091
0.01882 0.0091

0.656

0.759

Use equilibrium data shown in Figure 4-16,

n 1 E MV
h A active
30
0.8, K y a
1 2
n 1 .77
2 12

30

0.16, K y a

n 1 .69

.8 2 12

K ya

k xa

k ya

0.415

330.62

k xa

k ya

0.01, m

0.01882

30 and b

2 12 .

K ya

Solving simultaneously, k x a
b. x

still not possible

0.759

K y aA active h V [Eq. (16-77)] where V A c

Eq. (16-6a),

30.36 40

40 0.0003

16.D14. (was problem 15.D14 in the 2nd edition) a.


x 0.8, m 0.415; x 0.16, m 1.5
Mixed: E MV

2.024

0.0003

0.0091 , mV L

0.03082

n OG

0.00082

0.00082 0.06072

40 and x out

30.36 0.0003

y out

15 0.002

0.03082 0.06072

n 3.024

3.024

0.06072

, x

0.16,

330.62 where 1 2

263.5

1.5

263.5

k xa

k ya

1408.19 and k y a

0.8

366.317
142.18 & E MV

6.06. Then from Eq. (16-6a), K y a

0.468 from Eq. 16-77.

16-D15. (was problem 15.D1 in the 2 nd edition) Assume feed to Example 16-4 is satd liquid, z = 0.5, &
2.5 .5
separation complete x D ~ 1, x B ~ 0 . x F z 0.5, y f
0.7143
1 1.5 .5

xD

1,

1 0.7143

Vmin

1 0.5

At x

0.5714 , x B

.5, y intersection op lines

L
V

L
V
Calculate at x

0.6 0
actual

0.5 0

0,
L
V

V
xD

0.7143 0
max

.8 .5

0.5 0
.2 1.0

1.4286
0.6

1.2

0.1, 0.3, 0.7 in example , 0.9

405

x
0.1

Sect

L/V

Strip

1 x

1.2

2.5
1 .15

0.3

Strip

1.2

1.8904

1.1891

2.5

.8

2.5
1 1.05

0.9 Enrich

.8

const.

V
A active h

x
0.1
0.3
0.7
0.9

1.3447
1.767

0.97. Calculate Ept. Solve Eq. (16-78) E pt

and then K y a from Eq. (16-76a), K y a

.6348
0.971

1.0092

1.3447

1.767

0.971

1.0092

0.971
1.3447
0.971
1.767

L
mV

E MV
L mV

n 1 E pt

E pt
0.6348

mV

1.0092

0.453

L
0.6348

0.5949

2.5
1 1.35

E mV

Eq. 15 80

1 .45
0.7 Enrich

0.5891

0.6802

0.731

0.7733

K ya
133.42
171.02
196.9
222.65

16.D16. New problem in 3rd edition. Using Simpsons rule the new value for A1= 7.18, the new value
for A2 = 10.85 and the new total area = 18.03. Then the calculated height of the enriching section
(0.4054 m)(18.03) = 7.31 m compared to the previous result of 7.95 m. This is an error of 8.1 %.
Thus, a rather small error in mole fractions becomes a larger error.
16.D17. New problem in 3rd edition. Relative errors in k G a

24.4% .

Same relative errors in H G and H L .


In the enrichment section the slope of the mass transfer line is
Slope

L HG

0.4054 24.4%

V HL

0.253 24.4%

406

a) If H L correct but H G varies by


Value at

HG

24.4%, range of slope is from

1.242 to

0.761.

0.4054 is slope = 1.0015. At top:

If equilibrium line straight from azeotrope to point x = 0.7472, y = 0.7815, then can fit
this portion of equilibrium as,
y

MT line

If

s 1.242,

If

s 1.0015,

If

0.761,

0.7668x 0.2085
sx

b. Since y

0.8, b

0.8 1 s

b 1.7936
b 1.6012
b 1.4088

All calculations at y A

0.8.

mass transfer line, y

sx

y I at intersection equilibrium, y
b

0.7668x 0.2085 and

407

Substitute in for x,

yI

yI

0.7668

0.2085

Solve for y I
yI

0.7668b
s
.7668
1
s

0.2085

yI

s 1.242, b 1.7936 :

yI

0.81356

yI

0.013565

73.72

s = 1.0015, b = 1.6012: 0.812426 0.01246


s = 0.761, b = 1.4038: 0.80842 0.008419

80.48*
118.78

*83.3 in Example 16.1 since numbers rounded first. Amount of error depends on
distance between equilibrium and operating lines. Less error if closer, but more impact
on 1 y AI y I .
Assume same relative errors:
H L no error, H G higher,
H L no error,

1
y AI

yA

80.48 73.72

error

H G lower,

80.48
1
y AI

yA

8.4%
118.78 80.48

error

80.48

47.6%

Assume error in H is same every point. Thus enriching area can be a lot different than
calculated. But if H G is down by 24.4%, area is up by 47.6% so there is some
cancellation of error.
b. If H L & H G both vary could have s 1.634 and b
yI

Area
H GE

0.493,

1
yI

2.1072 . Thus, y I

0.81493,

= 66.97

66.97

19.6 16.31.
80.48
1.24(0.4054) 0.503 m, z H G,E n G,E

Since
8.20 m which is a 3.2 % increase.

16.D18. New problem in 3rd edition.


a. Equilibrium is y = mx. Value of m is unknown, but Cextract = mconc units Craffinate with

mconc _ units

20.8

kmol _ Benzoic / m 3extract


kmol _ Benzoic / m 3raffinate

. We need m in

408

mol Benzoic / mol extract

mmole fraction units

mol Benzoic / mol raffinate

mmole _ fraction _ units

mconc _ units (

MWextract

)(

extract

. The resulting conversion is,

raffinate

MWraffinate

Since the system is dilute, extract properties are essentially the same as pure solvent and
raffinate properties are essentially the same as diluent.
m = (20.8)(92.14/865)(1000/18) = 123.1
Note that y = xD and EMy = EMD.
EMy = (yin-yout)/(yin-yout*) where yout* = m xout = (123.1)(1.99E-06) = .000245
EMy = (0-.000230)/(0-.000245) = 0.939
b. From Eq. (16-76b), nO-ED = EMy/(1-EMy) = 0.939/(1 0.939) = 15.393
nO
KO

KO

ED

QD

ED

nO

ED

a Vmixer

MWD

ED

QD

, Vmixer

h D2 / 4

(0.75)

0.75 2 / 4

0.331m3

/ (Vmixer MWD )

(15.393)(.0012)(865) / [(0.331)(92.14)]
0.524 kmol / s m 2 mol frac disp
Note that K O

ED

m 2dispersed / m 3total volume

a is larger than in Example 16-5 because the residence time

Vmixer

37.45s is shorter than the 361 s in Example 16-5. Thus, this problem
(Q D Q C ) d
requires much more vigorous mixing.
t res

c. Differential Model

exp

E MD
n OED

n OED
KO

ED

nO

ED

QD

1 exp
1 E MD

n 0.061
D

n OED
0.061

2.797

n OED

2.797

/ (Vmixer MWD )

(2.7969)(.0012)(865) / [(0.331)(92.14)]
0.0952 kmol / s m 2 mol frac disp

d. Use of mixed models: If use K O


nO

ED

KO
QD

ED

a Vmixer

MWD

ED

m 2dispersed / m 3 total volume

0.524 from mixed staged model, then

15.393 and with differential model


409

E MD 1 exp
wrong results.

16.D19.

n OED

1 exp( 15.393) .9999998 . Obviously, mixing models gives

New problem in 3rd edition.


C D in C D out
a. E MD ,Conc
with C*D,out
*
C D in C D ,out

CD,in

0.000 , CD,out

C*D,out

mCD CC,out

nO

.00023

KO

ED

nO

ED

QD

.0002578

nO

ED

1 nO

0.00023 , CC,out

0.0481 CC,out

E MD

b. E MD,Conc

ED

ED

mCD CC,out
0.00536

0.0002578
.892

E MD,Conc

0.892

1 E MD,Conc

0.108

8.26

/ (Vmixer MWD )

(8.26)(.0012)(865) / [(0.331)(92.14)]
0.2811 kmol / s m 2 kmol/m 3

m 2 dispersed / m 3 total volume

c. Differential Model:
E MD,Conc 1 exp n OED exp
n OED
KO

ED

nO

ED

n .108
QD

n OED

2.2256

1 E MD,Conc
n OED

0.108

2.2256

/ (Vmixer MWD )

(2.2256)(.0012)(865) / [(0.331)(92.14)]
0.0757 kmol / s m 2 kmol/m 3

16.D.20.

m 2 dispersed / m 3 total volume

d. E MD,Conc 1 exp n OED


New Problem in 3rd Edition.

1 exp

8.26

a. Eq. (16-92). Terms:

0.167,

Dbenzoic-water

(from Example 16-6)


di

0.2070 16.6667

Note

9.807

0.99974
2.2 10-9

4 3 2 . 6 8

C
CD

5.8632

N in rps

410

di

0.2070

dp

0.0002524

820.13

dp

0.0002524

d tank

0.8279

d pg

0.00030487

865 0.0002524 9.807

96.447

0.0222

kC

0.00001237

2.2 10

0.0002524

x 0.00030487

12

Significantly higher. But Note:


With same k d

0.001905.

96.447

820.13

5 4

0.167

12

0.014801

0.06, so correlation may not be valid. Same

K LD

kD

m CD k C

K LD

0.001905

1
0.014801
20.8

K LD

0.0005181 & K LD a

E MD

5 12

432.68

K OED a

b.

1930.2

2.0586 s

865 2.0568 92.14 19.13


19.31
20.31

which agrees with guess for residence time.

0.951,

Resistance (dispersed) =
Resistance (continuous)
Sum of resistances

1
0.001905
1

524.9

1404.5
0.000712
1929.4 Gives K LD 1 1929.4

0.0005181

% resistance from dispersed


524.9 1929.4 100 27.2%
This is a significant contribution because of significantly higher predicted
continuous phase mass transfer coefficient.
c. If k D is ignored , K LD k C m CD 0.014801 20.8 0.000712
Note that this is significantly too high.
d. To be safe, use lower estimate of E MD . Note that d is really too large for use of
Eq. (16-92).
16.D.21. New problem in 3rd edition. In settler velocity approaches zero
( u t 0.00172 in Example 13-5, but d p 0.000204 not 0.000150 assumed). Thus
u t ~ 0.00495 m s and Eq. (16-88a) is reasonable approximation.
Sh C

k C d Dbenzoic-water

2.0 where in settler d

0.0002524

411

kC

Using

2.0 2.2 10

K LD ~ k C mCD

0.0002524
5

1.7433 10

1.7433 10

1 20.8

8.381 10

ms

If the interface in the settler is at the centerline, then the volume of aqueous phase is

D S2 4 L S . From Example 13-5, DS

1.023m and LS

4DS . Then Volume

aqueous phase 1.682 m3 . The water residence time is


280.3s. Entering total velocity is 0.0072 m3 s and

1.682 m3 0.006 m3 s
d

0.167 leaving mixer. A drop that starts at the bottom of the water phase travels a

vertical distance of DS 2 0.5115 m to reach the interface. This requires


0.5115 m 0.00495 m s

103.2s.

Assume equal distribution of drops. Everything (half the drops) above interface are
collected very quickly. All drops collected in 103.2 280.3 0.368 fraction of settler.
If average over this fraction is 0.5 d,IN , then D,avg 0.5 0.167 0.368 0.031.
6

6 0.031

dp

K OED a

737 and K LD a

0.0002524
D

settler

n OED,settler

K LD a

Vsettler Q D

settler

MWD

0.0058

settler

~ 0.000618

865 0.000618 92.14

3.364 0.0012 0.0058

865 92.14

16.D.22.

1.734

New problem in 3rd edition.

a)

E SD

y M,OUT

yS,OUT

y M,OUT

*
S,OUT

1.7314.

865 92.14

Since settler is not well mixed, E settler,D 1 exp


This is high because of long residence time.

y*M,OUT

0.82,

E MD

0.0058

0.823

y IN

y M ,OUT

y M ,OUT

y IN

*
M ,OUT

y*M ,OUT

0.794

mx raf ,OUT .

From Example 16.5 in mole fraction units m 123.1.


Mixer mass balance with y IN 0 (Example 16.5) is Fx F

Sy M,OUT

Fx C,OUT

In Example 13.5 feed is 0.006 m s and solvent is 0.0012 m3 s


3

0.0012 m3 s 865 kg m3
F

92.14 kg kmol

0.006 m3 s 998 kg m3

x M ,OUT

x feed

S F y OUT

0.01127 kmol s
18.02 kg kmol
0.00026

0.01127
0.3323

0.3323kmol s
y OUT

Then substitute into E MD


y M ,OUT
123.1 0.00026 0.03390 y M ,OUT

0.794
412

Solving, we obtain
x M,OUT

y M,OUT

m E MD

For settler,
Settler M.B. is,

y M,OUT

0.00589
y*S,OUT

Sy M,OUT
x S,OUT

0.00589.

123.1 0.794

m x S,OUT

Fx M,OUT

SyS,OUT

x M ,OUT

x S,OUT

6.027E 5.

y S,OUT

6.027E 5

Fx S,OUT
y M ,OUT

0.03390 yS,OUT

0.00589

Substitute into equation for E SD


0.82

0.00589 yS,OUT
0.00589

123.1

6.027E 5

Solving, we obtain yS,OUT


From Mass Balance,
x S,OUT

6.027E 5

*
S,OUT

m x S,OUT

E Total,D

From 16.C4

E Total,D

0.006174.

0.03390 0.006174 0.00589


y

b)

0.00019967 0.03390y S,OUT

123.1 5.0653E 5

yS,OUT

0.006174

*
S,OUT

0.006235

mS
F

123.1

5.0653E 5

0.01127
0.3323

0.006235

0.99015

4.17495

0.794 0.82 1 0.794 4.17495 1


1

4.1794 0.794

4.17495 0.82

4.17495 0.794 0.82 1 0.794 4.17495 1

.99003
16.G1 and 16.G2. New problems in Chapter 16. Aspen Plus runs showed that N =13 (Aspen Plus
notation) with feed on Nfeed = 11 (optimum location) gave ethanol mole fractions of xD = 0.7990
and xB = 0.020298. These values are within the specified tolerances. The stripping section starts
with the vapor leaving the reboiler (yin,strip = yreb = 0.18709) and ends at the intersection of the two
operating lines. This last value can be determined by calculating the points on the operating lines
(xn, yn+1). For example n = 1, x1 and y2 are on the enriching section operating line. When the
slope changes from 0.61 in the top to 2.07 in the bottom, the intersection point has been passed.
This occurs for yout,strip = y11 = 0.44631.
The mole fractions of ethanol in the liquid and vapor leaving each stage (Aspen stage notation is
used) are:
413

Stage
1
2
3
4
5
6
7
8
9
10
11
12
13

x
0.79904
0.77157
0.74561
0.71971
0.69245
0.66203
0.62575
0.57882
0.51135
0.40050
0.22553
0.10033
0.020289

y
0.81824
0.79904
0.78189
0.76569
0.74957
0.73263
0.71375
0.69129
0.66233
0.62090
0.55339
0.44631
0.18709

Equilibrium: Calculation m
The equilibrium parameter m is the average slope of the
equilibrium curve from x (calculated at y) to xI.
At the reboiler y = 0.18709 for x = 0.020289, and y* =
0.44631 for xoper =0.10033. Then yavg = 0.3167.
The slope can be determined by taking the chord from
x = 0.04 (y*= 0.29209) to x = .05 (y*= 0.33018).
m = (0.33018 0.29209)/0.01 = 3.809.
The equilibrium values are from Analysis in Aspen Plus.
At yout,strip = 0.44631, xoper = 0.22553, y*= 0.55339, and
yavg = 0.49985. m = (0.50405 0.49482)/(0.15 0.14)
or m = 0.923.

To use Simpsons rule for the first integral in Eq. (16-22a) we also need an average m for the y
and y* values calculated at the average between yin,strip and yout,stip, which is y = 0.3167. At this y,
xop = 0.1659 (determined from the stripping section operating line) and y*=0.5174. The average
between y and y* = 0.41705. m = (0.42921 0.41012)/(0.09 0.08) = 1.909. The second integral
in Eq. (16-22a) is the usual estimation from Simpsons rule of the nOG integral,
y A ,out

HG

dy A / ( y A * y A ) = (0.2835/6)(0.44631 0.18709)[1/(0.44631 0.18709) + 4/(0.5174 - 0.3167)


y A ,in

+ 1/(0.55339 0.44631)] = 0.4057 m


The first integral in Eq. (16-22a) can also be estimated from Simpsons rule,
y A ,out

( H L / ( L / V ))

mdy A / ( y A * y A ) = (0.1067/2.032){[(0.44631 0.18709)/6]


y A ,in

[3.8577(3.809) + 4(4.983)(1.909) + 9.3388(0.923)]} = 0.1393


Then h = .4057 + 0.1393 = 0.545 m, which is somewhat more than the 0.507 m calculated in
Example 16-1. Note that the Aspen Plus calculation does not assume CMO whereas the
calculation in Example 16-1 did, but the calculation here required an assumption of how to
calculate m. Thus, it is difficult to say which is more accurate.
16.G2. New problem in 3rd edition.
Enriching section yin,enrich = yout,strip = 0.44631, and yout,enrich = xD =x1 = 0.79904. At yin,enrich =
yout,strip = 0.44631, the calculation of m is the same as done previously for yout,strip: xoper =
0.22553, y*= 0.55339, and yavg = 0.49985; m = (0.50405 0.49482)/(0.15 0.14) = 0.923.
At yout,enrich = xD =x1 = 0.79904, y* = 0.81824, and yavg = .80864; m = (0.81180 0.80481)/0.01 =
0.699.
In Example 16-1 the enriching section integration was done with Simpsons rule in two parts.
From yin,enrich to y = 0.725 and from y = 0.725 to yout,enrich. For the calculation here we will use y =
0.71375, which is in the table of data from Aspen Plus, as a convenient break mole fraction to do
the integration in two parts. For y = 0.71375, xoper = 0.66203, y*= 0.73263, and to find m, yavg =
0.72319. m = (0.72623 0.72101)/(0.01) = 0.522.
414

The average y from yin,enrich = 0.44631 to y = 0.71375 is 0.58003, xoper = 0.44423, y* = 0.63659,
and to find m, yavg = 0.60831. m = (0.61020 0.6067)/(0.01) = 0.350.
For the 2nd integral in Eq. (16-22a) integrated from yin,enrich = 0.44631 to y = 0.71375 we obtain,
y A ,out

HG

dy A / ( y A * y A ) = (0.4054/6)[.71375 - .44631][1/(.55339-.44631) + 4/(.63659 -.58003) +


y A ,in

1/(.73263-.71375)] = 2.404 m
Same integral integrated from y = 0.71375 to yout,enrich = 0.79904. The average y from y = 0.71375
to yout,enrich = 0.79904 is 0.75640, xoper = 0.73068, y* = 0.77246, and to find m, yavg = 0.76443. m
= (0.76587 0.75985)/(0.01) = 0.602.
=(0.4054/6)[.79904 - .71375][ 1/(.73263-.71375) + 4/(.77246 -.75640) + 1/(.81824 - .79904)] = 2.041
Total for integral 2 = 4.445 m
For first integral from yin,enrich = 0.44631 to y = 0.71375 we obtain,
y A ,out

( H L / ( L / V ))

mdy A / ( y A * y A ) = (0.253/0.625)[(0.71375 0.44631)/6][(0.923)(9.388) +


y A ,in

4(0.350)(62.2665) + (0.522)(52.966)] = 1.102


Same integral integrated from y = 0.71375 to yout,enrich = 0.79904,
y A ,out

( H L / ( L / V ))

mdy A / ( y A * y A ) = (0.253/0.625)[(0.79904 - 0.71375)/6][(0.522)(52.966) +


y A ,in

4(.602)(18.2815) + (.699)(52.0833)] = 0.870


Total for integral 1 = 1.972 m
Total height of enriching section = 4.445 + 1.972 = 6.417 m
Total height of packing = 0.545 (from 16.G1) + 6.417 = 6.962 m.
This is less than the total of 8.457 m calculated in Example 16-1. However, it does agree
reasonably well with the number of stages (11) in the column since,
11 HETP (estimated = 0.655 m in Example 16-2) = 7.208.
Both this result and Example 16-1 require calculating a small difference and then taking the
inverse of this number. This type of calculation can cause very significant errors. The graphical
calculation was based on accurate experimental measurements of the equilibrium data, and this
data is probably more accurate than the NRTL correlation used in the computer. On the other
hand, calculation errors are probably larger in the graphical than in the computer calculation.
Both calculations depend significantly on the accuracy of the mass transfer data (HL and HG),
which can easily have errors greater than 20%, which can cause even larger errors in calculation
of NTU and HTU or in HETP (see problem 16.D16) for the staged calculation.
415

16.G3. New problem in 3rd edition.


Part b. Equilibrium stage optimum. (L/D)min = 5.1, L/D = 6.12, N = 33, NF = 14, 1 section, yC4,dist
= 0.008556, xC3,bot = 0.005706, Dia = 1.804 m on plate 32, minimum diameter = 1.565m on tray
14.
Part c. VPLUG optimum. N = 33, NF = 14, yC4,dist = 0.007953, xC3,bot = 0.005302, Dia = 1.812 m
on plate 32, DC backup/tray spacing = 0.391, weir loading = 0.021m2/s. Note that this separation
is better than the equilibrium result.
Part d. MIXED optimum. N = 49, NF = 22, yC4,dist = 0.007651, xC3,bot = 0.00510, Dia = 1.808 m on
plate 48, DC backup/tray spacing = 0.390, weir loading = 0.021m2/s. Note that this separation
require significantly more stages than both the equilibrium-stage and the plug flow results.
16.H1 and 16H.2. New problems in 3rd edition. Fit for the ethanol-water VLE was done in Appendix B of
Chapter 2. The spread sheet for both problems is given below with a y value chosen in the
enriching section (ignore the stripping section operating line values). Overall mass balances to
find D and B were done with Eqs. (3-3) and (3-1). L and V were determined at the total
condenser and flows in the stripping section were determined at the feed stage with the calculated
value of q. L bar = L + q F. The intersection point of the two operating lines was determined
from Eq. (4-38). yreb is the start of the stripping section and is the y value in equilibrium (yeq)
with y = x = xB. In the stripping section the value of 1/(yeq y) was determined at the start of the
stripping section (yreb), at the end of the stripping section (yintersection), and at the average of these
two values. Then Simpsons rule was used to calculate nOG,strip = 1.57787 from Eq. (16-24b). To
determine HOG from Eq. (16-27a) an average slope m of the equilibrium curve is required. The
slope of the chord from the equilibrium y at the intersection point of the two operating lines to the
reboiler vapor that is in equilibrium with xB, mstrip ( y eq ( xint er sec tion) y reboiler) / ( xint ersec tion x B)
= 1.95, and HOG,strip = 1.265. The resulting height of the stripping section, 1.998 feet, is somewhat
more than the 1.66 feet determined in Example 16-1.
In the enriching section a similar procedure was used except the integration to find nOG,enrich was
done in two parts. The average slope of the equilibrium curve was determined from,
menrich ( y1 yequilibrium( xint ersec tion)) / ( x D xint ersec tion) = 0.4558 with y1 xD . The resulting
value of HOG,enrich = 1.935. The resulting height of the enriching section, 29.35 feet, is somewhat
more than the 26.1 feet determined in Example 16-1 and more than the 25.35 feet determined in
problem 16.D1.
Note: in both 16.H1 and 16.H2 the average slope of the equilibrium curve m has to be calculated
with fairly large chords, not by taking the derivative of the 6th order polynomial fit to the VLE.
The reason is that the fit oscillates around the experimental data points and the slopes will
fluctuate greatly.

416

417

SPE 3rd Edition Solution Manual Chapter 17.


New Problems and new solutions are listed as new immediately after the solution number. These new
problems are: 17A2, 17A7-17A9, 17B2, 17C4,17D3, 17D8, 17D10-17D13, 17D15b-h, 17D1617D18, 17E1, 17H2-17H7. Chapter 17 was chapter 16 in the 2nd edition. Problems from that
edition have the same problem number, but the chapter number is now 17 (e.g., problem 16.D6 is
now 17.D6). Figures in the solutions to these old problems still have the designation of chapter
16.
17.A2. New problem in 3rd edition. Change the value of thetatot in the spreadsheet until the area matches
the desired value. This can be done as a manual trial and error or a loop can be added to the spreadsheet.
17.A7. New problem in 3rd edition. Part a. Increase the stirrer speed.
If no gel, increasing stirring increases mass transfer coefficient k which reduces M and hence xw is
reduced. This reduces xp increasing retention R.
Part b. Decrease the stirrer speed. This reduces k, increases M and xw. When xw > xgel, a gel forms and
R increases (probably to l.0) .
17.A8. New problem in 3rd edition. Since there is a gel the retention of the low molecular weight
compound also increases.
17.A9. New problem in 3rd edition. Do not invest. Osmotic pressure can often be ignored in UF because
with large molecules with high molecular weight the mole fraction is always low even if the weight
fraction is high. With low mole fraction the osmotic pressure is low. If there is a concentrated salt with a
low molecular weight the mole fraction will be high and the osmotic pressure cannot be ignored.
17.B1. Look at Suk, D.E. and Matsuura, T. (2006) Membrane-based hybrid processes: a review, Sep.
Sci. Technol. Vol. 41, pp.595626 for additional processes.
17.B.2. New problem in 3rd edition. One possible approach is as follows: Increase stirring to increase the
mass transfer coefficient and reduce the wall concentration to prevent gel formation. Then use a permeate
in series cascade with recycle of the retentate from the second module in series back to the feed of the
first module. The low molecular weight product is the permeate from the second module. The
intermediate molecular weight polypeptide product is the retentate from the first module.

17.D1.

PCO 2

15 10

cc STP cm

10

pr

PCH4

0.48 10

10

PCO2 PCH4 31.25


a) Generate RT curve from Eq. 17-6a.
pp
yp
1
1 yP
pr
yr
1 yP
AB
AB

1 10 6 m

tm

cm 2 s cm Hg

pH

12 atm

pP
CO

AB

pL
2

CH

12 76.0

0.2 atm
4

912.0 cm Hg

15.2 cm Hg

1.0, p p p r

0.016666

y P 1.5042 0.5042 y P
31.25-30.25 y P

418

RT

Curve

yP

yr

Op. Eq., FP FIN

0.1
0.20

0.00515
0.01114

0.30

0.01830

0.40
0.5

0.02721
0.03882

0.6
0.7

0.05504
0.08000

0.8
0.9
1.0

0.12492
0.2349
1.0

y out

y out

PCO2
t ms

cc STP cm

FP

FP

b.

1000

FIN

gmole

1 hr

3600 s

0.0002859

cc STP

J CO 2

Fin
Fin,1

FP1

2.125

0.32

Fin

0, y P

.15 .32

yP

pr yr

0.402, y out

0.46875

0.25625

0.0276 CO 2 conc.
(17-2b)

pP yP

0.088888

0.46875

2.125 .10

15.2 cm Hg 0.402

0.0002859

cc STP
cm 2 s

gmol
s

1.0 gmole

cm s 1000 cc 22.4 LSTP

FP y P,CO 2

FP

1 0.32

912 cm Hg 0.0276

1 10 cm cm s cm Hg

Area

0.15 CO 2 mole frac

0.10, y P

Answer (from graph)

J CO 2

FP FIN

Plot two arbitrary points:

FP

y IN

Slope

10

15 10

y out

y IN

y out

FP FIN

yP

J CO2
J CO2

1 FP FIN

yP

0.32

1.2764 E 8 gmole s cm 2

2.80 10 6 cm 2

0.32 kgmole/hr , Fout

Fin

FP

1 0.32

0.68 kgmole/hr.

1 kgmole/hr

1
2

FP,part a

Stage 1. FP1 Fin1

1
2

0.32

0.16

0.16 1.00

FP 2 , Fout1

0.16 , yin ,1

Fin2

FP1

1 0.16

0.84

Fin ,2

0.15

RT curve is unchanged!
1 .16
0.84
Op. Line: Slope
5.25
0.16
0.16
Find arbitrary points to plot line:
0.15
If
y out ,1 0, y p
0.9375 (off graph).
0.16

419

If

y out,1

0.04, y p

y out,1

If

5.25 0.04

0.08, y p

5.25 0.08

Answer (from graph):


Stage 2

FP 2

0.16

Fin 2

0.84

0.7275

0.9375

y P,1

0.5175

0.625, y out,1

0.1905,

0.0595

y in2

0.0595

FP2 Fin2

0.1905

yin,2

0.3123

1 0.1905

4.2500 . Plot curve


0.1905
Answer:
y P2 0.250, yout 2 0.015 (see graph)
Stage
15 10 10
cm 3 STP
912
0.0595
15.2
0.625
0.0006715
1 10 4
cm 2s
1L
1
mol
JCO 2 J CO 2
2.9976 E 8
1000 cc 22.4
cm 2s
mol 1 h
FP1
0.16 1000
0.04444 mol s
h 3600s
FP1 0.625
Area 1
92.67m 2
JCO 2 ,1
Slope

J CO 2

0.9375

Stage 2: J CO 2 ,2

15 1010
1 10
JCO 2 ,2

912 0.015

13.2 0.250

1L

J CO 2

0.0001482

6.6161 E 8

1000 cm 22.4
lh
FP 2
0.1905 840 mol h
3600 s
FP 2 y P 2
Area 2
1, 680, 000 cm 2
JCO 2 ,2

1:

cm 3 STP
cm 2s

mol
cm 2s

0.04445 mol s
168 m 2

It is interesting to compare parts a and b.

Part a:

1 stage

Area

280m 2

y out,CO 2

0.0276 or 97.24% CH 4

y P,CO 2
Part b:

2 stage

0.402

Total Area

260.67m 2

y out

0.015 or 98.5% CH 4

y P1

0.625

y P2

0.250

420

17.D2. a.

yP
Slope

1 FP Fin

y in

, yP
FP Fin
FP Fin
.7
2.333 , When y out
.3

y out

y out
0, y P

y in

0.2 CO2 , FP Fin

0.2
0.3

.3

0.6667

421

When

yP

y in

0.2

0.286
1 FP Fin
.7
RT curve is same as in Problem 17.D1.
Draw op line. From graph: y P,CO2 0.53, y r y out,CO2
b.

0, y out

J CO2

J CO 2

15 10

10

pP yP

t ms
cm 3 STP cm 76 cm Hg

1 10 4 cm cm 2s cm Hg
J CO 2

PA p r y r

0.06

0.002148

atm

cm 3 STP
2

cm s

60 atm 0.06
FP

, FP

0.6 mol s 0.53

Fin

Fin

0.6

1000 cm 3 STP
3

1 mol
cm STP
0.002148
22.4 L STP
cm 2s

L STP

3.3 0.53
mol
s

atm

3, 254, 000 cm 2

Or 325.4 m 2 . Very sensitive to y P & y r values.


Can also calculate J
Check:

Fin
Fin x in

FP

Fout , 2
FP y P

J CO 2

0.6 Fout

Fout y out , 0 .4

FP
J

J CH 4 . A

Fout

1.4

2 0.2

0.6 0.53

1.4 0.016

.402 , OK

422

17.D3.
New problem in 3rd edition Since no concentration polarization x w

J solv

K solv
t ms

pr

pp

a xr

R
and M.B.

xp

xp

xp

x out
1

xF

xF

x out

Then,

pp

K solv t ms
1

1 R

xp
Then

pr

x out ,

xp

J solv

pr

Solve for

xr

1 R x out

415.4

a xr

xp ,

1 R x out , R
,

0.22, x F

K solv
t ms

33.29, J solv

415.4

0.9804
0.0077

0.0077 0.22
0.78
1 0.9804
0.22

0.0098175

0.0001924

1.1 15.446 0.0098175 0.0001924


33.29
17.D4. Partially new problem in 3rd edition.

13.72 atm.

423

a.

xw

exp ( J solv /

xr

1 xP

Mxr

xP

pr

3.6 E
3.6 E

K solv

solv ) k

pP

4.625 / 997000 g/m 3

exp

6.94 10

Mxr

xP a

pr

pp

3.6 E

1.069 0.054
75 2

3.6 E

1.069 0.054

J solv

t ms

1.069

xP

4
4

59.895

4.625

a Mxr

Since

73

xP

1.0689 .054

K solv K A , K A

K solv

and

3.6 E

KA
t ms

59.895

2.29

1
atm

0.0665

g
2

m s atm

K solv
t ms

g
KA
m s atm 0.029 g
t ms
2.29 atm 1
m 2s
c. Write Eq. (17-37a) for old and new situations Divide new by old. Obtain
0.0665

0.75

k new

k old

Everything else divides out. Since

new

rpm ,

old

k new
17.D5. a.

K solv
t ms

J solv
pr

RT eq., R

k old

.75

2000

0.000117m / s

1000

1.5 10

cm s

pP

1
102 atm

1.47 10 5 g

1 FP Fin

1 x P x out , Op eq., x P

cm 2 s atm

x out

x in

FP Fin
FP Fin
Solve simultaneously & obtain Eq. (17-26), which with M = 1 is
1 R x in
0.003 0.05
xP
xP
0.000272 , x out
0.091
1 R
1 0.997 0.45
1 R
1 xP
x P pr

xr

pP

xP

xr

xP a

1 0.000272 0.091 0.000272


0.000272 102

0.091 0.000272 59.895

b. Plot the RT curve and operating line


xP xP
a 1
pr pP
xr
M 1
a
1 xP
See graph. Intersection occurs at x r

J solv

K solv
t ms

pr

pP

, xP

0.0585, x P
a M xr

3.44 atm

xr

x in

0.000752

xP

424

425

J solv

1.47 10
2

78 atm

cm s atm
FP 1 x P
, FP
A
J solv

2
3

Fin

1000 g kg

g
cm 2 s

3.36 10 6 cm 2

9.91 10 g cm s

Eq. (17-45) can be written as J solv /

solv

n xg

336 m 2

n xr

n x r . Slope = k and intercept = k n x g


cm

J solv /

solv

0.052
0.037
0.026
0.0134
See Figure. Plot J solv /

solv

Intercept

Stirred cell data:

cP

J solv /

J solv

g L,
solv

wt frac

xr

min

n xg

Slope

J solv

0.000991

3.33 kg s

Plot J solvent vs

17.D7.

5 kg s

3.33 kg s 1 7.52 10

A
17.D6.

59.895 atm 3 0.0585-7.52 10 -4

dextran
0.012
0.03
0.06
0.135
versus ln xr

0.0185
0.01596

K solv
t ms

J solv
p

23.1 / (997 g / L )

n xr
4.423
3.507
2.813
2.00

1.159, which is x g

69.25
3.0

23.1

0.314 .

g
2

m s bar
L
0.0232 2
m s bar

426

J A ~ J solv c P
Mc

With

0.00696 g (m 2 s) . Also, J A

0.0232 0.30
1, 1 R c

Spiral Wound:

JA

Solving for M c

JA

0.00696

c out J solv

10 (0.0232)

J so l v c P

0.03 R c

0.97

M c cout 1 R c J solv

cP

1.0

c out 1 R c

8 0.030

K solv

J solv

M c cout 1 R c J solv

t ms

4.1660
80.8g / (m 2s)

23.1 3.5

Since osmotic pressure is ignored, M c does not effect solvent flux in UF.

yp

He H 2

rd

17.D8. New problem in 3 edition From 17-6b, y rHe

He H 2

Check y rH

H 2 He

b.

Pr

1 y PH

1 y PH

H 2 He

y rHe
Then

Pp

H 2 He

pp
pr

1 y rH

0.975

y PHe

y r ,He

0.025 0.07656

y F,He (1
yP

2.8314 .975

.9234

0.07656 OK.

0.05 0.07656

.07656

3.8314 1 .2 .025

3.8314

y rout He

10

y FHe

y F,He
1

1 yp

1 y P He

0.5152

Use solution in Eqs. (17-9) to (17-10e)


pp
.75
a
1
.2 .261 1
1
pr
.25

Pr

0.2610 , p p p r 0.2
90.8 10 10
.739 0.2 .975 1
0.021397
0.025
.261
.739 0.25
.279475

y rHe

y PH

23.7 10

PHe PH 2

Pp

.739 .2 3

.261 .05

b2

4ac

2a

Must use minus sign to have positive y p . y PHe

.25

2.3648
.05

.25

.25

1.4874

0.0522

1.4874

2.212359 .49377
2

.15763
4.7296

2.3648

1.4874 1.645
4.7296

0.00333

427

y r,He

y F,He

y PHe

c. Solve RT eq. (17-6b) for y p : y r

yr

pp

1 yp

y 2p

pr

pp

yr

yr

pr

y rH2

.9234,

1 yr

pr

y rHe

y PHe

y rHe

FP y PH

FP

0.025,
2

.06

.739

.80786

.65264

y p,H 2

0.004842
0.1813

0.05,

y H2 IN

.95

.5152

0.975

FIN

pr yr pp yp
tm s
51.52 m3 STP h
1h

m3

3600 s

90.8 10

10

cm 3

cm 3 STP

14311.11

14311.11

a.

yp yr

PH 2 A

cm 3 STP

s
STP cm

cm 2s cm Hg

A
17.D9.

1 yp

pr

y He,IN

1000000 cm 3

Pp y P,H 2

.2956
0.05 0.06
0.004842 0.06

51.52m 3 STP

PH 2 Pr y r,H 2

.739 .2 1

.80786

y rHe

0.07656 y p,H 2

FP y PH tm s

pp

0.01566

4ac

2a
y FHe

Use 17-5a written as

FP

100m3 STP / h.

FIN

For Part A

pp

b2

Use + sign for positive y p ,


d.

y p2

0.1478

.06 .261
b

yp

.739 .2

pr

pr

0.10001

pp

.25
pp

.05

3 .00333

555,186 cm 2

.975 1.0 10 4 cm

380 .9234

76 .975

cm Hg

55.52 m 2

Plot the data on a semilog plot in the form of J solv /

solv

J solv L / (m 2 h)

428

J solv

xr

k n

xg

From graph, slope


When x r

k n xr.

Intercept

18.3 and k

18.3

k n xg

0, J solv /
n xg

k
solv

=J solv

82.9 L m h

23.0 g m 2s

4.53

5.08 m / s

xg

k n xg
L

5.08

m h

23.0g / (m s)

k n xg

92.8%

The value of xg is very sensitive.


b.

There is only one point further out on the n axis. Any error in point is greatly amplified in the
least squares regression. Hence, another point in this region would be most useful. The higher
the concentration, the better.

17.D10. a) New problem in 3rd edition

2700
800

3.375 , a

yin 1

yp

3.8884

Solve RT eq. and op. eq. simultaneously


pp
0.3 .5
1
2.375 1.6116
pr
0.7 2

pp
pr

yN
1

3.375 .25
3.8884

.7

2.375

.3

.25

.7

.7

.7

3.8884

1.20536

4 1.6116 1.20536

2 1.6116

.5

, use minus sign to have yp between

0 and 1..

429

yp

3.8884

2 1.6116

From op. eq.: y r


b)

Since Fp y p,A

15.1196 4 1.6116 1.205


y IN

yp

1
PA A A

t ms

PA A p r y r

pp yp

.365295

.7

pr yr

F p y p,A t ms

.3

0.365295
.25

0.2006

.7

pp yp

. Since F
IN

1 mol s, Fp

FIN

0.3 mol s

mol
0.365295 1.2 10 4 cm
s
3
cm STP cm
1.0 L STP
1
2
L STP 10 3 cm 3 STP
cm s cm Hg
22.4
mol

0.3
A
2700 10

10

76 cm Hg
atm
A

c)

F p y p,A t ms
PA A p r y r

pp y2

, F p

F IN

mol
s

0.325 1.2 10 4 cm

9.0823 104 cm 2

17.D11. New problem in 3rd edition.

xp

Gelling occurs at a solvent flux of J solv

J solv

Then x gel

solv

atm

0.4 mol s

1
3
cm
STP
cm
22.4L STP 1.0 L STP 76 cm Hg
2700 1010
2
cm s cm Hg
mol
10 3 cm 3STP
atm

J solv

.5 .365295

6.569 104 cm 2

0.4
A

2.0 .2006

5200

L
2

m day

x out exp (J solv /

997

0 since R 0

x IN
1

0.001
.6

0.5 0.325

atm

0.0016667

5200 L/(m 2 day ) which is

day

L 86400 s

solv ) k

1.0, x out

2.0 0.175

60.0

g
m2s

0.0016667 exp

17.D12. New problem in 3rd edition p p / p r

1.0 4.5 .

60.00g / (m 2 s) 997000g / m 3
2.89 10 5 m / s

He H 2

PHe
PH 2

0.01334

0.261. Use Eq. (17-6b),

430

y p ,He

He H 2

1 yp

pr

y r ,He

1 yp

Eq. (17-7c)

.261

y FHe

y rHE

.2 .254

y p He

y rHe

.1 .254

a)

If x p

x in , cut

.55 , perfectly mixed

(17-27)
1 R
0.55, what value R required. Find R (including concentration

0.00050 &

polarization effect). From Eq (17-27), x p

xp

0.25446

1 R xn

xp

R x IN

0.353

17.D13. New problem in 3rd edition .035 NaCl


Rejection

1
1 .1
4.5
.261 1 .1

0.1 .261 1

x IN

xp R

x IN

xp

x IN

x IN

x IN

0.035 0.0005

xp

xp

Rx IN which gives

0.035

.55 .0005

0.035 0.001

0.9935

b)

If

c)

If R (inherent rejection coefficient with M = 1) for part b is R


0.992, what was value of
M that gave R 0.9869
MA 1
M CaseB
.
R Case B 1
1 R CaseA . Let A be highly stirred
RA R
M CaseA

xp

0.0016,

M CaseB

17.D14.
wB

0.55 , R

xpR

RT curve: y w

Feed

xp

0.035

M CaseA 1 R CaseB

1.0 1 0.9869

1 R CaseA

1 0.992

wB

xw

wB

43 (mole frac). Since x w ,IN

x IN

141.6

cal

0.9869

1.6375

43 x w

1 xw

1 42 x w

0.10, only need RT curve below 0.10. Create table and plot

xw

yw

0.10
0.08
0.05
0.03
0.01
0.0025
0.001

0.8269
0.78299
0.6935
0.571
0.3028
0.0973
0.0413

kcal

g 1000 cal

0.9 10.5

.55 .001

74.12 g
mol

0.1 9.72

10.5 kcal mol

10.42 kcal mol

431

C PB

0.625

1
1000

0.046 , C PW

74.12

CP,in
MW feed

xW , F MWW

a) Assume y P
and

C PL,in

0.9 0.046

xB , F MWB

1
1000

0.1 0.018

Tout where Tin

0.5 10.5
Tout

30

then,

x in

0.018

0.0435 kcal mol C

0.5 9.72

molar ratio. Slope Op line

18.016

0.1(18.016) 0.9(74.12)

0.5 to calculate p

Tin

1.0

68.51

10.11
0.0435
10.11

30

0.129. This is a

6.75 , and op line goes through point (mole fractions)

0.10

0.775. Plot operating line. From graph, y P 0.57, x out


0.129
(mole fraction water). This value of yp is reasonably close to our assumption.
x out

0, y P

Fp / FF

( Fp MW p ) / ( FF MWF )

0.031

( MW p / MWF )

432

MWp

y p ,W MWW

y p , B MWB = 0.57(18.016) + 0.43 (74.12) = 42.13

.129(42.13 / 68.51)
Area

b)

Cut

Permeate Rate

0.08

x in,w

Then

Tin

c)

Tout

Flux

( 0.0791 100 lb h)

0.2 lb h ft 2

39.53 ft 2

0.10

1 cut

Slope

0.0791 in (lb/h)/(lb/h).

0.92

11.5. Find y P 0.68 from graph.


cut
0.08
0.32 10.5 0.68 9.72 9.97

C PL,in

x out

0.05, x P
FP Fin

30

0.08
0.0435kcal / (mol o C)

9.97 kcal mol

48.3 C

0.6935 (From RT table or graph).

x in

x out

0.10 0.05

yP

x out

0.6935 0.05

0.3065 10.5

0.6935 9.72

0.0777

9.959

433

Tin

Tout

C PL,in

17.D15. Parts b to h are new in 3rd edition

xP

0, x r,out

Fout

Fin

Fin

FP

Fin
Fout
1

0.0777
0.0435

RT curve:

xP

Fin x r ,in

FP x P

Mass balance perfectly mixed


Since

30 C

47.8 C

1 R M xr

0.

Fout x r ,out

x r,in
FP

0.8 . Then

Fin

Fin
Fin
Fout
1
x r ,out
0.10 0.125 , Fout 0.8Fin
0.8
Alternate graphical solution gives same result.
6.5

xP

9.959

1
0.8

80 kg h

Op. line

xP

-4

xr

x r,in 0.10
0.125

xr

x r,out

xP

RT curve

FP Fin
FP Fin

x r ,out

Slope

x r ,in
FP Fin

0.8

4
0.2
When x r,out 0, x P

x r,in

0.10

FP Fin

0.2

b. Area = Fp / Jsolv = (20kg/h)(1 L)/0.997 kg)(24h/day)/ (2500 L/m2 day) = 0.193 m2.
c. Gel formation occurs when x w = 0.5 and xw = M xout = 0.125 M. M = 0.5/0.125 = 4.0
d. Gel formation occurs when xw = 0.5 and xw = M xout = M xF / (1 ) = 1.2 (0.1)/(1 )
Then 1 = 0.12/0.5 = 0.24 and = 0.76.
e. Gel formation occurs when x w = 0.5 and xw = M xout = 1.2 xF / (1 ) = 1.2 xF / (1 0.2).
Obtain xF = 0.333.
xr,out = xF / (1 ) = 0.3333/0.8 = 0.416
f. We have xr,out = xF /(1 ) = 0.20/0.75 = 0.26667. M = xgel / xr,out = 0.5/0.26667 = 1.875.
First occurs when Jsolv = 2500 = k ln (M). Obtain k = 3977 L/(m2 day) = 4.60310-5 m/s.
g. M = 1.875 and k = 3977. Since we change the pressures, J changes which will change M. However
with constant stirring k is constant. First, assume no gel and calculate J and M.
pr pp
K
Jsolv
2500 L (m 2 kg)
L
J solv K solv
, solv
2083.33 2
t ms
t ms
p r pp
2.2 1.0 bar
m day bar
Then, without gel,

J solv

2083.33 3.4 1.0

5000 L m 2 d ay

From Eq. (17-34) M = exp (Jsolv / k) = 3.516.

434

0.5

0.2. Then, x w

3.516 0.2

Mx F
1

0.878, gel forms.

.8

0.2

xF

With a gel, previous work is incorrect. Set R = 1.0, x p = 0, x r

0.25,
1
.8
And from Eq. (17-45), Jsolv = k ln (xgel/xr) = 3977 ln (0.5/0.25) = 2756.6 L/(m2 day)
Note: The same answer is obtained in parts g and h if convert to Jsolv and use k in m/s.
h.

J solv

k n

x gel

.5

k n

xr

Case C, R C

xp

J solv

MB

x IN

1 R C x out

K solv

pr

tms

1.387 152 1.1

m d ay

pr

pp

1 RB

pr

pp

a M C xout

Equation:

log P

log P

0.27027

2446.6 L (m 2 d ay)

0.27027

1.0, p p,B

1.0 .061 14.1


.024 10.96

0.0007505]

1.1, p r,B

12.06

3.27

18.71 g m 2s

42 , P 10000,

1.6232
0.27875

1.9

4a b
4a b

1.9020 2b
b 0.95100 , a
0.16805 logP 0.951

log

.74

xp

a log 10, 000

8, log

0.0001,

a log 0.0001

log 1.9

0.5

0.2

0.01230

15.446[ 3.27 .01230

log 42

0.2, x r

15.2

1 RC

pp

0.26, x F

0.976, M B

1.1, p r,C

17.D17. New Problem in 3rd edition.


a)
log
a log P b , P

b) If

L
2

0.0093 0.26
.74
1 .939
.26
0.0007505

1 RC

3977,

Case B, R 0B

.939, p p,C

For Experiment C. M C

x out

3977

xr

17.D16. a) New problem in 3rd edition

b)

Gel forms since it did previously,

.16805

0.90309

log
0.16805

0.28509

.951
0.16805

PO2

1.93 Barrers

435

17.D.18. New problem in 3rd edition. Ideal Gas: Vol% = Mole %

PN2

250, PCO2
FP

2700,

PHe

300, PHe
3

0.4 m3 s. FIN

FIN

550

1.0 m s

10 2 cm

10
3

10 6 cm 3 s , FP

m
76 cm Hg

Part A

pr

2.5 atm

Part B

pr

76 cm Hg

t ms

0.8 mil

Eq. (17-11d)

10

190 cm Hg

atm

pp

0.4 76

.00254 cm

FP A

30.4 cm Hg

Need to guess value of FP A or of y P ,

Pi t ms p p

Since CO 2 has highest permeability, CO 2 will be concentrated; thus, guess y p,CO2

y r ,CO 2

Then

y p,CO 2 ,guess

FP A guess

Then,

PCO 2

y IN ,CO 2
1

t ms y p,CO 2

76 cm Hg

0.002032 cm

mil

Pi t ms p r

K m,i

pp

.4 106 cm3 s

yCO2,IN

0.4 and y IN,CO2

where
p r y r.CO 2

0.40
0.4.

p p y p,CO 2

Use FP A in Eq. (17-11d) to calculate all K mi


Then check

y IN,i

y r,i

K mi 1

1.0 ?

Put in Spread Sheet. Can use Goal Seek to force


Results: a. y P,N2

y r,N2
b.

.15037, y P,CO2

.3164, y r,CO2

.54446, yP,He

.3037, y r,He

y r,i

1.0 as change y P,CO2 .

.3351, yP,H 2

.06099, y r,H2

.3189

.27154,

.999885

1.0000766

Same answers for mole fractions since p r p p is same.

yF N2
F,cm3/s
pr, cm Hg
P N2
P He
Fp
yp CO2
Fp/A
K N2
K CO2

306 2008

HW 8

Problem 2

0.25
1000000
190
0.000000025
0.00000003
400000
change yp to
0.544455884
0.003983763

yF CO2
tm, cm
pp, cm Hg
P CO2
P H2

yF
0.4 He
0.002032 theta
76
0.00000027
0.000000055

get sum=1
yr CO2
A, cm 2
A, m 2
0.475237299 y r N2
1.792765611 yr CO2

0.05 yF H2
0.4

0.3

0.303696077
100407575.7
10040.75757
0.316417678
0.303696077

436

K He
K H2

0.549397235 yr He
0.851323363 yr H2
Goal seek
Sum
0.150373483
0.544455884
0.033509684
0.271546029
0.99988508

yp N2
yp CO2
yp He
yp H2
sum

0.060993544
0.318969314
1.000076613

17.E1. New problem in 3rd edition For dilute systems J solvent


Transfer Eq. (17-7c)

xp
x out

J total solution , FIN

Fp

xF

x out

0.022 0.056

FIN

xp

x out

0.00032 0.056

0.00032

0.9943 , J s u c rose

0.056

J solventx p

Fout , Basis: FIN

Fp

1.0

0.6106

J solvent

solution

Permeate

0.997 0.4 x p 0.997 0.4 0.00032 0.99713 kg L


Initial assumption is OK.
J solv J solv / solv (3.923 g / m 2 s) / (997 g / L) 0.003935 L / ( m 2 s)
permate

(3.923g / m 2 s)(0.00032)

J sucrose
b)

Eq. (17-27)

0.00126g / (m 2s)

1 xp

K water

water solute

K sucrose

x p pr

Mxr

pp

xp

Mxr

xp a

1 .00032 0.056 0.00032


w s

0.00032 60.0 1.1

From Eq. (17-16c),

K water
pr

Eq. (17-18)
c.

pp

a Mxr

59.895 1.0 .056

K sucrose

M xr

1 M 1 R

Then RT equation is

xp

1 R x out

Operating Equation is (17-23)

xp

0.0706 g

0.00126
0.056 0.00032

xp

Solution 1.

xp

0.00032

J sucrose

tms

g sucrose

xp

3.923
60 1.1

g water

J solv

tms

K water
t ms

0.056 0.00032 59.895

3.131

0.6393 x out

2.1 1 0.9943

m 2 s atm

0.0226

g sucrose
m 2s wt frac

0.98803

0.01197 x out
x out

xF

.39

0.036066

Solve RT & operating equation simultaneously. x out

.61

x out

0.05537 , x p

0.022
0.61

0.000663

437

Check

pr

t ms

60 1.1

pp

a M xr

K sucrose

Mx r

t ms

3.67g/(m 2s)

0.000663]

J water x p / (1 x p )

0.00243 g (m2 s)

3.67 0.000663 / (1 .000663)

Alternatively, J sucrose

xp

59.895[ 2.1 0.05537

J sucrose

J sucrose

0K

0.98803

x out

K water

J water
= 0.0706

xp

xp

0.0226[ 2.1 (0.05537) 0.000663]

0.00261

g
m 2s

6.9% different

xp xp

Solution 2. RT Eq. (17-21), x r

2.1 1
xr

Simplifies to,
Linearize x r @ x p
Slope =

3.140 59.895 0.997

0.0003

xr

0.02509

3.140 59.895 .997

1 xp

0.02509 . Note xp = 0.0003 is an arbitrary point.

0.01196

0.01196 x r or x p
Solve simultaneouly with Operating Equation
xF
1
xp
x out
0.6393 x out 0.036066

0.05538 , x p

2.1 391.66 x p

Then linear form of RT equation is x p

x out

3.140 .997 60 1.1

x p 186.5 x p 185.391

0.0003

xp

0.01196 x out

0.0006623 . Very close to value obtained with retention analysis.

C PL,in

FP

17.E2. (was 16.D11 in 2nd ed.) Eq. (17-59b):

Fin

Tin

Tout , Tin

Tout

85 25

60

Stage 1. Assume y P ~ .05 water, 0.95 ethanol


P

0.95

For Feed CPL,in

0.05

0.1 CPL,w 55 C
0.1 4.1915

2290.3 kJ kg (See Example 17-9)

0.9 CPL,E
0.9 2.7595

kJ
kg K

2.903

kJ
kg K

Where average temperature from 25 to 85 is 55C and C P values are from Perrys 7th, pp. 2306 and pp. 2-237.

FP1

2.903

Fin

2290.3

60

0.0760 and FP1

0.0760 100

7.60 kg hr .

438

Op. line intersects y P

x out

x IN
1

Slope
Op. Eq. is, y P

0.10 (water wt. frac.)


1 0.760

0.1

12.16 x out

12.16

0.0760

12.16 x out 1.32


0.0760
If
y P 1, x out 0.32 12.16 0.0263
Plot Op. Line on Figure 16-17a and find intersection:
y P1 0.66, x out1 x 2,in 0.055 (water values)

Fout1

Fin 2

100 7.60

92.40 kg h

Stage 1 Trial 2. Since yp yp, assumed, do a second trial.

yP

0.66 water, 0.34 E,


FP1

2.903 60

F1in

1892

FP1

0.0921 100

0.34

9.21 kg h , Fout1
9.86 , y P

0.0921
0.086

If y P

0.66

0.66 2359

0.34 985

1892 kJ kg

0.0921

1 0.0921

Slope

100 9.21 91.79 kg h


0.1

9.86 x out

0.0921

9.86 x out

1.086

1, x out

0.00872
9.86
Plot operating line and determine (from graph)

y P1 0.64, x out1
x in 2
value of yp is close to the assumed value of 0.66. Can proceed to stage 2.
Stage 2: Estimate y P
For x in 2
2

0.50 (water),

0.050, CPL,in

FP2

2.967

Fin 2

1672

Fout 2

60

0.05 4.1915

0.50

.1065 90.66

0.050

1 0.1065
0.1065

1672

2.967
9.64 kg h

8.40

8.40 x out 0.4421


0.1065
0.4421 . Draw op. line. Intersection gives y P 0.34

For x out 2 use MB. x out 2 ,

8.40 x out

0.95 2.903

8.10 kg hr , Slope

90.66 9.64

0, y P

0.50

0.1065, Fin 2

yP

If x out

0.05 water. This

Fin 2 x in 2
Fout 2

FP2 y P2

90.66 0.050

9.64 0.34

81.00

x out 2 ,w 0.0155 or x out,2 ,ETOH 0.9845.


This is a close as we can get graphically.
9.21 0.64 9.64 0.34
Mixed Permeate: y p,mix
0.487 wt frac water
9.21 9.64

439

Area

FP

kg 1000 g
h
kg

J g h m2

, J from Fig. 16-17b based on x out

Stage 1

0.8333 g h m 2 , A1

Stage 2

0.208 g h m 2 , A 2

9.34 1000 g h
0.8333
9.64 1000

11, 208 m2

46, 346 m2

0.208
Other flow patterns will reduce area. Area is large because of low flux caused by low ethanol
permeation rate.

17.F1.

RT eqn., y

x x
1

1 x

x Benz

0.2,

18.3, y

x Benz

0.1,

6.66, y

Operating equation Slope

, x Benz

0.3,

18.3 .2
1 17.3 .2

6.66 .1
1 5.66 .1
.9
.1

16.6, y

16.6 .3
1 15.6 .3

0.87676

0.8266

0.4253 . Plot RT equation.

9 . Plot on graph. Find y PBenz ~ 0.844, x out,Benz ~ 0.238

440

CPL,in

y Pbenz

benz

1 y Pbenz

x Pbenxw CPLbenz

iP

0.844 94.27

1 x benz,in CPL ,iP

1 0.844 164

0.3 0.423

.7 0.73

105.15 cal g

0.6379 cal g C

441

Tin

Tout

C PL,in

50 C

0.1 105.15

66.48 C

0.6379

17.H1. (was 16.G1 in 2nd edition)


This is set up for Area being the unknown and cut being known. Problem 17.H1
Fr,in
10000.000000 yin,A
0.2500 cut=Fp/Fin
0.2500
tmem,cm
0.002540 pr,cm Hg
300.0000 pp,cm Hg
30.0000
yin,B
0.5500 P,A
0.0000000200
Fptot
2500.000000 yin,C
0.2000 P,B
0.0000000050
Fr,out
7500.000000
P,C
0.0000000025
Guess values of A or equivalently Fp/A until sum y,r and sum u,p are = 1.00
Fp/A
0.0007059 (this is final result)
KA
2.507328 KB
0.7720 KC
0.4015
sum x eq
y,r,A
0.181576 y,p,A
0.455271198
y,r,B
0.583244 y,p,B
0.450268647
y,r,C
0.235190 y,p,C
0.094429331 Area, cm2
3541578.1
sum y,r
1.000010 sum y,p 9.999692E-01
These results agree very well with Geankoplis results.
17.H.2. New problem in 3rd edition Part a)
y p 0.5243, y r,out 0.0610, A
b)

y p,avg

0.6193, y r,out

3, 200,152 cm 2

0.0203, A

2, 636,196 cm 2

17.H3. New problem in 3rd edition


Counter current. Shows final guess for theta.
Fin,
cm3/s
100000 yin
0.209 thetatot
PA/tms
0.003905 pr, cm Hg
114 pp, cm Hg
M
15 N
100 yroutguess
df
0.9
j=N-i+1
Fr
yp
yr
Area
Fp
Fp/Fr,j-1
yp
Areatot
yincalc
Fincalc
Massbal
yrout

100
28600
0.173174301
0.144051968
9710.750234
714
0.024356963
0.235015561
824015.8215
0.208999973
100000
9.09495E-13
0.144051968

99
98
29314
30028
0.1738615 0.174547
0.1447613 0.14547
9664.4075 9618.879
1428
2142
0.0475556 0.069677

97
30742
0.1752323
0.1461768
9574.1438
2856
0.0907935

0.714 PB/tms
76
0.2 erroracc

0.00175
0.0000001

96
95
94
31456
32170
32884
0.175916 0.1765981 0.17727918
0.146883 0.147588 0.14829195
9530.183 9486.978 9444.50995
3570
4284
4998
0.110973 0.1302761 0.14875885

442

17.H4. New problem in 3rd edition The spread sheet equations are shown below for part b. Part a agreed
with problem 17.D14. Part b answers: yp,W = 0.412, = 0.2122, xout,W = 0.0160, = 0.158,
Area = 79.0 ft2. Note that if the starting guess for yp,W is too high, Goal Seek will converge
on an answer with yp,W > 1, which is obviously not physically possible.

17.H5.

This problem is very similar to Example 17.7. It is easiest to solve on a spreadsheet, which is

shown below. The results are shown in the spreadsheet.


New problem in 3rd edition

443

444

17.H6. New problem in 3rd edition The spreadsheet is similar to that for problem 17.H5 and is shown
below,

17.H.7. The same spread sheet that was used in problems 17.H5 is used.

445

446

SPE 3rd Edition Solution Manual Chapter 18.


New Problems and new solutions are listed as new immediately after the solution number. These new
problems are: 18.A3, 18.A16, 18.B4, 18.C4, 18.C14, 18.D3, 18.D8, 18D9, 18D14, 18D15,
18D18, 18D21, 18D24, 18D25, 18.D29, 18.D30, 18.F1, 18.H1-18.H2. Chapter 18 was chapter
17 in the 2nd edition. Most problems from that edition have the same problem number, but the
chapter number is now 18 (e.g., problem 17.D6 is now 18.D6).
18.A1.
1c; 2 b; 3a
18.A.2.
1c; 2a; 3b
18.A.3. New problem in 3rd edition. One barrier is lack of knowledge. Most chemical engineers are not
trained in use of adsorption, chromatography, and ion exchange. Thus, they do not think of these
processes as a potential solution. A second barrier is the simulation tools are not as developed and widely
available as the simulation tools for equilibrium staged separations such as distillation.
18.A4.
Regeneration is too difficult.
18.A5.
In the SMB the mass transfer zone between the two solutes stays inside the cascade. In a
chromatograph the MTZ exits the column and must either be completely separated,
which requires a significant amount of desorbent, or recycled appropriately.
18.A7.
d
18.A.8. New problem in 3rd edition. The LUB approach assumes constant pattern behavior. Linear
systems do not have constant pattern behavior.
18.A9.
18.A10.

d
e

18.B.4. New problem in 3rd edition. There are obviously many possibilities. One is to develop sorption
processes that use an energy separation processes (e.g., pressure or temperature) to produce purge or
desorbent from the feed so that a separate purge or desorbent does not have to be added.
18.C1.

Vavailable
P

18.C4.

rd

1 f cry

P1

clay

P4

P1

f cry

P2

1 f cry
f cry

1
cry

e
P2

f cry

P2
P1

K di Vcol.
1 f cry

f cry

P2

(same as 18-3b)

New problem in 3 edition.


Amount in mobile phase = e (Vol. Col. Segment)
Amount in pores = 0
(no pores)
Amount exchanged
z A c cRT y K DE
No 1
Obtain, u ion

z a c x cT

term because c RT is equivalent/L

e z A c x c T v int er

e z A c c T x
z A c c RT y K DE
v int er
Simplify to, u ion
(18-44)
c RT y
1
K DE
x
ecT

445

18.C7.

CA

C AF

z uAt

1 erf

4E eff t u A
v inter

12

Sketch of break through:

erf (a)

.9 a

1.164

95%

t final

5%

tw

t st

t1

erf (a)

At 5% point,

0.90 a 1.164

1.164

L
4E

t st u A
v

12

Or
let

u A t st

2.328 E t st
x1

12
st

t , uAx

2
1

uA

12

x2

By definition,
Use

u A t final
uA
v

12

12

L u A t st

v
2.328 E

12

u 1A2
v1 2

x1

2.328

L
2

0.

E u At
v

4Lu A

2u A
2.328

Let

4E t final

E 1 2 u 1A2
2.328
v1 2

x1

1.164

and at 95%,

12
fin

t , then x 2
t MTZ

t final

t st

x 22

E 1 2 u 1A2
v1 2

2.328 2 E u A
v
2u A

4L u A

x12

sign for both (has to be to have positive times).

446

4u

2
A

4u

2
A

2
2

2
1

2.328 E 1 2 u 1A2
v

2.328E 1 2 u 1A2
v

12

t MTZ

2
2

2
1

E uA

4L u A

2.328

12

4L u A

v
2

E uA

E uA

E uA

2.328

2.328

2.328E 1 2 u 1A2
v1 2

12

2.328E 1 2 u 1A2

12

2.328

2.328 E 1 2 u 1A2

2.328

E uA

4L u A

4L u A

4L u A

4u 2A
2

If

4L

2.328 E
v
t MTZ

2 2.328 E

12

v1 2 u A

18C9. New problem in 3rd edition.


For Figure 18-7B, In

In Out = Accumulation
t vinter A c CT,after x i,after

Out

Accumulation

very reasonable since E is usually small ,

t vinter A cCT,before x i,before

LA c yi,after

yi,before CRT

LAc x i,after CT,after

Note that C RT is constant. After dividing both sides by

v int er x i ,afterC T ,after x i ,beforeC


L

C RT y i ,after

y i ,before

x i,before CT,before

t A c , mass balance is

T ,before

x i ,afterC T ,after x i ,beforeC T ,before


t
t
For Figure 18-7B with a total ion wave,
L
u total ion v int er
t
The first and third terms in the mass balance cancel each other. Thus,
L
C RT y i,after y i,before
0
t
Which requires,
yi,after yi,before
18.C10.

447

1
A

v1

v2

v2

v3

u A ,i

C A ,i L i

u A1

M 1A u port

vF

u BL

C B,i Vi

u B2

M 2 B u port

v B,prod

u A3

M 3A u port

vD

u B4

M 4 B u port

v3
v4

v4

B
4

v A ,prod

v1

CB v2

M 2B u port

CA v2

C A v3

M 3A u port

CA v 2

vF

M 3A u port
CA M 2B

If all

18.D1.

Rearrange:

vD

(2)

M 3A u port

CB

M 2B
CB

v4

M 3A
CA

v1 where v 4

u B4 CB

v1
Thus

(1)

M 2B u port

vF

u port

vD

CB

CA vF

Subtract eq. (2) from (1),

Then

CA

u A1 CA

M 4B u port CB
M1A u port CA

M 4B u port

M 1A u port

M 4B

M 1A

CB

CA

CB

CA

M 1A
CA

M 4B
CB

M 3A
CA

vD

vF

Mi

1.0,

M 4B
CB

D
F

pA

qA

q MAX

1
CB

pA

1
CA

1
q MAX K A

1
CB

1
CA

M 2B
CB

vf

M 3A
CA

1.0

. Plot p A q A vs. p A

448

296 K
p/q
135.863
278.679
478.666
696.073
939.619
1116.143
1189.735

p
275.788
1137.645
2413.145
3757.6116
5239.9722
6274.1772
6687.8589

At 296 K
Intercept
Slope

KA

0.163636

1
q max

, q max

1 q max

0.163636

1 q max K A

80

80

480 K
p/q
1786.943
1709.129
1974.657
2309.538
2778.150
3011.134
3122.979

At 480 K

Intercept

q max K A

6.1125

0.00204545

p
637.7598
1296.2036
2378.6716
3709.3486
5329.6030
6246.5981
6687.8589

Slope

KA

1380

0.260606

q max K A
1
, q max
q max

1 q max

0.260606

1 q max K A

1380

3.8372

0.00018884

449

18.D2.

L soln

a = 22 liter soln/kg ads = 22

1 kg

kg ads 1000 g

b = 375 liter soln/g mole anthracene = 375

q max K A,c C A

18.D.3. New problem in 3rd edition.

uj

Part a

0.022

K A,C

2.104

Kd

uj

2.104

c.

u AN

u DN

HETP

L
g anth

v super

40

5.671 cm min

L u S,DN

4.576

5.567

u S,AN

L N

g ads

K ij

1782 0.00301

4Ru

N1 2

From (18-83)

g ads.

40
1 0.69

2.104

g anth

Time AN = L/uAN = 25/5.671 = 4.408 min


40
u DN
5.474, time
1 0.69 1782 0.00316

b.

104.33

u S,AN

N 10885

0.002297 cm
2

From (18-81),

width at half height


5.54 peak max N
To find width in time units, peak max is in time units = retention
L u S,AN 4.40864 min , width 0.09946 min

d.
time

0.425 width 1/2 height

18.D4.

1 0.69 1782K j

1 mol

0.10456

, u AN

g ads

, vinter

mol 178.22 g

v inter

40

, thus, K A,C

1 K Ac C A

q max

0.022

v sup er

10.0

v int er

0.43

23.26

0.042271 min

cm
min

v int er

a) u s

p
e

us 0
1

Kd

e
e

(18-15c)
p

Kx T

23.26
0.6027 cm min
0.57 .48 1.0
0.57 0.52 2100
17.46
0.43
0.43
684
t br 200 cm 0.6027 cm min 331.8 min

450

b) Assume wall heat capacity is small:


v int er
u th
1 e 1
1 e
1
p
e

u th
1.636
t th,br
c) K x

t br
M.B.in

10.0

23.26
.57 .52 2100 2000

C ps

C pf

5.911

cm
min

23.26

1.23 g g @80 C , u s 80

200 cm 5.4868
cm
min

A c cm 2

331.8 min 0.684 g cm 3

10.0 A c 0.684

331.8 33.84 .0011

Alternate: Eq. (18-24)

C 80
C 0

0.0011 g tol
g fluid

33.84 0.0011

2.611 Cconc C conc


us 0
1
u s 80

0.0011 113.92

5.4868 cm min

1.636 2.603
36.45 min , see figure.

C 80

.43
684 1841
200 cm 5.911 cm min 33.84 min

out
Simplifying:

1
u th
1
u th

36.45 33.84 C conc

297.96 .0011

1
0.6027
1
5.4868

2.611
1
5.911
1
5.911

0.1255 wt frac.

113.92

0.1253 wt frac . A very considerable amount of concentration occurs.

451

80, C = 0
z
usol (80C)

uth

0.0011

C=0
0.0011
331.8
min

0.0011

t
Cconc

33.84
36.45 min
0.1255
Cout
0.0011
0
33.84
18.D5.

36.45

vsuper 20 cm min
vint er vsuper e 20 0.4 50 cm min
For step input w. unfavorable isotherm, get a diffuse wave.
v int er
Langmuir formula: u s
1 e 1 p
1 e
a
1
Kd p
s
1 bc
e
e
But now

us
1

b
.6
1.01 .54
.4

0.46

0
50
.6 .46 1.124 kg
1.2
.4
liter 1 0.46 c

c,g/l us, cm/min


18.2437
0
16.676
0.25
14.794
0.50
12.565
0.75
9.997
1.00
7.1813
1.25
1.50
4.3499

1.81

time, min

50
0.93067
1 0.46 c

tout = L/us, min


2.741 min
2.998
3.3797
3.979
5.002
6.9625
11.4944

452

18.D6. a)

u th
1

u th

.57
1
.5
.43

C ps

.684 2240

.57
.5 920 1.80
.43

50 cm 12.61 cm min
v int er

c)

C pf

If wall effects are negligible,


0.684 2240 30

b) t thermal,br

u s 300K

C p f v int er

Kd

e
e

K xy

W
C pw
eAc

12.61 cm min

3.965 min
30
.57
.57
1
.5 1.0
.5 12.109
.43
.43

u s 350K

6.5298 K xy

t br 300K

50 3.0964 min . Exits at c F

3.0964 cm min

4.423 in same eqn.

0.010 .

453

At t = 20, start hot, t br,hot


Feed is concentrated.
C 350
C 300

50 12.61 20

1
u s 300

1
u th

C 350

0.010 3.2989

1
u s 350

20 L u s 350 K

1
u th

23.965 min

1
3.0964

1
12.61

1
6.5298

1
12.61

3.2989

0.032989 g L . This continues until breakthrough at

20 50 6.5298

27.6572 minutes

0.032989

18.D6.

g/L

0.010

0
t

18.D7.

vint er

vsuper

16.1478

15 0.434

Kd

tr

L us

(A)
p
s

K 4

34.56
0.566 0.43

0.566 0.57 1.0

0.434
60 cm 0.3715 cm min

161.49 minutes. Then exits at C F

27.6572

34.56 cm min
v int er

a) At 4C: u s

us 4 C

23.965

0.3715 cm min

1820 0.08943
0.434
161.49 min . Concentration out is zero from t = 0 to t =

0.01 .

454

v int er

b) u th

1.

C ps

C pf

WC pw
e

A c C pf

34.56
17.293 cm min ,
0.566 0.43 0.25 1820
0.434
1.00 1000
60.0 17.293 3.4696 min +1200

u th
1.743

t br,th

L u th

Eq. A but with K(60)

u s 60 C

34.56
0.720258 cm min
0.566 0.43
1.743
1820 0.045305
0.434
t br,conc 60 L u s 60
60.0 0.720258 83.3035 min +1200
C=0
60
60

60

0 60
C=0

uth
Elution time: 0

c high 60

chigh

cF

CF

1
us 4

3.4696

1
u th

0.01 2.6918 0.05783

83.3

C high

1
u s 60

1
u th

1.38839 0.05783

18.D.8. New problem in 3rd edition. Example 18-3: vinter,F

0.3799 cm min , u s v inter,purge,0 C

u th vinter,purge,

6.466 cm min , u s vinter,purge,80 C

If t purge

t purge

0.019796 kmol m3

18.60 and yinter,purge

u s vinter,F, 0 C

u s v inter,F, 80 C

C=0

25.58
18.60

25.58 cm min .

0.3799

0.5225 cm min

4.343 cm min

18.60

4.343 3.158 cm min


25.58
hot purge time and t F is cold feed time, with

t hot wave breakthrough

vinter,purge

18.56 min (from Example 18-3) then breakthrough

equation is u s v Inter,F, 0 C t F u s v Inter,purge,0 C t

thermal,breakthro
ugh

120 cm

455

120

tF

0.5225 18.56

290.35 min
0.3799
The next feed input at 290.35 + 18.56 = 308.91 min. This starts a cold thermal wave at
v Inter,F , u th v Inter,F
4.701 cm min which breaks through in another 25.53 min for total time
to cold breakthrough of 308.91 + 25.53 = 334.44 min.
The solute is hot, first at v Inter,purge u s 80 , v Inter,purge

18.60

u s 80 ,v inter,F

3.158 cm min after 18.56 minutes. Next solute step is

4.343
25.58
u s vinter,purge, 80

t
Exit Time Solute

18.56

u s,F 80 , vinter,F

120 4.343 18.56


tF

4.343 cm min and then

t purge

3.158

120

12.47 min

290.35 18.56 12.47

334.44

Since Exit Time Solute


entire time.

321.38 min.

breakthrough cold wave, the solute is at 80C the

Solute exits from 290.35+18.56=308.91 min to 321.38 minutes = 12.47 minutes & it exits at
superficial velocity of 8.0 cm min .
Mass Balance
All solute in = Solute out
t F vsuper A c c IN t out vsuper A c c out,AVG

tF

c out ,AVG

t out

290.35

c IN

0.0009 wt frac

12.47

0.02096 wt frac.

This is same as peak concentration in Example 18-3, but greater than x out,avg 0.00748. To
have same concentrations need to recycle the material exiting at feed concentration in counterflow system. NOTE: Counter flow system has advantages of not contaminating the product end
of the column and typically has less spreading of the zone.
18.D.9. New problem in 3rd edition. a.

u s,feed,M

vinter

vSuper
e

Kd

0.05 m s. vinter

v inter

RTK A,p

0.5 0.43

0.1163 m s

0.01712 m s

from Eq. (18-27) is same as Example 18-4, M 0.2128


Pressurization Step
Feed end (for pressurization) 0.75m (Measured from closed product end)
M

z after
which is 0.75 0.5584

y M after

0.003

Feed Step
u sfeed

0.01712 m s

0.75 m

4.0 atm

0.2128

1.0 atm
0.1916 m from feed and

4.0

0.5584 m

0.2128 1

1.0
7 sec

0.001007
0.11984 m

0.1916 m for pressurization step

= 0.3114 m. Does not breakthrough in first cycle. From 0 to 0.11984 m, concentration is y F .

456

Blowdown.
Measuring from closed top, z before

z after

0.4386 m

0.2128

1.0

0.5890 m
4.0
The far end of the feed wave does not get removed from the bed.
0.11984 or 0.75 0.11984 0.6302 m from closed end has
z after

1.0

0.6302

0.4386

0.75 0.3114

0.2128

0.8463 m, so it all exits. The mole fraction of this portion is

4.0

y after

y feed

The close wave

1.0

0.2128 1.0

0.003 2.9781 0.00893


4.0
Part of the feed that was pressurized also exits during blowdown.
1.0

0.2128

1.3431 z before
z before 0.5584 m from closed
4.0
(product) end. This is 0.75 0.5584 0.196 m from feed end. This gas entered at an unknown
pressure between p L 1.0 and p H 4.0. Can calculate this pressure from Eq. (18-28c)
This z after

0.75

p before

z before

p after

z after

z before

y after,press

0.003

y after,BD

0.001007

0.5584

4.0

4.0

1 0.2128

1.00003 atm

0.75

0.2128 1

0.001007
1.00003
This gas is depressurized to 1.0 & exits column

Exit from Col


y

After Pressurization Step.

0.2128 1

0.00300 or essentially the feed composition.

0.008933
0
.0030
time

Part b. Want z after blowdown

0.75, then z before

z after

p after
p before

0.2128

1.0
z before 0.75
0.5584
4.0
from closed end, which is 0.75 0.5584 0.1916 m from feed end. Want the feed to end at this
point. During constant pressure feed step, feed travels u s,feed t F 0.01712 t F . Then for
pressurization step z after (from feed end)

0.1916 0.1712 t F . From closed end this is

457

0.5584 0.1712 t F
0.5584 0.1712 t F

z after

z before

p before

0.2128

0.5584 or t F 0.
1
Thus, need a purge step if have feed step at constant pressure for complete cleanout.
18.D10. a)

pt.10 : z after

y after

0.4,

0.002

Travels,

0.75

p after

0.2128, p before

3.0

0.4
0.5

0.000876

1.015128
0.4 m

25.126s 1.0s for blow-down

0.01592 m s

3.0

.48

1
.2128

1.05128 atm

0.2128 1

3.0

b) Start with Arbitrary point at t = 1 sec (end repress) z after

p before

1
0.2128

2.4763 atm , y after

0.002

26.126s

0.48 (.02 from feed end)


3.0

0.2128 1

0.00172

.5
2.4763
Dist. Traveled @ t = 30s: 0.02 + 0.01592 29s = 0.48168 m
For blow-down: distance from closed end = 0.01832 cm

z after

0.01832

Purge: u M,purge

0.5

0.5

.2128 1.0

0.026824 , y after ,BD 0.00172


0.007048
3.0
3.0
0.01751 m s . Exits bottom column during purge (point 11)

(distance traveled)/upurge
18.D11.

.2128

31 s +

0.5-0.026824
0.01751

58.023s

If repressurize with product, bed remains clean.


Feed step is same as to point 3 (at 0.462 m from feed end) on Figure 18-13.
Blowdown then pt. 4 (0.056 m from top) and purge exits at pt. 8 (56.36s)
Product gas is cleaner (y = 0), but there is lower productivity less feed per cycle.
See Figure.

458

BD
3

y=0

y=0

y = 0.0082

8
18.D12. a) The clean bed receiving feed has a shockwave for Langmuir isotherm.

320 cm 3 min

v sup er

vsup er

r2

A c , where A c

6.366 cm min , vinter

vsup er

4 cm

50.2654 cm 2

6.366 10.434 14.669 cm min

v int er

u sh
1

Kd

p
s

q after

q before

c after

c before

c after

where c before

50 mol m 3 , q after

q
c

0, q before

0.190 50
1 0.146 50

1.1446 mol kg

14.669

u sh
1

0.566 0.57 1.0

0.566 0.43

0.434

0.434

t br

L u sh

1.1446

1820

50 cm 0.5843 cm min

0.5843 cm min

50

85.579 min

Outlet concentration is zero until t br then becomes 50.


Concentrated solution eluted by dilute soln. Gives diffuse wave for Langmuir isotherm.
v
u s u diffuse
1 e 1 p
1 e
a
1
K
p
d
p
2
1 bc
e
e

us
1

0.566 0.57 1.0

14.669
0.566 0.43 1820

0.19

0.434

0.434

1 0.146 c

1.74336

14.669
193.92
1 0.146c

Create Table.

459

18.D13.

50, u s

3.218, t

L us

50 3.218 15.537 min

0, u s

0.07497 cm min , t

40, u s

2.491, t

20.071 min

30, u s

1.737, t

28.779 min

c 15, u s

0.7052, t

70.898

0.2205, t

226.80

5, u s

de xtran, B

L us

666.93 min

fructose
(1)

CA v1

u A1

M1u port

CB v 2

u B2

M 2 u port (2)

C A v3

u A3

M 3u port

CB v 4

u B4

M 4 u port (4)

(3)

v F,sup er
1000 cm 3 min
, vF
2
40
e
4
CB vF
Solve eqs. (2) and (3) simultaneously, u port
CB
M2
M3
CA
v2

CA

v3

1
1

e
e

v2

v3
v4

KA

1
.6
1
0.23
.4

0.7435 , C B

M1
CA
M2
CB

u port

u port

0.97
0.7435
0.99
0.4914

3.03175 cm min

60
3.03175

M 4 u port

1.03 3.03175

CB

0.4914

KB

0.4914

19.791 min

6.1079 cm min ; V2,sup er

0.7435

1
.6
1
.69
.4

L t sw

3.03175

CA

V1,sup er

3.955 cm min : V1,sup er

1.01 3.03175

Recycle flow

1
1

3.03175

M 3 u port

1.9894 cm 3 min

0.4914 1.9894 cm min


0.4914
0.99
1.01
0.7435
L
t sw
u port

u port

v1

Vol Feed
D2
4

v F , v F,super

v1

D2
e

1988.176 cm 3 min

3070.15 cm 3 min

4.1184; V3,sup er

2070.14 cm 3 min

6.3547; V4,sup er

3194.19 cm 3 min

1988.176 cm3 min

460

V4,sup er

Check:

3194.19 1988.176 1206.0 cm3 min ,

V1,sup er

VD

VF

V4

V1

VF

F
V2

Extract Product

V4

M 1 u port

CB

CA

vF

M2

V3

M1
CB
CA
CB
M3
CA

.97
.4914
.7435
1.2060 , OK
.4914
.99
1.01
.7435
V1 3070.15 1988.18 1081.97 cm 3 min
3194.19 2070.14 1124.05 cm3 min

18.D.14. New problem in 3rd edition. From Eq. (18-40c) K K

KK

Andersons data:

1.2060

1.03
.4914

Raffinate Product

M4

M4
CB

2.9 1.3

KK

Li

KH

Li

2.2308

DeChows data:
K K H 2.63 1.26 2.0873
For the shockwave Eq. (18-46) holds for K+
Since resin is initially in H+ form, x K,before CK,before CT 0 and y K,before

a)

x K,after

CK,after CT

y K,after

CR ,K,after CRT

CR ,K CRT

0.

1.0
1.0
v inter

u sh ,K

y K ,after
1 C RT
K DE ,K
x K ,after
e CT

25 0.42

u sh ,K

y K ,before
x K ,before

44.84 min
1
2.2
1 0
u sh
1
1.0
0.42 0.1
1 0
Same for both sets of data since K K H does not enter into equation when initial and feed
contain only one ion.
b) C t 1.0, u Sh,K 9.542 cm min , t sh 5.24 min
c)

Ct

1.0, x K,before

yK
Andersons Data: y K ,before

y K ,after

0.2, x K,after

0.85. y K values depend on equilibrium parameter.

K KH x K
1

K KH 1 x K
2.2308 0.2

2.2308 1 0.2

2.2308 0.85
1

1.115 cm min, t sh

2.2308 1 0.85

0.3580
0.9267

461

u sh

25 0.42
1
2.2
1
0.42 1.0

1.0

0.9267 0.3580
0.85 0.2

10.662 , t K

L u sh

4.69 min

DeChows data:

y K ,before
y K ,after

2.0873 0.2
1

0.3148

2.0873 1 0.2
2.0873 0.85

0.9220

2.0873 1 0.85

25 0.42
L u sh 4.95 min
10.100 , t K
1
2.2
0.9220 0.3148
1
1.0
0.42 1.0
0.85 0.2
4.69 4.95
% difference
100 5.55%
4.69
d) There is a difference if either initial or feed contains both ions. System with higher
K K H had higher shock velocity.
u sh

18.D15. New problem in 3rd edition. Part a. u sh ,i

y i,after
1 c RT
K DE
x i,after
e cT

1
For both

Na & K ,

x i,after

y i,before
y i,after

t center

0
1.0

u i,sh
1

y i,after
1 c RT
K DE
x i,after
e cT

y i,before
x i,before

For both Na+ and K+: xbefore = 0.4 and xafter = 0.9. For Na+
K Na H xNa
(2.0 / 1.3)(0.4)
y Na ,before
1 ( K Na H 1) xNa 1 [(2.0 / 1.3) 1](0.4)

y Na , after

x i,before

25 0.42
5.186 cm min
1
2.2
1
0.42 0.5
L u sh 50 5.186 9.64 min

Thus same u sh , u sh

Part b.

x i,before

y i,before

K Na

1 ( K Na

xNa
1) xNa

(2.0 / 1.3)(0.9)
1 [(2.0 / 1.3) 1](0.9)

0.506

0.933

462

u sh ,Na

y Na ,after
1 c RT
K DE
x Na ,after
e cT

1
t Na

L / ush , Na

50 / 5.98

(25 / 0.42)
1(2.2)(1.0) 0.933 0.506
1
(0.42)(0.5)
0.9 0.4

y Na ,before
x Na ,before

5.98

8.36 min .

For K+ we obtain,

y K ,before

y K , after

KK

1 (KK

KK

1 (KK

u sh ,K

Part c.

(2.9 / 1.3)(0.4)

1) xK

L / ush , K

xK

(2.9 / 1.3)(0.9)

1) xK
v

50 / 7.054

(25 / 0.42)
1(2.2)(1.0) 0.953 0.598
1
(0.42)(0.5)
0.9 0.4

y K ,before
x K ,before

7.054

7.09 min .

1 c RT
dy
K DE
dx
e cT
dy Na

K Na

dx Na

dx Na

t Na

K Na

L u shNA

KK
1

xK

KK

K Na
x Na

dy K

.9

Li

KH

Li

KH

Li

Li

1 x Na

25 0.42
1 2.2
1
0.955
.42 0.5

u Na

5.409

1 xK

xK

KK

0.855, u K

Li

Li

KH

KH

Li

Li
2

1 xK

25 0.42
1 2.2
1
0.855
.42 0.5

5.979

2.0 1.3 1.538,

u Na

3.477,

t Na

14.38 min

KK
xK

dy Na

dx K

K Na

KK

2.9 1.3 1 .5

dx Na

0.955,

2.9 1.3

dy
dx

9.244 min

dx K

x Na

1 x Na

2.0 1.3 1 .5

dy K

x Na

K Na

2.0 1.3

dy Na

Part e.

0.953

1 [(2.9 / 1.3) 1](0.9)

u
1

Part d.

0.598

1 [(2.9 / 1.3) 1](0.4)

y K ,after
1 c RT
K DE
x K ,after
e cT

1
tK

xK

2.9 1.3

2.231,

uK

2.442,

tK

20.47 min

.9

463

dy Na

1.538

dx Na

dy K

0.538 .9

2.231

dx K

0.502,

1.231 .9

0.698, u

7.159, t Na

Na

uK

6.984 min
tK

9.5075,

5.259 min

Part f. The velocities and hence the derivatives are equal. Thus,
K Na

dy Na
dx Na

K Na

KH

Li

KH

Li

Li

1 x Na

KK

dy K

Li

dx K

KK

Li

Li

KH

KH

Li

Li

1 xK

With xNa = xK. The result from a spreadsheet is x = 0.35056


18.D16.

vint er

vsup er

15 0.40

MW p
f

37.5 cm s

28.9 g mol 50 kPa

1.0 kg

0.5832 kg m 3

m kPa
1000 g
298 K
mol K
q
kg toluene kg carbon
. Then, shockwave velocity is
is in
c
kg toluene kg air
v int er
RT

u sh

0.008314

Kd

q
y

s
f

37.5 cm s

u sh
1

0.6 0.65 1.0

0.6 0.35

0.4

0.4

1500 kg m 3 q 2
0.5832 kg m 3 y 2

q1
y1

37.5 cm s

u sh

For

1.975 1350.308

q2
y2

u sh ,1 : y1

0, y 2

0, q1

u sh 2 : y1
y2
u sh 2

q1
y1

0.0005, q 2

37.5

u sh ,1

At

2000 0.0015
1 2200 0.0015

L min

0.47619
0.104976 cm h

0.69767

37.5
0.69767 0.47619
1.975 1350.308
0.0015 0.0005

L min : u sh1t

1 2200 0.0005
0.00002916 cm s

0.47619
1.975 1350.308
0.0005
0.0005, q1 0.47619

0.0015, q 2

2000 .0005

0.00012539 cm s

0.451393 cm h

u sh 2 t 10 h where t is in hours.

464

Solve for

u sh 2 10

0.451393 10

13.03 h
0.451393 0.104976
cm
L min u sh1 t 0.104976
13.03 h 1.368 cm
h
Thus, for any column of partial length we will see a single shockwave exit the column.
v sup er 21.0
18.D17. v int er
52.5 cm s
0.4
e
pV n RT
u sh 2

Cinit

Since

u sh1

MW n

MW p

28.9 50

RT

1000 g kg 0.008314 298

C F , Get 2 diffuse waves


v int er

us
1

Kd

1
y
0.0010
0.00075
0.00050
2nd wave (0.00050)
0.00025
0.00

us

52.5
0.6 0.35

.6
0.65 1.0
0.4
0.4
q
u s cm s
y
195.31
0.0001991
284.799 0.0001365
453.515 0.00008573
- add
20 hours
832.466 0.00004671
2000
0.00001914

us y

0.5383 kg m 3

0.001

q
y

where

2000

1 2200y

52.5 cm s
q 1.975+1350.23 q
y
y
t L u s 25 u s

1500
0.583

125,581s = 34.8835 h
183,117.6s = 50.866 h
291,596.6s = 80.999 h
100.999 h
535,250.5 = 148.681 + 20 = 168.681 h
1285937.96 = 357.205 + 20 = 377.205 h

0.00075

0.00025

us y

us y
0

0.005
t

us y

0.0005

465

80.999

34.88
0.001

50.866

0
0.00075
0.0005

100.99

168.88

0.00025
t

18.D.18.
Part a.

. New problem in 3rd edition.


u S,G 11.12 S cm min is calculated in Example 18-9.

20
0.61 1.0 0.88

u S,F
1 0

8.416 , u

From Eq. (18-93), N

4Ru u S,G

From Eq. (18-78a)

Part b.

u S,F

L v E eff

u S,F

9.771

229.465

2 229.465 5.0 cm 2 min

2N E eff
v

114.73 cm

20 cm min

tG

L u S,G

114.73 11.25 10.20 min

tF

L u S,F

114.73 8.416 13.63 min

Part c. Eq. (18-80a),

K Ag

Li

uS

13.63 min

t ,F

K AgK

uS,G

0.39

18.D19.

377.2

KK

a) Ion wave: u total con

Li

vint er

8.5 2.9

vsuper

1/ 2

10.20 min

t ,G

229.465

1/ 2

0.673 min

1/ 2

1
229.465

2.93 , y Ag
e

3.0 0.4

Breakthrough of ion wave, 50 cm 7.5 cm min

0.900 min
2.93 x Ag
1 1.93 x Ag
7.5 cm min

6.667 min

466

b) Shock wave,

v int er

u sh

y Ag after
1 C RT
KE
x Ag after
e CT

before: x Ag

7.5 cm min
1 2.0 1.0 1.0

u sh
1

0.4

1.2

u s,Ag

x Ag

x Ag

0.5, u s

x Ag

0, u s

From spreadsheet:

1.0 .

y Ag

1.4516 cm min , t sh
v int er
1 C RT
dy
KE
dx
e CT

7.5 cm min
1 2.0
2.93
1
1.0
0.4 1.2
1 1.93 x

1.0, u s

7.5
12.208
2.93

50 cm

u sh

1.4516 cm min

1 C RT
KE
e CT
1

34.44 min

2
Ag

3.0965

cm
min

u_dif,Ag
3.097163211
2.852615804
2.598940969
2.337763116
2.071284133
1.802351071
1.53450084
1.27196632
1.019637556
0.782936124
0.567638906

2.0 eq L , c T

0.02 eq L , x Ca

K CaK C RT

0.6183 2.0

CT

0.02
75 cm, vsuper

61.83

K Ag -K

K Ag

1 x Ag

7.5
12.208
1 1.93 x Ag

, t out

us

7.5
50
1.8021 , t out
12.208
1.8021
1
3.86
7.5
L
0.5678 cm min , t out
13.208
us
xAg
1
0.9
0.8
0.7
0.6
0.5
0.4
0.3
0.2
0.1
0

Column: L

v int er

16.147 min

27.745 min

50
0.5678

88.0555 min

t_dif,Ag
16.14380534
17.5277722
19.23860549
21.38796684
24.13961427
27.74154314
32.58388572
39.3091764
49.0370324
63.86217015
88.08416667

Was 18D23 in 2nd edition. Table 18-5, K CaK


a.) c RT

x Ag before

1.0

c) Diffuse wave: u s

18.D20.

0 . after: x Ag

y Ag

y Ag before

K Ca

Li

K K Li

0.8 at t

5.2
2

2.9

0.6183

0.

shockwave .

20 cm min ,

vinter

20 .4

50 cm min

467

Feed:

u sh

0,

0.4, K E

v int er
, before: x Ca
C RT K E y
1
x
e CT

from Eq. (18-43) y Ca

u sh

1.0

50
1
2.0 0.971965
1
1.0
0.4
.02
0.8

x Ca

K Ca C RT
CT

0.8

0.971965
75

0.16407 cm min , t br

b.) Regenerate: at 500 min Ion wave at vint er


New

0 , after: x Ca

0, y Ca

u sh

50 cm min takes

75
50

457.1 min

1.5 min.

1.2366 y Ca unchanged. Use Eq. (18-43) with new value K Ca C RT CT .

0.9689677 . Obtain diffuse wave.


3

v imter
dy Ca 1.2336 1 y Ca 1 x Ca
u diffuse
where
3
C RT K E dy Ca
dx Ca
1 x Ca 1 y Ca
1
C T e dx Ca
(Wankat, 1990, Eq. (9-25b)).
dy Ca
50
At x Ca 0,
1.2366, u Ca
6.96088 cm min
1.0
2.0
dx Ca
1
1.2366
0.4 1.0
75 cm
t out
10.7745 min (slow wave)
6.96088
At x Ca 0.96897, and y Ca 0.971965

dy Ca

1.2336 0.028035

dx Ca

0.03103

1.96897

0.908386

1.971965

50
9.022 cm min , t out
2.0 1.0
1
0.908386
1.0 0.4
At x Ca 0.5, y Ca 0.534927

75

u Ca

dy Ca

1.2336 0.465073

dx Ca

u Ca

.5

18.D.21. New problem in 3rd edition.


vF
u port
, C Tol
M2 M3

C Tol

1.5

8.312898 min (fast wave)

0.97013

1.534927

50
2.0 1.0
1
0.97013
1.0 0.4

Cy

9.022

75

8.546 cm min , t out

8.546

8.776 min . (in-between)

1
1

e
e

Kd

0.132234
p

K Tol 300

468

0.0061 e 2175.2696 300

K Tol 300 K
C xy

.95
.10007

vF

0.6479 v F 0.6479 cm min , L u port t SW 64.79 cm


1.05
.132234
0.95 .6479
C xy
6.1447 cm min , v3 v 2 v F 5.14476
0.10017
0.6479
C Tol
.95
4.6547
0.132234

v2

M 2 u port

v1

M1 u port

v4

M 4 u port C xy

1.05 0.6479

v Tol prod

v2

v1

6.7914
0.10017
6.1447 4.6547 1.4900

v xy,prod

v4

v3

6.7914 5.14476 1.6466

vD
Check: vOut
18.D22.

0.10017

1.63627 0.68930 12.1092

u port

E eff

v sup er
us

v4

v1

6.7914 4.6547

v tol prod

ED

v xy,prod

u s2

dp
6 k M,c

20 cm 3 min

1 0

v
1

0.0105 e 2115.1052 300 12.1092

8.5972 , K xy 300 K

cm

3.1366 , v total in

2.035

D F

2.1367

vD

3.1367

vF

, where

6.366 cm min ,
15.915

2.1367,

dp

6 k m,c

k m,c a p

v sup er

OK.

15.915 cm min

7.821 cm min

E eff

0.15

Eq. (17-69) X

cm
7.821
min

0.6 0.69

5.52 min

CF

Argument of erf, a

0.4

8.063 cm 2 min

z ust

1 erf

4 E eff u s t
v int er

12

for step up

200 7.821 t
15.849 t

12

469

Step down:

X L, t 8

L u s t 8u s

1 erf

12

4 E eff u s t 8
v int er

323.04 7.821 t

Argument of erf , a

15.849 t

Total Solution X X
L
If t
25.573 min
us

X
If t

and X

2
L 8u s

1.998

4
X

31.2835

341.89

12

1
2

0.979235 0.0084
cF
See also Problem 18.G1.

cF

1
2

0.499 c

1.44473, erf a

12

1
2

50 0.970835

Cinitial

1 .983186

.998
24.975 .

0.95847

Kd

CF1 X z, t 17.5

CF1X z, t 28

1.63627 0.68930 12.1092

0.132234
p

K Tol 300

8.5972 , K xy 300 K
1

0.0084

48.54

0.0061 e 2175.2696 300

C xy (300K)

0.999

0.983186 , X

Cinitial

Tol

K Tol 300 K

2.773, erf

0.979236

0.970835 , c

18.D23. Was 18D24 in 2nd edition. Cout

12

532.13

468.663

1.95847

25.0 (for smaller t, can ignore X )

63.96

31.2765

29.575, a

1.69189, erf a

18.D.24. New problem in 3rd edition.


vF
u port
, C Tol (300K )
M2 M3
1
C
C

0.50 or c

1 2, a

0, X

xy

0 , c cF

7.358

12

279.6

For higher t, X = 1.0,

0.999

Peak at 25.575

12

123.03

0, a

1 1.000

33.575, a

us
1

126.792

0.0105 e 2115.1052 300 12.1092

0.10017

470

u port

.90
.10007

vF

1.4812 v F

1.10
.132234

M 2 u port C xy (300K)

v1

M1 u port C Tol (300K)

v4

v2

v1

vD

v4

Check: vOut

v4

v1

13.3084 cm min , v3
0.10017
1.4812
.90
10.0812
0.132234

13.3084 10.0812

v3

v tol prod

v xy,prod

v2

vF

12.3084

16.2655

0.10017
16.2655 12.3084

16.2655 10.0812

148.12 cm

3.2272

1.10 1.4812

M 4 u port C xy (300)
v xy,prod

u port t SW

0.90 1.4812

v2

vTol prod

1.4812 cm min , L

3.9571

6.1843,

7.1843 , v total in

vF

D F
vD

6.1843
7.1843

OK.

18.D.25. New problem in 3rd edition.


Zones 2 & 3 are same as in 18.D.24 since at 300 K
u port 0.6479 v F 0.6479 cm min , L u port t SW 64.79 cm

v2

6.1447 , v3

v1

M1 u port CTol 273 K

M1

5.14476

0.5 and M 4

and v 4

M 4 u port C xy 350 K

2.0 (reciprocal values).

K Tol 273K

0.0061 exp 2175.2695 273

17.612

K xy 350K

0.0105 exp 2115.1052 350

4.423

C Tol 273K

C xy 350K

v1

0.2135

1.63627 0.68930 4.423

0.5 0.6479 0.07259

vTol prod

0.07259

1.63627 0.68930 17.612

v2

4.4627 , v 4

1.6820 , v xy prod

v1

v4

v3

2.0 0.6479 0.2135


0.9260 , v D

v4

v1

6.0707
1.608

D / F 1.608

18.D26. a)

u sD

4Ru s
u sA
1.0
1 5.8

u s,B

, R

0.147059 , u

1.5 , u s A
1

1.0
s

KA

6.5

0.15385

0.15045

471

4 1.5 0.15045

Need N
b) t R ,A

t ,A

CA
C A ,max

17689 , L

0.0067873

884.45

uA

0.15385

uA

5748.88

95.813 min

tR

2
t

exp

0.05 N

884.45 cm.

95.813 min

12

exp

t,min
CA CA,max

12

0.7204 min

17689

t 95.813
2 0.7204

90
92
94
95
95.813
96
97
7.27E-15 8.3E-7 0.0421 0.52898 1.00 0.9669 0.2573

CA

0.33 X A L, t

CF

18.D27.

18.D28. a) u p

25.0 cm

L t center

35.4 min

b) Large-Scale system

1.0 0.33 X A L, t

1 .55 X A L, t .8t F

0.55 0 X A L, t

0.706 cm min , L MTZ,lab

u pt MTZ

t MTZ, LS

d 2p ,LS D eff

1.0

t MTZ,lab

d 2p ,lab D eff

0.12

t MTZ,LS

Independent of velocity

0.4t F

69.44 2.8

tF

0.706 2.8

1.9774 cm

69.44

194.44 min

v super
u p ,LS

u p ,lab

LS

v super
e

lab

12

u p,Ls

0.706

4
3

0.941 cm min

lab

L MTZ,larg e scale u p t MTZ


0.941 cm min 194.44 min 183.03 cm
For frac. bed use = 0.80 & symmetrical pattern,
0.5 183.03
0.5 L MTZ
L
457.6 cm 4.576 m , t br t center
1 Frac bed use
1 .8
t center

457.6

L up

486.27 min , t br

486.27

194.44

0.941
2
This is length of feed time if column is completely regenerated.
18D.29. K CaK
a.) c RT

K Ca

Li

K K Li

5.2
2

2.9

2.5 eq L , cT

t MTZ
2

389.05 min .

0.6183

0.03 eq L , x Ca

0.7 at t

0.

472

K CaK C RT

0.6183 2.5

CT

0.03

Column: L
p

Feed:

u sh

90 cm, vsuper

0,

shockwave .

51.525

25 cm min ,

0.39, K E

y Ca
(1 y Ca )

25 / .39

64.10 cm min

1.0

v int er
, before: x Ca
C RT K E y
1
x
e CT

from Equilibrium,

vinter

0, y Ca

0 , after: x Ca

K Ca K C RT

x Ca

CT

(1 x Ca ) 2

0.7

400.75

Solve this for unknown y value. I used a spreadsheet.


yCa 0.95128

u sh

64.1
1
2.5 0.95128
1
1.0
0.39
.03
0.7

b.) Regenerate: Ion wave at vint er


New

K CaK C RT

(0.6183)(2.5)

CT

1.1

0.22000 cm min , t br

35.0 / 0.39
1.4057 y Ca

old y and with new value K Ca C RT CT 400.75


,
find x Ca

90

409.10 min

u sh

89.74 cm min takes

90

1.003 min.

89.74

0.95128 unchanged. Use equilibrium with


y Ca
(1 y Ca )

K Ca K C RT

x Ca

CT

(1 x Ca ) 2

to

0.94251 . Obtain diffuse wave.


3

v imter
dy Ca 1.4057 1 y Ca 1 x Ca
u diffuse
where
3
C RT K E dy Ca
dx Ca
1 x Ca 1 y Ca
1
C T e dx Ca
(Wankat, 1990, Eq. (9-25b)).
dy Ca
89.74
At x Ca 0,
1.4057, u Ca
9.7631 cm min
1.0
2.5
dx Ca
1
1.4057
0.39 1.1
As an alternative can do numerical calculation of derivative. At x = 0, y = 0. x = 0.001, y = 0.001404 and
y / x (0.001404 0) / (0.001 0) 1.404 , which is reasonably close.

90 cm

9.22 min (slow wave)


9.7631
At x Ca 0.94251, and y Ca 0.95128
t out

dy Ca

1.4057 0.04872

dx Ca

0.05749

1.94251

1.95128

0.85169

473

89.74
15.049 cm min , t out
2.5 1.0
1
0.85169
1.1 0.39
From equilibrium, at the arbitrary value x Ca 0.5, y Ca
u Ca

dy Ca

1.4057 1 0.55544

dx Ca

.5

1.5

15.049

5.981min (fast wave)

0.55544

0.95282

1.55544

89.74
2.5 1.0
1
0.95282
1.1 0.39

u Ca

90

13.695 cm min , t out

90
13.695

6.572 min .

This is in-between the other two waves.


c. To not have a diffuse wave must have

K CaK C RT

(0.6183)(2.5)

CT

CT

1.0

This requires CT > 1.546.


18.D30. New Problem in 3rd edition. K K

KK

Li

KH

Li

DeChows data:
K K H 2.63 1.26 2.0873
a.) This will be a shock wave since K+ is more concentrated in the feed to the column than it
is initially and KK-H > 1.
v inter

u sh ,K

y K ,after
1 C RT
K E ,K
x K ,after
e CT

Ct

1.0, x K,before

yK

y K ,before
y K ,after
u sh

tK

0.2, x K,after

0.85. y K values depend on equilibrium parameter.

K KH 1 x K

2.0873 1 0.2
2.0873 0.85

x K ,before

K KH x K

2.0873 0.2
1

y K ,before

2.0873 1 0.85

0.3148
0.9220

25 0.42
0.9220 0.3148
1.0
0.85 0.2

1
2.2
1
0.42 1.0
L u sh 49.5 min
All three times are the same for the shock wave.

10.100 cm/min,

474

b.)
This will be a diffuse wave since K+ is less concentrated in the feed to the column than it
is initially and KK-H > 1.

v inter

u diffuse,K

dy K
1 C RT
K E,K
dx K
e CT

dy K

K KH

At xK = 0.15, dxK

u diffuse,K

25 / 0.42
2.2(1) dy K
1
(0.42)(1.0) dx K

(1 ( K KH

2.0873
1) xK )

25 / 0.42
2.2(1) dy K
1
(0.42)(1.0) dx K

1.543

[1 (1.0873)(0.15)]2
59.524
dy
1 5.238 K
dx K

59.524
1 5.238(1.543)

6.554cm / min

Thus, at xK = 0.15, tK = L/udiffuse,K = 500/6.554 = 76.29 min. Then at xK = 0.5 we obtain

dy K

K KH

2.0873

dxK

(1 ( K KH 1) xK )
59.524
u diffuse,K
dy
1 5.238 K
dx K

0.876
[1 (1.0873)(0.5)]2
59.524
10.65cm / min
1 5.238(0.876)

Thus, at xK = 0.5, tK = L/udiffuse,K = 500/10.65 = 46.94 min. Then at xK = 0.8 we obtain

dy K
dxK
u diffuse,K

K KH
(1 ( K KH

59.524
dy
1 5.238 K
dx K

1) xK ) 2

0.5970

59.524
1 5.238(0.597)

14.42cm / min

tK = L/udiffuse,K = 500/14.42 = 34.67 min.


18.D31. New problem in 3rd edition. a.

vSuper

10

vint er

10 .4

25,

0.4,

30.0

475

c RT

2.4,

1.10,

KK

2.9
Na

y K ,after
1 c RT
K DE
x K ,after
e cT

b.

u sh,exp t

c.

L MTZ

L t center

u sh t MTZ

L MTZ l arg e scale

1 2.4
1.0
.4 1.1

x K ,before

1 0
1 0

7.75 min . , t center ,measured

7.31 min

7.31 7.75

100
6.00%
7.31
30 7.31 4.10 cm min .

4.10 7.57 7.06

Frac. bed use (symmetric wave)


d.

25

y K ,before

3.783 cm min, t center,exp ected


% error

1.45

2.0

v int er

u sh ,K

u sh,K

cT

L MTZLab

2.093 cm

1 0.5 L MTZ L

d 2p v Super D eff

16 d 2p

l arg e scale

d 2p v Super D eff

0.965
d 2p

Lab

Lab

Lab

200 D eff

L MTZ,Lab

100 D eff

With same beads assume no change in D eff .

L MTZ,larg e scale

16 2 2.093 cm

frac bed use

1 0.5 L MTZ L 1 0.5 66.98 200


t center

Breakthrough start time

v inter,large scale

u sh

u sh ,lab ,exp tl

v inter,lab scale

Breakthrough start time

0.5 t MTZ

L u sh

2580 ft 3 , h=2580/860=3ft.,

End
View

0.833

0.5 L MTZ u sh

8.2

[200 0.5(66.98)] / 8.20

18.F1. New problem in 3rd edition. Constraints: w L

wLh

66.98 cm

860 ft 2

p0

20.31min

T max

6 atm

500 C

932 F

88.14 psia

73.14 psig

Weight vessel

Di

ts

0.8 D i t s

Seider etal,
s

Eq.16 59

w
Seider etal. (2004), Eq. (16.61)

476

Pd

exp

0.60608 0.91615

Wall thickness

tp

(Eq. 16.60)

Relate w to h.:

n p0

96.66 D i

2SE 1.2 Pd

2 13,100 1

96.66 psig

3.7057E 3 D i

1.2 96.66

490 lbm ft 3 0.284 lbm in 3 p. 529 (Seider et al, 2004)

cos

cos

90
D

Pd D i

0.0015655

S 13.100 psi p. 529 with SA-387B steel, E = 1.0

where

Weight

n p0

3.7057E 3 D

0.5h

1.5

r
0.5w

r
w

D
w

cos 90

860
D cos 90

cos

D in ft. (In Spreadsheet A cos

cos

cos

3 D

cos

0.8D

3D

3.7057 E 3 D 490

In Spreadsheet, angle is in radius


90
2
D
Weight
Width
L
3.5 33655
1.80
477.0
3.7 31358
2.17
397.1
3.8 30735.9 2.33
368.7
3.9 30325.
2.49
345.1
4
30070.31 2.65
325
4.1 29933
2.7947 307.7
4.2 29889
2.939 292.6
4.3 29918
3.08
279.2
6
35103
5.196 165
8
44837
7.42
115.96
10 56197
9.54
90.15
12 68940
11.62 74.02
14 83144
13.67 62.88
Goal seek L = 60
D = 14.64 ft Weight = 88052.75 Width =
14.333 L = 60
From Seider et al, p. 527: Cp

Cv

(Eq. 16.53) horizontal


(Eq. 16.55)

CpL

From p. 531, Fm

Fm C v

Bare module factor, FBm

CBm

0.20294

118,323 Cp

1.0 in 2000

Cp

144, 711 in mid 2000

118323 2724 121047

3.05 for horizontal

Cp Fm

Absorbent: p. 553 Cp

0.04333 n w

2724

1.2 for low-alloy steel, C v

Installed Cost: Calc C p with Fm

0.054266 ft

CPL in mid 2000 (MS = 1103)

exp 8.717 0.2330 n w

1580 D

ts

1.0, 2000

$60 ft 3 , Cp

3.05

1.0 1.0 1.2

60 2580

$393, 400

$154,800

477

18.G1. Was 17G1 in 2nd edition. Figures are labeled 17G1.

478

479

480

18.G2.

Was 17G2 in 2nd edition.


a.) With QDS with 50 nodes find t center

t MTZ
18.G3.

6.0 3.13

4.52 min
2.87 min

Was 17G3 in 2nd edition.

Find

D F 1.0. D 141.55 E R
CA 0.343 and CB 0.219

Eq. (17.31a)

u port

a)

M 2B
CB

M 3A
CA
141.55 cm 3 min

vF
e

u port
t sw

v1,int er

v1,sup er

Dc

0.4

10

4.5057 cm min

4.5057
1
0.219
L u port

CA vint er

Recycle Rate

vF

F.

u A1

2.7295 cm min
1
0.343
50 2.7295 18.32 min

M1u port

2.7295 cm min

2.7295

7.9577 cm min
0.343
0.4 v1,int er 3.18308 cm min

3.18308

10

250 cm 3 min

Obtained raffinate = 96.6% and extract = 94.3%.


b) One approach is to keep a symmetric cycle.
Then D = 283.1 and
E F
E R
212.325
2
Flow optimizer can be used to give t sw ~ 9.1 and Recycle rate ~ 500. Depending on
values obtain raffinate and extract > 97%.
18.G4.

Was 17G4 in 2nd edition. Figure below is labeled 17G4.

18G5.

Was 17G5 in 2nd edition. Figure below is labeled 17G5.

481

482

483

18.G6.

Was 17G6 in 2nd edition.


a.

k m,a p

1.5 min1 , L

25.0 cm

484

v sup er

20.2 ml min

Eq. (18-66)

2.0 m 2 4

6.366 cm min
6.366 cm min

19.1
1.5 min -1
Satisfied, but close. Thus some bypasses but most undergoes equilibration.
18.G7.

25.0 cm < 4.5

Was 17G7 in 2nd edition. Figure is labeled 17.G7.

485

18H1. New problem in 3rd edition. Spreadsheets with numbers and formulas shown.

486

487

18.H.2. New problem in 3rd edition. The spreadsheets are shown on the next pages. They are based on
the previous, but includes both a step up and a step down. Because of the quirk in Excel not
allowing negative arguments, it was set up with multiple solution paths. The correct solution
occurs when there are numbers.
Time, min

15

20

22.5

25

.0134

1.798

24.96

25.5726 27.5
25.0

30

33.575

42.32 48.52 24.97

35

37.5

11.13 1.114

40
.040

488

489

490

Você também pode gostar